TJPT 4y2s Nursem

You might also like

Download as pdf or txt
Download as pdf or txt
You are on page 1of 121

CHN WITH IMCI

Date: December 19, 2022


Time: 8AM - 2PM Bag Technique
Concept: CHN W/ IMCI →Tool to render effective nursing care
Lecturer: PROF. JESSIE DACLIS
Principles:
CHN: outside the hospital, most of the – MOST IMP! Prevent the spread of infection
patients are healthy. Can be private. – last priority: px with contagious diseases
– save time & effort
Primary responsibility: – performed in a variety of ways
→Health education – not overshadow concern for patient
→Health teaching
Focus: health promotion & illness
prevention

Health promotion - ↑ level of health,


healthy today, should be healthier
tomorrow
→Healthy lifestyle
Diet
Exercise
Rest
Stop smoking

Illness prevention - maintain health,


healthy today, healthy tomorrow
→Specific protection. eg.
condom, ppe, handwashing

PUBLIC HEALTH NURSING (PHN)


- Salary from government

SG15 entry-level position, applicable to all


Family care plan
government employees. 36k/month
– use nursing process, systematically

CLIENTS OF CHN
APIE
Individual - patient with specific ek ek
Family - home visit
ASSESSMENT: data collection
Group/population/aggregate - shares
First level assessment
common characteristics. Eg. pregnant, post
– what are the problems
partum, elderly
Categories
Community - primary client
a. Wellness state: healthy. Eg. regular
exercise
HOME VISIT
b. Health deficit: (+) disease, disability.
Principle:
Eg. high blood, diabetes
Purpose/objective: help the client
c. Health threat: ↑ risk for illness & injury.
Use of available information:
eg. wet floor, scattered things
Needs: not the financial, only health
d. Foreseeable crisis/stress points:
Involve: active participation
anticipated problem → developmental
Flexible
/ social. Eg. migration, pregnancy,
college, adjustment
Public Health Bag
→ carry
Examples of types of nursing problem
→ equipment that is necessary - syringe, NO
(1) Wellness state
bp apparatus (not sterile)
(2) Health deficit - pregnant w/ eclampsia
1
CHN WITH IMCI

(3) Health threat - pregnant taking ↑ Walang antibiotic - S


sodium
(4) Foreseeable crisis - pregnant SANITATION
→ environmental
Second level assessment PD 856: sanitation code of PH
– why? → general
PD 825: garbage disposal act. From 8oclock
PLANNING 2 5oclock may pasok, need magtapon basura
Goal setting - SMART
Construct plan of action Food establishment
Choose nursing intervention Sanitary permit: owner
Develop an operational plan - prioritization Health certificate: Food handlers
Xray, stool, blood
4 criteria of prioritization wt.
(1) Nature of problem 1 Class A: excellent
(2) Modifiability of problem 2
(3) Preventive potential 1 Class B: very satisfactory
(4) Salience 1
TOTAL 5 Class C: satisfactory
* Lowest score, lowest priority
4 rights in food safety
Nature of problem - type of nursing problem (1) Right source: fresh
Modifiability of problem - solution of the (2) Right preparation: wash
problem (3) Right cooking: 70degrees
Preventive potential - future recurrence of (4) Right storage
problem Room temperature: max 2 hrs
Salience - perception of client to the problem Cool: 10degrees
Warm: 60degrees
IMPLEMENTATION
3 types of nursing intervention MNEMONICS: SPCS - saint paul college
school
Independent - scope of nursing practive, no
doctor
Levels Water Facility
Dependent - requires doctors order
Level 1 approved
Interdependent - collaborative care – point source
– developed spring
EVALUATION – protected well
→ check outcome of care – avoid > 250 meters away from the source
→ QAE - quality assurance elements – should be clean
(1) Structural elements - physical
settings. Public or private. Manpower. Level 2 approved
Money. Material. – communal faucet
(2) Outcome elements - changes – stand posts
resulting from nursing intervention. – 1:4-5 household
Eg. paracetamol given. From 39 to 38.
1 degree change is the outcome Level 3 approved
(3) Process elements - steps of nsg – individual household connection
process. – waterworks system
– nawasa, maynilad
Q’s – cleanest among all levels, rigid water
Nagpaculture sensitivity for antibiotic - O treatment
Kumain na ba before giving antib - P

2
CHN WITH IMCI

TOILET FACILITY ASSESSMENT


Danger sign (4)
Level 1 Convulsion/seizure
– nonwater carriage Unable to drink/feed
pit latrines (25 meters away from Vomits everything
home) Abnormal sleepy/lethargy/stupor
– small amount water toilet facility **Atleast 1 danger sign = very severe
pour flush toilet ds.

Level 2 Main symptoms (4)


– water sealed/ flushed type → septic tank DOB/cough
Diarrhea
Level 3 Fever
– level 2 but better Ear problem
– sewerage system → tx plant Mnemonic: DeDe ni FE
– cleanest, rigid treatment process Minor symptoms:
Malnutrition
Anemia
HIV status
Mnemonic: MalAneH
IMCI - INTEGRATED MANAGEMENT
CHILDHOOD ILLNESS MAIN SYMPTOMS
→ 5yrs below
DOB/COUGH
1. Greet the client
2. Ask
CLASS ASSESS TX
Problem/s
✓ Common childhood prob Severe pneu – Stridor → – FAB(IM) →
Ø Rare / very severe harsh sound ampicillin
Ø Trauma disease during Gentomycin
Ø Genetic inspiration – refer to
– danger doctor
Age
sign (1)
Sick infant: 0-2mo
Sick child: 2mo-5yrs Pneumonia – fast – oral
Visit breathing antibiotic
Initial: new problems – chest – amoxicillin
Follow up indrawing for 5 days
*atleast 1

No Cough Home:
CASE MANAGEMENT PROCESS pneumonia breastmilk, a
medication
A C I T C Fol in IMCI

Assess Fast breathing:


Classify 0-2 mo: 60 ↑
Pink: severe→ hospital, except severe 2-12 mo: 50↑
dehydration 12-60 mo: 40↑
Yellow: moderate → RHU
Green: mild → home Chest Indrawing
Identify treatment When a child Breath in, the Lower Chest wall
Treat the child goes in
Counselling
Follow up

3
CHN WITH IMCI

QUESTION #1 DEHYDRATION
– 2 or more signs should be present
– ORS - orisol
CLASS ASSESS TX

Severe – ASIS – plan C: RHU


DHN very slowly – IVF →
LR(isotonic) / NSS

Some – RIDS – plan B: RHU


DHN slowly – ORS amount:
75ml x wt(kg) – 4
hrs
Answer: No danger sign. 18 months, 41
No Not Plan A: HOME
breaths per minute = fast breathing. DHN enough Feeding: breast
PNEUMONIA. signs Fluids - ORS
Zinc→ daily for 14
QUESTION #2 days
When to return

SOME SEVERE

Restless Abn sleepy/lethargy


Irritable Strupor

Drink eagerly/thirsty Inability to drink/feed

Sunken eyes Sunken eyes

Skin turgor: Slowly < Skin turgor: Very


2sec slowly > 2 sec
Answer: 10 months, 55 breaths/ min = fast.
With stridor → Severe Pneumonia Notify immediately

Other tables:
DIARRHEA
1Pink sign + 1Yellow sign = Pink classification
→ 3 or ↑ loose stool/day
*DHN = dehydration
DHN table:
CLASS ASSESS TX 1Pink sign+1Yellow sign = yellow (some
DHN)
Severe (+) DHN – refer to
persistent > 14 days doctor
diarrhea – Vit A QUESTION #1
(retirol)

Persistent (-) DHN – Vit A


diarrhea > 14 days (retirol)
– fluids

Dysentery – blood in – cifloxacin


stool for 3 days
(shigella)
● Vit A is a medication in IMCI used for Answer: P+Y= yellow, some. blood stool,
strengthening of mucous membrane dysentery. Abn sleepy, severe. Thirsty, some.
● No one dies from diarrhea, it’ll worsen Skin turgor, slow, some.
turning out to be dehydration

4
CHN WITH IMCI

QUESTION #2
ayaw niya
kumain ng lumi
sa canteen

Malaria (-) blood Paracetamol if ≥


unlikely smear 38.5° C
(+) s/sx
measles
Answer: SEVERE. Y, restless.Y/P, sunken
eyes. P, not able to drink. P, skin turgor v
MEASLES
slowly.
– “tigdas”

QUESTION #3 CLASS ASSESS TX

Severe – clouding – FAB(IM) →


complicated of cornea ampicillin
measles – deep Gentomycin
mouth ulcer – refer to
doctor

Measles – pus → tetracycline


with eye & discharge ointment
mouth – mouth → gentian
complication ulcer violet
Answer: SEVERE. Y/P, sunken eyes. P,
difficult to feed. P, v slowly turgor. There is no Measles – red eyes Paracetamol if
very severe dehydration!! – runny ≥ 38.5° C
nose
– cough
– rashes

FEVER
Axillary: ≥ 37.5° C DENGUE
Rectal: ≥ 38° C CLASS ASSESS TX
High Fever: ≥ 38.5° C, paracetamol
Severe (+) tourniquet – Plan B/C
3 causes: dengue test – refer
with high – persistent – ORS/IVF
Malaria fever abdominal pain
Measles – Persistent
Dengue vomitting
– bleeding, low
platelets count
MALARIA: – slow capillary
refill < 3 secs
Risk: malaria-endemic place (past 4 wks)
Blood transfusion (last 6 mo) Dengue (-) No aspirin
unlikely
CLASS ASSESS TX

Very – stiff neck – FAB(IM) →


severe – danger ampicillin
febrile ds/ sign Gentamycin
malaria – refer to doctor

Malaria (+) blood – oral


smear antimalarial
(-) s/sx drugs
measles – artemether -
lumefrantine
Arte ni meter

5
CHN WITH IMCI

EAR PROBLEM
CLASS ASSESS TX

Mastoiditis – tender – FAB(IM) →


swelling Ampicillin
behind ear Gentamycin
– refer to
doctor

Chronic ear – no ear → quinolone


infection pain otic drops 14
– pus days
discharge > – wicking -
14 days antibiotic
– no
cottonbuds

Acute ear – pus – wicking


infection discharge – antibiotic
<14 days amoxicillin for
5 days
– ear pain – paracetamol

No ear (-) (-)


infection

6
CHN WITH IMCI

CHN W/ IMCI PRE-TEST C. Family


D. Individual client
1. A home health nurse is made aware that a
client and family are about to lose their home 7. The most important role of a community
due to foreclosure. The nurse contacts the health nurse is in the area of:
social service department of the home health A. Change agent
care agency for assistance in finding housing B. Health education
for the family. The actions taken by the nurse C. Advocacy
best exemplify which of the following roles of D. Care provider
the community health care nurse?
a. clinician 8. When you have already organized,
b. advocate follow-up visits are likewise needed. Which of
c. consultant the following should you do first?
d. researcher A. Explain the purpose of the visit
B. Wash your hands and perform the
2. During home visits, Ruthie identified cases necessary procedure
of severe dehydration among children and C. Do an environmental surveillance
immediately referred them to the district D. Greet the patient/resident introduce self
hospital. Which among the following roles is
she performing? 9. During a home visit, socialization is
a. Clinician important in order to:
b. Researcher A. Do procedures
c. Leader B. Establish rapport and put the client ease
d. Health educator C. Identify the needs of the client
D. Evaluate the visit
3. Mrs. Tadejo came to the health center and Situation – While working in the community,
asked about ways on how to conceive a child. nurses observe various situations that require
She has history of repeated miscarriages. attention.
This condition falls under:
A. Heath deficit 10. Nurse Sussy was doing case finding in a
B. Health threat barangay in the rural communities of Cuilon.
C. Foreseeable crisis Her target is to identify undocumented
D. Stress point Hansens Disease cases. Seeing her in
uniform, a woman remarks. “You’re not a
4. It includes stressful occurrences such as nursing student are you? I have may chores
death or illness of a family member: to do today, please!” and ignores the nurse.
A. Heath deficit Sussy’s IMMEDIATE course of action would
B. Health threat be to:
C. Foreseeable crisis a. Visit the next house so as not to waste the
D. Stress points day’s activity and return to re-visit this home
after a day or two.
5. This refers to the tool which enables the b. Communicate the incident to the Barangay
nurse during her visit, to perform a procedure Captain and get him/her to join her in her next
with ease and deftness, to save time and visit.
effort, with the end view of rendering effective c. Ignore the woman’s behavior, turn around
nursing care. and simply leave her alone
A. Bag technique d. The feelings and behavior manifested by
B. Public health bag the woman and get to the root causes of such
C. Home visit a behavior.
D. Black bag
11. The following are nursing interventions to
6. The basic unit of care in CHN is: address the problems on parasitism EXCEPT:
A. Community A. Treatment of patients with the broad
B. Groups spectrum antihelmintics

7
CHN WITH IMCI

B. Stress that vegetables should be


thoroughly washed especially if eaten raw
C. Teach proper disposal of stools and stress
that is shouldn’t be use a fertilizer
D. Encouraging hand washing before and
after eating

Situation – Carol is 15 months old, weighs


8.5 kgs, is not eating well and is unable to
breast feed. She is not vomiting, has no
convulsion and not abnormally sleepy or
difficult to awaken. Her temperature is 38.5
C. Use the Integrated Management of
Childhood Illness or IMCI strategy.

12. If you were the nurse in charge of Carol,


how will you classify her illness?
a. febrile with a general danger sign
b. very severe disease
c. severe pneumonia
d. severe malnutrition

13. Which of the following signs is considered


to be a general danger sign in Carol’s case?
a. no vomiting
b. has no convulsion
c. temperature of 38.5 C
d. unable to breastfeed

14. Which of the following signs is NOT


included in the general danger sign?
a. vomits everything
b. unable to drink or breast fed
c. has had convulsion
d. fast breathing

15. Which of the following steps is NOT


included in the IMCI strategy?
a. identify treatment
b. counsel and follow up
c. referral
d. assess and classify

8
CHN WITH IMCI

CHN W/ IMCI POST-TEST d. encourages others to speak for the main


client to get an unbiased view
SITUATION: The public health nurse
analyzes the data collected in community 6. Ana has been caring for a child in the
health assessment in accordance with the home. She has a tracheostomy tube and is
nurse’s conception of the source of the oxygen dependent. Ana prefers to do the
problems and needs that can be met through care early in the morning due to a heavy
nursing interventions. caseload. The family requests that care be
done when the child is awake by 10am. The
1. What category of health problem occurs BEST response by Ana would be:
when there is a gap between actual and a. “I will see if I can find another nurse who
achievable health status? can meet your preferred schedule.”
A. Heath deficit b. “If I give the care at 10am, other children
B. Health threat needing my care may not be attended to.”
C. Foreseeable crisis c. “I’m sorry, I wish I could, but I cannot do so
D. Health need due to heavy workload.”
d. “Let’s look at what time is in the child’s best
2. These are conditions that promote disease interest and then we can work out a better
or injury and prevent people from realizing schedule.” – flexibility
their health potential.
A. Heath deficits 7. Which of the priority conditions will be
B. Health threats determining factors on the frequency of home
C. Foreseeable crisis health visits?
D. Stress point A. Acceptance of the community of the said
health practice interventions
3. In an epidemiological study conducted in B. the level of health need of the family
Barangay C, it was noted that the children C. The result of the studies conducted and
eligible for basic immunization are not level of family understanding
adequately immunized against Hepatitis B D. Administration of medication and the
and Measles. This situation is considered: schedules set
A. Heath deficit
B. Health threat Situation – One of your responsibilities as a
C. Foreseeable crisis community health nurse is to conduct home
D. Stress point visits to evaluate the health condition of
families and communities
4 . With the stated problems, the nurse and
the community prioritizes these using a set of 8. In every home visit, you are not only
criteria. Which of the following criteria refers concerned with the client but you should also:
to the probability of reducing, controlling, or A. Pay particular attention to the economic
eradicating the problem? status of the family
A. Modifiability of the problem B. Observe the family and environment
B. Preventive potential C. Put emphasis on the children’s education
C. Magnitude of the problem D. Observe the family’s spiritual practices.
D. Nature of the problem
9. In your community home visits, you have
5. When beginning a relationship with a scheduled several cases. Who is your LAST
family on home visit, the community health priority?
nurse: A. Family planning defaulters
a. is accepting and listens carefully to B. PTB case on D.O.T.S.
problem of all family members C. G4P3 3 day Post Partum
b. allows the discussion to be open and D. G3P2 36 weeks AOG
non-directional
c. focuses on the problem of one of the family Situation – One of your responsibilities as a
member community health nurse is to conduct home

9
CHN WITH IMCI

visits to evaluate the health condition of C. Expected consequence of the problem


families and communities D. Cooperation and support of the community

10. Which of the following signs should NOT Situation– Management of resources and
be included in deciding that “the child vomits environment includes every potential and
everything”? existing resources which the nurse can utilize
a. child experience occasional vomiting to promote and maintain health, prevent
b. when offered fluids, may not be able to illness, and assist in the task of cure and
drink rehabilitation. The IMCI chart prescribed by
c. maybe too weak to drink and eat at all the World Health Organization is one such
d. child is not able to keep anything down at resource. The following questions apply.
all (what goes down comes back)
Situation: Applying IMCI as an approach to 15. The Integrated Management of Childhood
the management of childhood illnesses Illness (IMCI) chart provides the necessary
improves nursing care of the child under procedure when identifying the appropriate
general conditions. interventions to be done. However, the
community health nurse should be aware that
11. Vitamin A is given to a child with measles the following factor should be considered in
because it: utilizing the case management chart:
a. resists the viral infection in the eyes, cell a. age of the child
layers in the lungs gastrointestinal tract, b. chief complaint
mouth and throat c. danger signs
b. reinforces the body’s immune response d. problem of the child
c. is needed for supplementation
d. helps the immune system prevent other 16. In the IMCI classification tables, color
infections yellow indicates that a:
a. child needs an appropriate antibiotic or
12. The child’s symptoms are under the other treatment
classification of severe dengue hemorrhagic b. child does not need specific medical
fever of the IMCI strategy. Which of the treatment
following treatments should be done c. referral or admission is needed
IMMEDIATELY? d. chief complaint that needs an urgent
a. observe the patient at the health center attention
and refer urgently if the condition worsens
b. give fluids or ORS and refer urgently to the Situation– Karina, 5 months old, weighs 5.2
hospital kgs, temperature 38 C., is brought to the
c. immunize the child before referral center. Her mother says she is not eating
d. give the first dose of appropriate antibiotics well, feels hot to touch, able to drink, no
and refer urgently to the hospital vomiting, no convulsions and not lethargic, no
cough. They live in a malaria risk area. Her
13. Health deficit is another component of a fever started 2 days ago. She has no signs of
health problem. Which is an example of measles, no stiff neck or runny nose.
health deficit?
A. Father with active tuberculosis 17. How will you classify Karina’s illness?
C. a pregnant mother a. plain fever
B. No regular health supervision b. malaria
D. untimely disability and death c. sever febrile disease
d. fever: no malaria
14. One family presents several problems.
Which of the following criteria will you apply 18. The following treatments are appropriate
to determine the priority health problems? for Karina EXCEPT:
A. Family involvement in identifying the a. follow up in 2 days
problem b. give oral anti malaria drugs
B. Modifiability of the problem c. advise when to return immediately

10
CHN WITH IMCI

d. give 1 dose of paracetamol for temp. 37.5 23. The nurse should consider the following
C assessing the child for chest in drawing
EXCEPT:
19. After assessing Junjun, your nursing A. Chest in drawing should be present at all
diagnosis is dehydration secondary to time
diarrhea. ORS was administered. Which B. The lower chest wall does not go in when
health instruction regarding ORS therapy will the breaths in
you give the mother once the diarrhea has C. The lower chest goes in when the child
stopped? breaths in
a. Increase the administration of ORS D. The child should be calm
b. Return to the clinic
c. Stop the administration of ORS 24. If a child is 2 months to 5 years. Which of
d. Decrease the administration of ORS the following steps is NOT appropriate in
checking for general danger signs?
Situation – the integrated Management of A. Ask if the child has fever
Childhoods illness of IMCI technology is a B. Ask if the child is feeding?
World health organization strategy which C. Look and see if the child is abnormally
aims to curb the growing infant/child mortality sleepy or difficult to awaken
and morbidity concerns in many countries D. Ask if the child has convulsion
especially the 3rd world economies like the
Philippines. This program integrates many Situation–Diarrhea in young children may be
dependent health programs to singular life-threatening and requires immediate
approach. Safety and quality would be any attention.
nurse’s focus. The following questions apply.
25. Joel, 3 years old, has had diarrhea for 5
20. James is an 18 month old child who has days and is irritable. He has sunken eyeballs
cough for 7 days with no general danger but has no blood in his stools. He drinks
signs with temperature 37.50 C and eagerly when offered liquid. Skin pinch at the
respiratory rate of 41 breaths/minute. How abdomen shows slow return. Using
will you classify jame’s breathing? Integrated Management of Childhood Illness
A. Slow breathing (IMCI), Joel’s illness maybe classified as:
B. Fast breathing A. No dehydration
C. Normal breathing B. Some dehydration
D. Very fast breathing C. Severe dehydration
D. Persistent Diarrhea
21. James illness can be classified as:
A. Pneumonia 26. Which of the following treatments should
B. Very service disease be immediately considered in Joel’s case?
C. Severe pneumonia A. Reassess the child after 4 hours and
D. No pneumonia classify his dehydration
B. Give 900-1,400 ORS during the first four
22. After 3 days, the nurse notes that hours
james has chest in drawing and stridor. His C. Explain to the mother how to prepared
mother returned him to the health center ORS
immediately. The nurse should: D. Continue feeding
A. Change the medicine to the second line
antibiotics 27. The nurse should know that the
B. Advice mother to observe the child and examination of drinking water by the
continue giving the antibiotics government or non-government personnel
C. Give first dose of antibiotics and refer must be coordinated by the municipality
urgently through the RHU. Certification of potability of
D. Observe the child in the center an existing water source is issued by the:
A. Sanitary Engineer
B. Municipality

11
CHN WITH IMCI

C. Secretary of Health or his representative


D. DOH

28. Level 1 approved type of water facility is:


A. Communal faucet and system or individual
house connections
B. Point source
C. Waterworks system or individual house
connections
D. Unapproved type of water facility

29. Approved type of toilet facilities may need


water or not depending on receiving space.
What type of toilet is without need of water
A. Pit latrines
B. Water sealed
C. Flush toilet
D. Aqua prives

30. The level of toilet facility commonly found


in highly urbanize communities that have
sewerage systems and treatment plants are:
A. Level I
B. Level II
C. Level III
D. Level IV

12
TRICIA JESHIEL P. TORRES, RN OBSTETRICS

Date: December 26, 2022 Continued vomiting → fluids &


Time: 9 AM - 4 PM electrolyte imbalance → respiratory
Concept: OB alkalosis
Lecturer: PROF. ARCHIE ALVIZ Quickening: movement felt by mother. Real
quickening happens at 5th month (20wks)
Maternal and Child Nursing (MCN) Chloasma/Melasma – mask of pregnancy,
– manages child & mother discoloration of face&neck
Probable signs
Pregnancy – objective signs, seen by nurse
– 3 periods: 1st-3rd trimester Hegar sign: thinning of uterus
Chadwicks: vaginal blue/purplish
First Trimester discoloration ← lack of circulation
– organogenesis: creation of organs Goodell’s: softening of cervix
– most critical part: teratogenic effects Non pregnant state: soft as nose
– OTC medications not allowed. Safest Pregnant state: soft as earlobe
drugs: category A: biogesic/paracetamol Labor state : soft as butter
– development of germ layers HCG– human chronic gonadotropin (+) PT
Ectoderm–brain Braxton Hicks contraction: painless, irregular
Mesoderm–heart Ballotement: bouncing of fetus
Endoderm–GI Ballotable–not engaged
– ambivalence: presence of opposing feelings Non ballotable–engaged

Second Trimester Positive signs


– easiest part (adapted to pregnancy) / – confirmatory
comfortable UTZ: ultrasound
– increased libido (sexual urge) Transvaginal: early pregnancy.
Bladder empty
Third Trimester Abdominal: late pregnancy. Drink
– feelings of unattractiveness, low water (2-3 glasses) to distend
self-esteem → sudden hormonal increase abdomen→push uterus up→easy
– talk to the husband to ↑ self-esteem of visualization
mother. If single mom, talk to the family or FHT: fetal heart tone
siblings or friends normal:120-160 bpm. If brady/tachy
→ fetal distress
Movement felt by examiner

SIGNS AND SYMPTOMS OF PREGNANCY

Presumptive sign
– subjective from the mother ADAPTATIONS IN PREGNANCY
– systemic changes in pregnancy
Breast changes
Amenorrhea Cardio/Circulatory changes
Urinary changes – ↑ total cardiac output-
Nausea & vomiting – palpitations (should be temporary)
– edema

1
TRICIA JESHIEL P. TORRES, RN OBSTETRICS

Lower extremities: normal → elevate


foot RESPIRATORY SYSTEM CHANGES
Upper extremities /anasarca – SOB→ gravid uterus → inadequate lung
(generalized edema)/periorbital: expansion
abnormal. Sign of PIH:
pregnancy-induced hypertension URINARY SYSTEM CHANGES
– varicosities of lower extremities/varicose → inc urinary freq (compression of bladder by
veins: tortuous veins/vein enlargement ← gravid uterus)
pooling of circulation in lower extremity → → dec urinary threshold
elevate
Homan’s sign: triggered by MUSCOSKELETAL SYSTEM CHANGES
dorsiflexion of foot→ pain in calf → lordosis (pride of pregnancy) → loss of
muscle → (+) → thrombus formation balance→safety
→ DONT massage, blood clot will → leg cramps/backpain→ rest/elevate
dislodge → elevate
– DVT: deep vein thrombosis
Virchow’s triad:
Venoustasis→ anti-embolic stockings
Venal wall damage→
Vlood coagulation

Temperature→ increased → ↑BMR (basal


metabolic rate), to suffice for the growing
fetus

Adrenal Glands: inc activity(inc glucose prod.


→ suffice fetus)

Skin changes:
GASTROINTESTINAL CHANGES – melasma/chloasma: mask pregnancy (face
– morning sickness: hypoglycemia in the and neck)
morning (continuous getting of glucose of – linea nigra: blackish ine in abdomen during
fetus at night) →simble carbohydrates: pregnancy→ linea alba: whitish line after
crackers/dry toast → before getting out of bed – striae gravidarum→ cocoa butter:minimize
– hyperemesis gravida → excessive vomitting scarring
→ SFF: small frequent feeding
– constipation & flatulence → high fiber diet + Uterus:
↑ OFI (oral fluid intake) → stimulate – hegar’s sign: thinning of uterus
peristalsis
– hemorrhoids→↑ intraanal pressure → Cervix:
bulging veins in anal area – goodell’s sign: softening of cervix
Internal: asymptomatic
External: painful Vagina:
***warm sitz bath/witch hazel – chadwick’s sign: purplish/bluish
(astringent/ vesicant) discoloration of vagina. No management.
– heartburn: chest pain→ HCL reflux→ (SFF) Only temporary
– vaginal pH: slightly acidic (normal: 5.5-6.5)
→ Protects vagina from certain infection
** UTI→ preterm labor

2
TRICIA JESHIEL P. TORRES, RN OBSTETRICS

FETAL DIAGNOSTIC PROCEDURES


Amniocentesis: amniotic fluid sample
AFP (Alpha FetoProtein)
Increased: neural tube defects:
hydrocephalus, menigocele,
anencephaly
Dec.: down’s syndrome/trisomy 21

PREGNANCY DIAGNOSTIC PROCEDURES


Ultrasound
STAGES OF FETAL DEVELOPMENT Purpose
– diagnose
Fertilization (ampulla) → implantation – confirms sex
(uterus/upper uterine segment) → – confirm growth & abnormalities
** implantation in lower uterine segment: – fetal maturity (AOG)
placenta previa Types:
– transvaginal: early pregnancy, empty
bladder
Conceptus Period
– abdominal: late pregnancy, drink water →
Ovum Ovulation→ easier visualization
fertilization

Zygote Fertilization→
imPLANTation

Embryo Implantation→ eight


weeks

Fetus 8 weeks → Full term

Amniotic Fluid LEOPOLD’S MANEUVER


– protection – palpation → empty bladder
– temperature regulation
– supports growth 1st
– fetal movement – fundal grip→ presentation (cephalic,
breech, transverse, footling)
Umbilical cord
– AVA: normal 2nd
– VAV: abnormal→ congenital heart defects – umbilical grip → fetal back (PMI: point of
– wharton’s jelly: protective covering of maximum impulse), FHT is most audible
umbilical cord
3rd
– Pawlick’s grip → engagement→ stations
-3 floating (above ischial spine)
-2 floating (above ischial spine)
-1
0 engaged → at the level
+1
+2 crowning (below ischial spine)
+3 crowning (below ischial spine)

4th
– pelvic grip→attitude/fetal lie(degree of
flexion & extension of fetus)

3
TRICIA JESHIEL P. TORRES, RN OBSTETRICS

Psyche: overall status of mother (physical,


emotional, psychological)

Clean catch urine


FALSE LABOR
– method of collecting urine sample
– remain irregular
First stream: discard (contaminated)
– confined in abdomen
Second stream/middle: use
– no increase in frequency, duration, interval
– purpose: detects UTI
– relieved by ambulation
– absent cervical changes / no cervical
Discomforts of pregnancy
dilatation and effacement
– nausea & vomiting → SFF/dry
crackers/toast
TRUE LABOR
– breast tenderness → supportive bra (firm
– regular and predictable
fitting)
– pain in lower back to abdomen
– ptyalism (excessive salivation) →
– increased frequency, duration, shortened
mouthwash, gum, hard candy
interval
– ankle edema → elevate/no prolonged
– not relieved by ambulation
standing
– with cervical dilatation and effacement/with
– backache: low heeled shoes with grip
cervical changes
– constipation→ high fiber diet, OFI
– bloody show/mucus plug erxpulsion
– heartburn → SFF
– rupture of BOW (bag of water)
– hemorrhoids: warm sitz bath + witch hazel
– SOB→pillows, left lateral position (prevents
orthostatic hypotension/venacava syndrome)
– travelling: every 2-3 hrs, rest for 15-20 mins

1. DILATATION STAGE
3 phases:
Latent: 0-3cm, mother able to communicate
(health teachings)
Components of Labor Active: 4-7cm, inc frequency, duration,
shortened interval, mother loses self-control,
Passenger: fetus FOCUS: Pain
Passageway: pelvis(main), cervix, vagina Transition: 8-10cm, fully dilated and fully
**Cephalopelvic disproportion (CPD)→ CS effaced, transfer mother froim labor room to
Power delivery room
Primary: uterine contraction, stronger
Secondary: bearing down

4
TRICIA JESHIEL P. TORRES, RN OBSTETRICS

2. EXPULSION STAGE/FETAL EXPULSION


Types of placenta
Mechanisms of labor: Schultze : S-hiny
Descent: feal head goes down to maternal Duncan: D-irty/meaty (risky: higher chance of
pelvis(engagement) placental fragment retention
Flexion: preparation of fetal head to pass the
cervix ** count cotyledons: 20-30

Internal Rotation: actual passage of fetal 4. RECOVERY AND BONDING/POST


head to cervix PARTUM
– first 1-4hrs after delivery
Extension: expulsion of fetal head – most critical stage, rapid deterioration of
mother, continuous monitoring
External Rotation: expulsion of fetal
shoulders (lack of external rotation: shoulder Considerations:
dystocia) – assess fundus (firm and globular)
– lochia
Expulsion: expulsion of the whole fetus Rubra: 0-3 days, reddish
Serosa: 4-9 day, pinkish to brownish
Alba : 10th day to 6 wks, whitish →
normal involution: return of the organs
to the non-pregnant state
– bladder: empty
– perineum: s/sx of infection
– lactation
Oxytocin: milk ejection/milk letdown →
uterine contraction
Prolactin: milk production
3. PLACENTAL STAGE – blood pressure: s/sx of shock →
3 signs of placental separation hypotension, tachycardia, tachypnea
– lengthening of the cord – blood loss
– sudden gush of blood NSD: 500ml
– Culkin’s sign: uterus becomes firm and CS: 800-1000ml
globular (well contracted)

** uterus not well contracted (soft and boggy)


→ bleeding → massage uterus, give oxytocin
** uterus dislodge to the right (full bladder) →
encourage to urinate/catheter
**if given oxytocin, uterus doesn't contract →
re-clean uterus, placental fragments
** uterus well contracted, but there’s bleeding
→ perineal laceration
EPISIOTOMY/EPISIORRAPHY AND
PERINEAL CARE

– ice pack: stop bleeding (first 24 hrs)


5
TRICIA JESHIEL P. TORRES, RN OBSTETRICS

– warm sitz bath: healing (after 24 hrs) MANAGEMENT:


– witch hazel: hemorrhoids 1. Cervical Cerclage
– use cotton underwear (absorbent) - prevent 2. Vaginal rest
infection 3. Prepare for child birth if with rupture of
membranes
** cleaning of perineum: single stroke then
discard. Front to back.
ABRUPTIO PLACENTA
– increase fiber and OFI: prevent - Premature separation of the placenta after 20th
constipation/ encouraging early ambulation week of AOG
– sexual activity
Vaginal bleeding stopped MANIFESTATIONS:
episiotomy/episiorraphy is healed (6 1. painful, dark red vaginal bleeding
weeks) TOXEMIA/PIH
• GESTATIONAL HYPERTENSION
HANDOUT BY SIR ARCHIE ALVIZ • CHRONIC HYPERTENSION • PIH
MATERNAL COMPLICATIONS - triad: hypertension, edema, albuminuria
ANTEPARTUM COMPLICATIONS A. Mild Pre-Eclampsia
ABORTION B. Severe Pre-Eclampsia
▪ Threatened ▪ Imminent ▪ Therapeutic C. Eclampsia
▪ Missed
HEMOLYTIC DISEASE OF THE NEWBORN
MANAGEMENT: - Rh incompalibility
• CBR 12-24 hrs DIAGNOSTICS:
• D and C 1. Amniocentesis – measures bilirubin in amniotic
fluid
HYDATIDIFORM MOLE 2. Direct Coomb’s test – measures Rh antigen in
• Aka Gestational Trophoblastic Disease the baby
• No fetus, no amniotic sac, no blood vessels 3. Indirect Coomb’s test – measures Rh antibody
in the maternal circulation
Manifestations:
• High levels of HCG MANAGEMENT:
• rapid increase in fundic height Rhogam administration:
1. At 28 weeks AOG
Interventions: 2. Within 72 hours after birth
• D & C • Use of methotrexate INTRAPARTUM COMPLICATIONS
PREMATURE RUPTURE OF MEMBRANES
ECTOPIC PREGNANCY
Common sites: fallopian tube, ovary, cervix, MANIFESTATIONS:
peritoneal cavity - Amniotic fluid gushing into the vagina
- Fetal tachycardia
Manifestations:
▪ severe sharp stabbing knifelike abdominal pain MANAGEMENT;
▪ spotting 1. minimize IE
▪ (+) Cullen’s Sign – means RUPTURE 2. Assess for signs of infection
▪ Cushing’s Triad
CAESARIAN BIRTH TYPES:
Interventions: 1. Classic or vertical
▪ Combat SHOCK 2. Transverse or Pfannenstial
▪ Elevate foot of bed
Factors leading to CS
INCOMPETENT CERVICAL OS 1. CPD
• painless cervical dilatation os without 2. Herpes
contractions 3. Previous cs
• 20th week of AOG 4. Hypertension or heart disease
5. Placenta previa
MANIFESTATIONS: 6. Abruption placenta
1. Cervical dilation 7. Transverse lie
2. Prolapse of the membranes 8. Fetal distress
9. macrosomia

6
TRICIA JESHIEL P. TORRES, RN OBSTETRICS

OB PRE TEST A. Ask the child if he/she would like to take


the medication from a cup or spoon.
PRETEST B. Give the child a drop or two of liquid over
an hour top help prevent spitting it out.
C. Tell the child the medicine is sweet candy
1. Juday has been experiencing regular, syrup that he/she will like.
coordinated contractions with cervical dilation D. Tell the child firmly to take the medicine.
moving from 4 cm to 6 cm in the last half, and
her membranes are still intact. Juday is in 6. After a tubal ligation, it is not uncommon
which of the following stages of labor? for the woman to complain of:
A. Latent phase of the second stage of labor A. shoulder pain.
B. Active phase of the first stage of labor B. hemorrhoids.
C. Placental stage or the third stage of labor
D. Predelivery stage or the prelabor stage of C. leg pain.
labor D. breast tenderness.

2. Lhia is admitted in active labor. The 7. When examining a postpartal woman,


nurse locates fetal heart sounds in the upper the nurse should immediately report:
left quadrant of her abdomen. The nurse A. a fundus that is palpated 2 cm below the
recognizes which of the following? umbilicus on the second postpartal day.
A. Lhia will probably deliver very quickly and B. a fundus that cannot be located by
without problems. palpation on the ninth postpartal day.
B. This indicates Lhia will probably have a C. a soft, spongy uterine fundus noted during
breech delivery. the first four hour postpartum.
C. The fetus is in the most common anterior D. red, bloody vaginal discharge on the
fetal position. perineal pad on the first day postpartum..
D. This position is referred to as being left
anteropelvic. 8. Which of the following clinical practices
is appropriate to solicit initial respiration of the
3. After the third stage of labor, the nurse high-risk newborn?
may have which of the following A. Rubbing the back
responsibilities? B. Spanking the buttocks
A. Administration of intramuscular oxytocin to C. Slapping the face
facilitate uterine contractility. D. Squeezing the thorax
B. Monitoring for blood loss greater than 60
cc, which would indicate gross hemorrhage. 9. On the third hospital day in the nursery,
C. Noting if the placenta makes a Schultze Mr. and Mrs. Smith’s newborn baby girl Rhian
presentation, which is a sign of gross is diagnosed with Rh incompatibility. Mrs.
complication. Smith is a gravida 2, para 1, abortion 1. This
D. Pushing down on the relaxed uterus to aid disease causes blood cell hemolysis that is
in the removal of the placenta probably directly due to
A. the exchange of fetal and maternal blood
4. Which action of the following instructions in the utero
to a child would be most appropriate before a B. Rh positive fetus and Rh positive father
painful procedure? C. Rh negative mother, Rh positive father
A. “Do not tell the child anything, just proceed D. the sin of the abortion of the first child
with the procedure.
B. “This won’t hurt at all.” 10. Baby girl Luisa was born large for
C. “This is going to hurt.” gestational age. After being delivered
D. “Would you like t help me?” vaginally, this infant should be carefully
assessed for:
5. Which method would be helpful when A. increased intracranial pressure
administering liquid medication to a B. hypothermia
preschool-aged child? C. decreased red blood levels (anemia)

7
TRICIA JESHIEL P. TORRES, RN OBSTETRICS

D. hyperglycemia

11. Which of the following should the


examiner do to assess for scoliosis?
A. Assist the child to look in a mirror for
characteristic curing.
B. Have the child place the chin on the chest,
place hands together, bend over, and let the
hands hang freely.
C. Have the child turn sideways, and look for
bulging in the thoracic region.
D. Palpate the spine for indentation in the
lower back.

12. During an episode of somnambulism, it


would be appropriate for the nurse to:
A. ask the child what he/she is doing
B. immediately wake the child.
C. pull the child back to bed.
D. observe for safety measures to avoid
injury.

13. Which instruction should the nurse


provide the adolescent who is experiencing
acne?
A. Avoid chocolate in the diet.
B. Inspect skin for adverse reactions to
treatment.
C. Pop pimples only when they are white with
pus.
D. Scrub skin twice a day with prescribed
medication.

14. Which of the following is an inappropriate


technique to use when giving a back rub?
A. Long, even upward strokes
B. Tapping lightly with the edge of the hand
C. Firm scratching motion
D. Application of lotion

15. You are giving a parenting class. You will


discuss which of the following with the
parents of Dora, a 2-year old child?
A. Dora can be expected to be cooperative
and easily cooperative and easily controlled
at this stage of development.
B. Dora may prefer finger foods and clothing
she can put on without assistance at this age.
C. Expect Dora to want to be held and
cuddled a great deal at this stage of her
development.
D. At this age, Dora will take the initiative in
activities and will question everything.

8
TRICIA JESHIEL P. TORRES, RN OBSTETRICS

OB POST-TEST
7. The client begins to question the nurse
1. Which drug category may be safely about sexuality and becoming sexually active.
administered during pregnancy? During sexual counseling, the nurse should
A. Category A place a major point of emphasis on:
B. Category B A. douching after sexual intercourse.
C. Category C B. sex during menstruation.
D. Category D C. performing Kegsl exercises.
D. safe and responsible sex.
2. Which of the following is a positive
indication of pregnancy? 8. Using Naegle’s Rule, calculate the
A. Quickening estimate date of delivery if the woman’s LMP
B. Chadwick’s sign was October 1.
C. Auscultation of fetal heart sounds A. June 23
D. Ballottement B. July 8
C. December 23
3. During the last months of pregnancy, the D. January 8
nurse should instruct the client to:
A. rest on her left side for at least 1 hour in
the morning and afternoon
B. sleep on her back during the night and
during naps
C. start nipple exercises and stimulation twice
9. Which of the following would be
a day
appropriate to teach the pregnant woman
D. start to cut back on water intake,
regarding exercise?
especially at night
A. Avoid any physical activity during the
second trimester
4. When teaching a pregnant woman about
B. Contact sports are allowed during the first
traveling during the pregnancy, it is important
trimester
to focus on which of the following?
C. Extremely active women should reduce
A. If traveling by car, stop every 2 hours for
the level of exertion
10 minutes
D. Relaxation and stretching exercises should
B. Get plenty of rest before long trips made in
be discontinued.
automobiles
C. Travel in any type of aircraft is acceptable
10. One of the functions of the amniotic fluid
D. Travel can be completed anytime
is to:
throughout the pregnancy
A. exchange nutrients from mother to fetus
B. immobilize the fetus
5. Which of the following is recommended
C. protect the woman’s uterus
for all women during the childbearing age?
D. regulate temperature
A. Additional B vitamins
B. Additional vitamin A
11. A feeling of ambivalence about the
C. Folic acid supplement
pregnancy is:
D. Vitamin C supplement
A. a sign of unwanted pregnancy
B. normal in early pregnancy
6. An 18-year old girl visits to obtain a
C. rare at any stage of pregnancy
contraceptive. She elected to use the
D. typical in late pregnancy
intrauterine device (IUD) as a method of birth
control. The major concern with the use of the
12. Which assessment relates most directly
IUD is:
to rupture membranes and release of
A. thrombophlebitis.
amniotic fluid?
B. pain on intercourse.
A. Bloody show
C. infection.
B. Fluid with a pH of 7.0 to 7.5 with nitrazine
D. abnormal bleeding and hemorrhage.
test
9
TRICIA JESHIEL P. TORRES, RN OBSTETRICS

C. Fluid with a pH of 5.0 with nitrazine test D. The major structures of the embryo are
D. Woman complains of urge to push forming, and damage can result in major birth
defects
13. When the placenta is delivered with
the dull side out (Duncan presentation), the 19. Which of these statements is most
woman is at risk for: accurate about the placenta?
A. excessive bleeding A. The blood of the baby mixes with the
B. hemorrhoids mother’s blood to permit exchange of
C. increased lacerations of the perineum nutrients and oxygen.
D. sterility B. The blood of the baby and the mother do
not mix; exchange occurs across blood
14. To assess the uterine contraction vessels and the walls of the villi.
during labor, the nurse: C. The placenta lets the blood from the fetus
A. asks the woman if she is having a cross to the mother, but the mother’s blood
contraction. does not cross to the fetus.
B. palpates above the symphysis pubis. D. The placenta serves as a complete barrier
C. palpates just below the xyphoid process of between the baby and mother so that any
the sternum. drugs the mother takes do not cross the baby.
D. performs a sterile vaginal examination.
20. Which of the following is the best
15. The nurse knows that a postpartum recommendation about taking medicines
client’s susceptibility to hemorrhage is most during pregnancy?
likely related to a: A. All over-the-counter (OTC) drugs are safe
A. boggy uterus during pregnancy
B. firm fundus B. All herbal preparations are safe during
C. long labor pregnancy
D. negative Homan’s sign C. Don’t take anything during pregnancy
without asking your health care provider
16. Between 24 and 28 weeks, all pregnant D. Take an OTC diuretic if you have swelling
women should be screened for: during the pregnancy
A. Anemia
B. Bladder infections 21. When providing postpartum teaching
C. Diabetes about self-care, one of the danger signs that
D. Neural tube defects a lactating woman should know to report to
the birth attendant is:
17. Which of these measures would be A. breast engorgement to a degree that the
helpful for the pregnant client complaining of baby can’t latch on.
sleeplessness? B. breast fullness just before feeding .
A. Eat evening meal close to bedtime C. nipple soreness after feedings.
B. Sit in a sitz bath before bedtime D. nipple dryness before feedings.
C. Try to remain in one position when
sleeping 22. “Show” is usually present in:
D. Use pillows to help find a comfortable A. Braxton Hicks contraction.
position B. false labor.
C. true labor.
18. Why is the first 8 weeks of pregnancy D. Second stage of labor only.
known as the critical period of human
development? 23. APGAR score assessments are
A. By the time this period ends, the embryo is completed at:
completely safe from any damage A. birth and 10 minutes.
B. Many embryos die during this period B. 1 and 5 minutes.
C. The infant’s sex is determined at the end C. 5 minutes and upon arrival to the nursery.
of the eight week D. the time of birth.

10
TRICIA JESHIEL P. TORRES, RN OBSTETRICS

24. A newborn of 4 hours displays grunting C. relief of pain.


respirations and a respiratory rate of 70 D. the uterus remaining firm.
breaths/minute. The priority nursing
intervention would be to: 30. During active labor, the mother usually
A. begin resuscitative measures and call for exhibits which of the following behaviors?
help. A. Difficulty following directions
B. continue to monitor respiratory status B. Excitedness and talkativeness
variations are normal. C. Frustration and irritability
C. obtain vital signs every 15 minutes. D. Serious expression and apprehension
D. transfer the newborn to the mother’s room
for feeding. 31. When evaluating the effectiveness of
instruction regarding breastfeeding, which of
25. Which measure would be used to the following responses by the mother
prevent loss of heat in the newborn? indicate that she understands the teaching?
A. Immediately give the baby a bath A. “I need to rub my nipples to toughen them
B. Place the baby on the mother’s bare up.”
stomach B. “I should apply lotion to my nipples to
C. Offer the baby warm glucose water prevent cracking.”
D. Wrap the baby in room-temperature C. “I should nurse at least 10 minutes on one
blankets breast before offering the next breast.”
D. “I should use soap and water to gently
26. Which of the following indicates that the cleanse my breast gently.”
new mother understands how to handle
breast milk safely? 32. Which of the following should the nurse
A. “I can store fresh milk in the refrigerator for recommend to the breastfeeding mother to
only 24 hours.” limit in her diet?
B. “I can store frozen breast milk for up to 1 A. Cheese
month.” B. Fruit
C. “I need to express my breast milk into a C. Strongly flavored foods
clear glass.” D. Vegetables
D. “I should never store my breast milk in a
frozen-food locker.” 33. The fontanels are soft spots formed by
the:
27. To prevent infection of the perineal area A. blood accumulated between the bone and
after delivery, the nurse should instruct the periosteum.
client to: B. edema of the scalp from birth pressure.
A. Begin sitz bath at the first sign of infection C. junction of individual skull bones.
B. Pull panties straight down D. pressure of a vacuum extractor.
C. Use hot water to cleanse the area after
bowel movement 34. Signs of respiratory distress in a neonate
D. Wipe with sweeping motion, from front to include:
back A. grunting with expiration.
B. respiratory rate of 50 breaths per minute.
28. Analgesics given too late in labor can C. synchronized movement of the baby’s
result in which of the following? chest and abdomen.
A. Contractions that increase in intensity D. the baby’s chest expanding as a whole.
B. Early deceleration
C. FHR dropping to 100 beat per minute 35. The most efficient way for a baby to
D. Pain during contractions regulate temperature is to:
A. burn body fat.
29. In evaluating the effects if oxytocin after B. move arms and legs.
delivery, the nurse should monitor for: C. shiver.
A. effective breastfeeding. D. use brown fat.
B. engorged breasts.

11
TRICIA JESHIEL P. TORRES, RN OBSTETRICS

36. Which measure would be most effective B. Keep the mother calm
in preventing the transfer of gonorrhea or C. Place bulb suction at head of the bed
Chlamydia to the infant’s eyes from the D. Rub the baby’s back
mother?
A. Administering Vitamin K 43. In which position should the newborn
B. Bathing the newborn with intracranial hemorrhage be placed?
C. Cleaning the infant’s eyes with warm A. Prone
saline B. Side-lying
D. Applying erythromycin ointment C. Slightly elevated head of bed
D. Supine
37. A newborn will respond to sudden noises
or jarring movement by throwing out the arms 44. Which of the following contraceptive
and drawing up the legs. This is called a:t methods also offers protection against
A. Moro reflex sexually transmitted infections?
B. Babinski reflex A. Abstinence
C. Rooting reflex B. Coitus interruptus
D. Tonic neck reflex C. Fertility awareness methods
D. Oral contraceptives
38. The nurse is caring for a newborn at 12
hours of life. The newborn has just voided. 45. In teaching the postoperative client who
The most appropriate response by the nurse has just had a vasectomy, it is important to
would be: focus on:
A. immediately check vital signs. A. asking how his partner feels about the
B. notify the physician. surgery.
C. continue to monitor voiding patterns. B. alternate methods of expressing sexual
D. obtain an order for a straight catheter. needs and desires
C. discussing that a vasectomy does not
39. Which position should newborns be make client less of a man
placed when sleeping? D. using birth control until sperm counts are
A. Back zero for 6 weeks.
B. Head of bed elevated
C. Prone 46. Which assessment most closely relates
D. Side lying with pillow to a diagnosis of ectopic pregnancy?
A. Brownish red, tapioca-like vesicles
40. Most babies should be fed: B. Elevated temperature
A. Every 1 to 2 hours C. Spotting or bleeding 2 to 3 weeks after a
B. Every 2 to 4 hours missed menstrual period
C. Every 4 to 6 hours D. Sudden absence of fetal movement
D. On demand
47. The drug of choice to treat
41. Which of the following drugs can be pregnancy-induced hypertension is:
given to the mother before a preterm birth to A. iron and vitamins
help reduce the severity of respiratory B. diazepam (valium)
distress syndrome? C. furosemide (Lasix)
A. Betamethasone D. magnesium sulfate
B. Diazepam
C. Phenobarbital 48. Which nursing intervention would be
D. RhoGAM appropriate for a client who has a diastolic
blood pressure of more than 20 mmHg on
42. Which intervention is the priority the “roll-over” test?
immediately after the delivery of a newborn A. Increase intake of oral fluids
who does not breathe? B. Rest on left side as much as possible
A. Clear air passages of obstructive C. Schedule follow-up care every 2 weeks
substances D. Use the stairs to increase activity level

12
TRICIA JESHIEL P. TORRES, RN OBSTETRICS

49. A mother receiving medications for


pregnancy-induced hypertension should have
her diastolic blood pressure maintained in the
range of 90 to 100 mmHg to:
A. avoid causing fetal anoxia.
B. ensure progression of labor.
C. prevent premature contractions.
D. present sudden elevations in pulse.

50. Which of the following would be a priority


intervention for a client with the prolapsed
cord?
A. Cover the cord with a dry sterile tower
B. Monitor the mother’s vital signs
C. Place the woman in the Trendelenburg
position
D. Start medication as ordered

13
TRICIA JESHIEL P. TORRES, RN ENDOCRINE & RESPIRATORY DISORDERS

Date: December 12, 2022


Time: 9AM - 4PM
Concept: RESPI ENDO
Lecturer: PROF. KEITH GARINO

Sir Keith’s Reminders:


Black: normal process
Red:
Blue pen: intervention

Pituitary Gland - anterior & posterior


Anterior Pituitary Gland - TSH, LH, FSH
Anterior: ↓TSH
Growth hormone ↓LH
Thyroid stimulating hormone ↓FSH → amenorrhea
Prolactin
ACTH
LH TSH (Thyroid stimulating hormone) → ↑TG
FSH (thyroid gland)
MSH - Melanocytes stimulating hormone 1. ↑T3 (Triiodothyronine) - ↑metabolism → ↓
weight ↑ appetite
Posterior: - ↑ GIT → diarrhea
Oxytocin - uterine contraction 2. ↑T4 (thyroxine)- ↑heat → heat intolerance
ADH/Vasopressin - for fluid retention & absorption → diaphoresis, sweating
in renal tubules 3. ↑Thyroid hormones → ↑SNS →
hyperactive → restless & irritable
→ ↑ V/S → thyroid storm (usually
affected HR)

Drugs - thioamides * GI irritant, taken with meal


→ methimazole
→ propylthiouracil (PTU)

* GI irritant, taken with meal


* A/E → agranulocytosis (↓ WBC, infection)
→ bleeding

↑ calorie & monitor daily wt


↑ fluid & ↓ fiber
HYPERTHYROIDISM Provide cool environment
↑ thyroid hormone Non-stimulating environment
Common cause - grave’s disease → autoimmune Thyroid storm → report
→ hyperactive thyroid gland Surgery: thyroidectomy
Appearance SUMMARY:
Inflammation → fat pads behind eyes → grave’s All increased except wt, menstruation, & FSH
ophthalmopathy: exophthalmos (bulging of the
eyeball) → can't blink eyes
Dryness → eye drops
Sun → sunglasses
Sleep → tape/eye patch

1
TRICIA JESHIEL P. TORRES, RN ENDOCRINE & RESPIRATORY DISORDERS

→ ↑absorption → empty stomach


→ taken in the morning
→ A/E or S/E→ “hyperthyroidism”

↓ calorie
↑ fluids↑ fiber
Warm environment
Assess LOC & avoid sedatives

HYPOTHYROIDISM
↓thyroid hormones

Adults - myxedema → puffiness of face


→ assess facial features
→ “ABC”

Children - cretinism (mental reatrdation)

Cause:
Hashimoto’s disease
→ autoimmune → destroy thyroid gland

ADRENAL GLANDS - APG


→ACTH - adrenal cortex
All decreased except wt & TSH →MSH - skin → melanin
SSS - sugar, salt, sex
APG
Adrenal Cortex - “steroids”
↑TSH
Glucocorticoids
TG → cortisol → natural release: morning
→ anti-inflammatory → ↓ immunity
1. ↓T3 ↓metabolism ↑wt ↓appetite → “stress”
(anorexia) → ↑ glucose
→ ↓ GIT → constioation → breaks down protein & bones
2. ↓T4 → heat → cold intolerance
3. ↓ Thyroid hormones → SNS → Mineralocorticoids
hypoactive →aldosterone → Na & H2O (reabsorption)
↓ V/S →↓K

Androgen (sex characteristics)


Drug: levothyroxine (thyroid hormone) → “for life” → testosterone
→ estrogen
2
TRICIA JESHIEL P. TORRES, RN ENDOCRINE & RESPIRATORY DISORDERS

→ cardiac dysrhythmia - check V/S (HR)

↑ Androgen
↑ Testosterone - female → virilization → hirsutism
& amenorrhea
↑ Estrogen - male → feminization →
gynecomastia
*altered body image → allow px to verbalize
feelings

DRUGS:
MMK ↓ steroid prod.
Mitotane
Metyrapone
CUSHING’S SYNDROME Ketoconazole
↑ GMA

Cause:
Adrenal adenoma → adrenalectomy
Prolonged steroid therapy

↑Cortisol
Increases glucose - hyperglycemia
↑ insulin → ↑ fats → moon face
→ dorsocervical - buffalo hump
→ truncal/central obesity

↑Breakdown of protein
→ skin → thin w/ easy bruising ADDISON’S DISEASE
→ stretchmarks or strea Adrenal insufficiency
→ ext. → thin peripheral → auto immune → destroy → adrenal cortex →
→ bones are weak ↓ cortisol & ↑ aldosterone

Imunosuppressive action - ↑infection APG

INTERVENTIONS: ↑ACTH & ↑MSH


Avoid stress
Monitor blood glucose & avoid sweets A. Cortex skin
Avoid crowded places & ill person
↓Steroids ↑ melanin (Bronze skin)

↑ Aldosterone - hyperaldosteronism
* all decreased except bronze skin, ACTH & K
→ ↑Na - hypernatremia - ↓ Na diet (avoid salty
foods)
↓ Cortisol → ↓ glucose → monitor
→ ↑H2O - fluid volume excess - fluid restriction
→ ↓ coping mech. - stress, avoid
→ ↓K - hypokalemia - ↑ K diet
3
TRICIA JESHIEL P. TORRES, RN ENDOCRINE & RESPIRATORY DISORDERS

Urine specific gravity


Aldosterone → ↓ Na → ↑ Na diet →increase & concentrated
→ ↓ H2O - FUD → fluid replace
→ ↑ K → ↓K diet Drugs: Declomycin → blocks ADH
Removal of tumor
Drugs: Fluid restriction & check daily weight
Check I&O
Corticosteroids - “for life” Diuretics except Thiazide. causes opposite effects
→ GI irritants - w/ meal
→ immunosuppresant ↑ infection

DIABETES INSIPIDUS

Trauma/ Surgery (head)


↓ ADH → neurogenic
Damaged kidney → nephrogenic
↓ fluid retention → FUD → polydipsia
↑ urine (polyuria) ↓ USG, diluted

For Neurogenic: Drug: vasopressin (desmopressin)


→ A/E water intoxication
→ fluid intake

For nephrogenic: Drug: thiazide, clofibrate &


Anti Diuretic Hormone normal function chlorpropamide

↓fluids → posterior pituitary gland release Monitor daily weight & I&O

ADH

Kidneys → fluid retention

↓ urine output

Syndrome of inappropriate antidiuretic hormone


secretion (SIADH)

Tumors → ↑ ADH

Kidneys → ↑ fluid retention → FVE →


delusional ↓Na → cerebral edema DIABETES MELLITUS

↓ urine Chronic disorder of carbohydrate, fat and protein


Oliguria Cause is unknown
4
TRICIA JESHIEL P. TORRES, RN ENDOCRINE & RESPIRATORY DISORDERS

1. Fasting blood sugar


Predisposing factors: → NPO 8-10hrs
Stress → ↑ cortisol ↑ glucose → normal: 70-110 mg/dL
Heredity
Obesity ↑ fats → insulin resistance 2. Capillary blood glucose
Autoimmune - destroy pancreas → B cell ↓ insulin → normal: 80-120 mg/dL
→ self-monitoring → report if 250mg/dL,
risk for DKA

3. Glycosylated hemoglobin (HGBA1C)


→ blood sugar for past 3 months
→ normal: 4-6%. DM goal: <7%

Types:

DM1 DM2

Juvenile onsite Adult onset


Thin Obese
↓ insulin Insulin resistance
Diabetic keto acidosis hyperosmolar
hyperglycemic
Mgt. DIE non-ketotic syndrome DM1
Diet → complex carbs
Insulin → primary Mgt. DIE OHA Glucose → insulin → cell
meds. Diet → hyperosmolarity
Exercise → moderate Insulin → ↑ viscosity → poor circulation
3-5x/wk Exercise ↓
S/E → hypogly Complication
→ eat prior exer. Oral hypoglycemic → hyperglycemia
agents → primary ↓
meds. Kidney
→SGLT2
→ glucose reabsorption (200mg/dL)
→ exceed → glycosuria →
Strenuous exercise → ↑ cortisol ↑ glucose
attracts h2o →polyuria → FVD →
polydipsia

→→→ cell → starvation → polyphagia


No insulin → breakdown of protein → thin
→ fats → ↑ ketones → DKA
→ lungs - fruity odor
→ kidney - ketonuria
→ brain - ↓ LOC

→→→ DKA → Metabolic Acidosis → ↓PH &


↓HCO3 ↓
Compensatory: lungs
→hyperventilation
DIAGNOSTIC TESTS →deep labored breathing

5
TRICIA JESHIEL P. TORRES, RN ENDOCRINE & RESPIRATORY DISORDERS

→kussmaul’s respiration → avoid exercise, massage,


warm application

* lipodystrophy - hardening of subq


→ prevention - reuse of site after 2-3wks
→ distance: 1-1 ½ inch
→ room temperature
→ storage of unopened: ref
→ rotation of site

DM2

HYPOGLYCEMIA
Blood glucose below 60mg/dL

DKA & HHNS “Hunger” → neuro & SNS

Cause: stress ↑ cortisol ↑ glucose S/S


→ infection Sweating → SNS
→ surgery Tremors → Neuro
→ trauma Light headedness → neuro
Tachycardia → SNS
Mgt.
1. IV NSS → improve circulation Mgt
2. IV regular insulin Give 15-30g of fast acting carbohydrates
→“15-15 rule” 3x
Hard candy, 4tsp of sugar, ½ fruit juice or soda,
low-fat milk
In unconscious - give glucagon

INSULIN
Route: Subq → 45-90 degrees angle
→ thin: 45-60 degrees
→ insulin syringe: gauge 27-29 ½ inch
→ units
→ area: abdomen ↑ absorption
→ thigh, buttocks. Upper arm

6
TRICIA JESHIEL P. TORRES, RN ENDOCRINE & RESPIRATORY DISORDERS

Date: December 12, 2022


Time: 9AM - 4PM
Concept: RESPI ENDO CHRONIC OBSTRUCTIVE PULMONARY
Lecturer: PROF. KEITH GARINO DISEASE

Types
1. Emphysema - the presence of
ASTHMA over-distended and non-functional
Problem with the airway, not breathing alveoli
Cause:
1. Extrinsic - allergens ↑ air trapped in alveoli → trapping → barrel
2. Intrinsic - temperature, stress, chest → ↑anterior-posterior diameter
exercise 2. Chronic bronchitis - productive cough
that lasts 3 months in each of 2
consecutive years, in a patient in
whom other causes of cough are
excluded

Cause:
1. Smoking
2. Environmental
3. Genetics - Alpha-1 Antitrypsin
Deficiency (AATD)
→ macrophage destroys elastin &
collagen → ↓ elasticity of lungs

Metered dose inhaler


Doctor’s order will be by puff
1. Shake
2. Press & inhale at the same time
→ use a spacer to ↑ absorption
3. Hold breath to ↑ absorption 10 secs.

7
TRICIA JESHIEL P. TORRES, RN ENDOCRINE & RESPIRATORY DISORDERS

ALVEOLI FUNCTION Injury →↑permeability→fluid shifting in alveoli


→↑fluids ↑ hypoxemia
Normal: + gas exchange
COPD: - gas exchange → vasoconstriction Injury → type 2 cells →↓surfactant→ lung
→ ↓ blood flow collapse → atelectasis (lung collapse)
→ ↑ pressure (pulmonary
hypertension) Injury → healing → fibrotic → ↓ elasticity →
↓ lung compliance

Acute Respiratory Distress Syndrome


(ARDS)
- a form of acute respiratory failure that
occurs as a compilation of some other
condition; caused by a diffuse lung injury and
leads to extravascular lung fluid

8
TRICIA JESHIEL P. TORRES, RN ENDOCRINE & RESPIRATORY DISORDERS

RESPI ENDO PRE TEST 7. Which of the following disorders which is


1. Which of the following statements best describe characterized by hyperglycemia due to a decreased in
Cushing Syndrome? secretion of insulin by the pancreas?
A. A group of sign and symptoms which is caused by an A. Diabetes type 1
increase in cortisol in the body. B. Diabetes type 2
B. A group of sign and symptoms which is caused by a C. Diabetes Insipidus
decrease in cortisol in the body. D. Gestational Diabetes
C. A group of sign and symptoms which is caused by a
decrease in melanin in the body. 8. Which of the following would a client experience
D. A group of sign and symptoms which is caused by when there is an excessive glucose in the blood?
an increase in melanin in the body. A. Anuria
B. Oliguria
2. Which of the following statements is true about C. Polydipsia
syndrome of inappropriate anti-diuretic hormone D. Diaphoresis
secretion (SIADH)?
A. It is characterized by an excessive water retention in 9. A client with asthma has pronounced wheezing upon
the renal tubules. auscultation. Suspecting an impending asthma attack, a
B. It is characterized by a decrease in water retention in nurse should:
the renal tubules. A. have the client cough and deep breathe.
C. It is characterized by a decrease in blood calcium B. prepare to intubate the client.
levels. C. prepare to administer a nebulized beta-2 adrenergic
D. It is characterized by an increase in blood calcium agonist.
levels. D. have the client lay on his or her right side

3. Which of the following signs and symptoms would 10. A client, newly diagnosed with asthma is preparing
you expect in a client with diabetes insipidus? for discharge. Which point should a nurse emphasize
A. Polyuria during the client’s teaching?
B. Concentrated urine A. Contact care provider only if nighttime wheezing
C. Fluid volume excess becomes a concern
D. Increase in body weight B. Limit exposure to sources that trigger an attack
C. Use peak flow meter only if symptoms are worsening
4. Which of the following disorders where a client may D. Use inhaled steroid medication as a rescue inhaler
experience exophthalmos?
A. Hypothyroidism 11. Which finding should a nurse expect when
B. Hyperthyroidism completing an assessment on a client with chronic
C. Hyperparathyroidism bronchitis?
D. Hypoparathyroidism A. Minimal sputum with cough
B. Pink, frothy sputum
5. Which of the following medications is given to client C. Barrel chest
with hypothyroidism? D. Stridor on expiration
A. Levothyroxine
B. Propylthiouracil 12. A home health nurse is visiting a client whose
C. Lugol’s solution chronic bronchitis has recently worsened. Which
D. Saturated Solution of Potassium Iodide instruction should the nurse reinforce with this client?
A. Increase amount of bedrest
6. All the following symptoms may be experienced by a B. Increase fluid intake
client who has Addison’s disease except: C. Decrease caloric intake
A. Hyperglycemia D. Reduce home oxygen use
B. Hyponatremia
C. Hyperkalemia 13. Following an unrestrained motor vehicle crash, a
D. Weight loss client presents to an emergency department with
multiple injuries, including chest trauma. A physician
notifies the care team that the client has progressed to
acute respiratory distress syndrome (ARDS) and

9
TRICIA JESHIEL P. TORRES, RN ENDOCRINE & RESPIRATORY DISORDERS

requests that the family be updated on the client’s


condition. The nurse should plan to discuss with the
family that:
A. the condition generally stabilizes with positive
prognosis.
B. the client can be discharged with home oxygen.
C. the condition is always fatal.
D. the condition is highly life-threatening and that
end-of-life concerns should be addressed.

14. A nurse observes for early manifestations of acute


respiratory distress syndrome (ARDS) in a client being
treated for smoke inhalation. Which signs indicates the
possible onset of ARDS in this client?
A. Cough with blood-tinged sputum and respiratory
alkalosis
B. Decrease in both white and red blood cell counts
C. Diaphoresis and low SaO2 unresponsive to
increased oxygen administration
D. Hypertension and elevated PaO2

15. A client learning about chronic obstructive


pulmonary disease self-care at a community health
class, asks a nurse why the participants are being
taught about the “lip-breathing.” The nurse should
respond by explaining that pursed-lip breathing can
help to:
A. reduce upper airway inflammation.
B. reduce anxiety through humor.
C. expel trapped carbon dioxide
D. increase effectiveness of inhaled medication

10
TRICIA JESHIEL P. TORRES, RN ENDOCRINE & RESPIRATORY DISORDERS

RESPI ENDO POST TESTS A. High risk for aspiration related to severe
1. Which of these signs suggests that a male vomiting
client with the syndrome of inappropriate B. Diarrhea related to increased peristalsis -
antidiuretic hormone (SIADH) secretion is hyperthyroidism
experiencing complications? C. Hypothermia related to slowed
A. Tetanic contractions - no connect metabolic rate - ↓T3 & T4
B. Neck vein distention D. Oral mucous membrane, altered related to
C. Weight loss - D.I disease process
D. Polyuria - D.I
7. The nurse should expect a client with
2. A male client is admitted for treatment of hypothyroidism to report which health
the syndrome of inappropriate antidiuretic concerns?
hormone (SIADH). Which nursing intervention A. Increased appetite and weight loss -
is appropriate? hyperthy
A. Infusing I.V. fluids rapidly as ordered - B. Puffiness of the face and hands -
B. Encouraging increased oral intake myxedema
C. Restricting fluids - FVE the PROBLEM C. Nervousness and tremors - hyper
D. Administering glucose-containing I.V. fluids D. Thyroid gland swelling - hyper
as ordered
8. The nurse is assessing a client with
3. Which outcome indicates that treatment of possible Cushing’s syndrome. In a client with
a male client with diabetes insipidus has been Cushing’s syndrome, the nurse would expect
effective? to find:
A. Fluid intake is less than 2,500 ml/day. - A. Hypotension.- fve → hypertension
normal B. Thick, coarse skin. → breakdown of
B. Urine output measures more than 200 protein → thin
ml/hour. - polyuria C. Deposits of adipose tissue in the trunk
C. Blood pressure is 90/50 mm Hg. - low and dorsocervical area. - buffalo hump
D. The heart rate is 126 beats/minute. -high D. Weight gain in arms and legs. - should be
thin
4. The nurse is developing a teaching plan for
a male client diagnosed with diabetes 9. The nurse evaluates no need for further
insipidus. The nurse should include instruction for self-care for patient with
information about which hormone lacking in Cushing syndrome who states:
clients with diabetes insipidus? A. “ I know I should have salt to everything I
A. antidiuretic hormone (ADH). eat.” - ↑ Na
B. thyroid-stimulating hormone (TSH). B. “ I make a point to avoid excessive
C. follicle-stimulating hormone (FSH). exposure to sun.” - irrelevant
D. luteinizing hormone (LH). C. “ I avoid to being exposed to anyone
with an infection.”
5. The client is diagnosed of having D. “I am careful to wear well-fitted shoes.” -
hyperthyroidism (Graves’ disease). Which of DM, foot care
the following is a drug of choice for his
condition? 10. A patient has diabetes type 2. The nurse
A. Furosemide (Lasix) -FVE has taught him about the illness and
B. Digoxin (Lanoxin)-HF evaluates learning has occurred when the
C. Propranolol (Inderal)-hypertension client makes which statement?
D. Propylthiouracil (PTU) A. “My cells have increased their receptors,
but there is enough insulin.”
6. The nurse is caring for a patient with a B. “My peripheral cells have increased
diagnosis of hypothyroidism. Which nursing sensitivity to insulin.” - decreased
diagnosis should the nurse most seriously C. “My beta cells cannot produce enough
consider when analyzing the needs of the insulin for my cells.” - for DM1
patient?

11
TRICIA JESHIEL P. TORRES, RN ENDOCRINE & RESPIRATORY DISORDERS

D. “My cells cannot use the insulin my C. Give 4 to 6 oz (118 to 177 mL) of orange
pancreas makes.” juice. = 15-15 rule
D. Administer insulin
11. A client with a diagnosis of diabetic
ketoacidosis (DKA) is being treated in the ER. 16. Which of this explains why epinephrine is
Which finding would a nurse expect to note given to patient having asthma? =
as confirming this diagnosis? bronchodilator
A. Elevated blood glucose level and a low A. It increases patients output thereby
plasma bicarbonate relieving the patient difficulty of breathing.
B. Decreased urine output - polyuria B. It dilates the bronchus thereby enable
C. Increased respirations and an increase in the patient to breath easily.
pH - acidosis C. It loosens mucous secretions of the
D. Comatose state patient.
D. It relieves dilation of the bronchus.
12. A nurse is preparing a plan of care for a
client with DM who has hyperglycemia. The 17. Which of the following positions would
priority nursing diagnosis would be: you place a client who is having an asthmatic
A. High risk for deficient fluid volume attack?
B. Deficient knowledge: disease process and A. lateral
treatment B. orthopneic
C. Imbalanced nutrition: less than body C. dorsal recumbent
requirements = should be more than D. supine
D. Disabled family coping: compromised.
18. Which of the following assessment
13. A nurse is caring for a client admitted to findings would help confirm a diagnosis of
the ER with DKA. In the acute phase the asthma in a client suspected of having the
priority nursing action is to prepare to: disorder?
A. Administer regular insulin A. Circumoral cyanosis
intravenously B. Increased forced expiratory volume
B. Administer 5% dextrose intravenously = C. Inspiratory and expiratory wheezing =
no, it contains glucose narrowed airway
C. Correct the acidosis D. Normal breath sounds
D. Apply an electrocardiogram monitor.
19. Which of the following types of asthma
14. A nurse performs a physical assessment involves an acute asthma attack brought on
on a client with type 2 DM. Findings include a by an upper respiratory infection?
fasting blood glucose of 120mg/dl, A. Emotional
temperature of 101, pulse of 88, respirations B. Extrinsic = allergy
of 22, and a bp of 140/84. Which finding C. Intrinsic
would be of most concern of the nurse? D. Mediated
A. Pulse
B. BP 20. A client with acute asthma showing
C. Respiration inspiratory and expiratory wheezes and a
D. Temperature = fever, infection, stress, ↑ decreased expiratory volume should be
cortisol ↑ clucose → HHNS treated with which of the following classes of
medication right away?
15. A 39-year-old company driver presents A. Beta-adrenergic blockers = should be
with shakiness, sweating, anxiety, and agonist
palpitations and tells the nurse he has Type I B. Bronchodilators = fast acting
Diabetes Mellitus. Which of the follow actions C. Inhaled steroids
should the nurse do first? = hypoglycemia D. Oral steroids
A. Inject 1 mg of glucagon subcutaneously. 21. Which of the following acid-base
B. Administer 50 mL of 50% glucose I.V. imbalance is present in the early stage of
asthma?

12
TRICIA JESHIEL P. TORRES, RN ENDOCRINE & RESPIRATORY DISORDERS

A. Respiratory alkalosis
B. Respiratory acidosis = late stage 26. A client with emphysema should receive
C. Metabolic acidosis only 1 to 3 L/minute of oxygen, if needed, or
D. Metabolic alkalosis he may lose his hypoxic drive. Which of the
following statements is correct about hypoxic
22. A 58-year-old client with a 40-year history drive?
of smoking one to two packs of cigarettes a A. The client doesn’t notice he needs to
day has a chronic cough producing thick breathe.
sputum, peripheral edema, and cyanotic nail B. The client breathes only when his oxygen
beds. Based on this information, he most levels climb above a certain point.
likely has which of the following conditions? C. The client breathes only when his
A. Adult respiratory distress syndrome oxygen levels dip below a certain point.
(ARDS) = blue bloaters D. The client breathes only when his carbon
B. Asthma dioxide level dips below a certain point.
C. Chronic obstructive bronchitis
D. Emphysema 27. A nurse is teaching an elderly client about
23. A 66-year-old client has marked dyspnea the importance of using a spacer that is
at rest, is thin, and uses accessory muscles attached to the inhaler. The nurse should
to breathe. He’s tachypneic, with a prolonged explain that a spacer:
expiratory phase. He has no cough. He leans A. allows for a greater amount of
forward with his arms braced on his knees to medication to be delivered.
support his chest and shoulders for breathing. B. lets the client see the medication as it is
This client has symptoms of which of the delivered.
following respiratory disorders? C. keeps the mouthpiece sterile.
A. ARDS D. allows for activating the medication
B. Asthma canister by simply inhaling
C. Chronic obstructive bronchitis
D. Emphysema 28. Wich of the following disorders is a
complication of COPD?
24. Its highly recommended that clients with A. Pulmonary hypotension = hypertension
asthma, chronic bronchitis, and emphysema B. Cor pulmonale
have Pneumovax and flu vaccinations for C. Left sided heart failure = right
which of the following reasons? D. Emphysema
A. All clients are recommended to have these
vaccines 29. A client was admitted to the hospital 24
B. These vaccines produce bronchodilation hours ago after sustaining blunt chest trauma.
and improve oxygenation. Which earliest clinical manifestation of acute
C. These vaccines help reduce the tachypnea respiratory distress syndrome (ARDS) should
these clients experience. the nurse monitor for?
D. Respiratory infections can cause A. Cyanosis and pallor
severe hypoxia and possibly death in B. Diffuse crackles and rhonchi on chest
these clients. auscultation
C. Increase in respiratory rate from 18 to
25. Clients with chronic obstructive bronchitis 30 breaths per minute
are given diuretic therapy. Which of the D. Haziness or “white-out” appearance of
following reasons best explains why? lungs on chest radiograph
A. Reducing fluid volume reduces oxygen
demand. 30. Which of the following positions would
B. Reducing fluid volume improves clients’ you place a client who has a diagnosis of
mobility. acute respiratory distress syndrome?
C. Restricting fluid volume reduces sputum A. Supine
production. B. Trendelenburg
D. Reducing fluid volume improves C. High fowler’s
respiratory function. D. Reverse Trendelenburg

13
TRICIA JESHIEL P. TORRES, RN PEDIATRICS

Date: February 20, 2022 hips of patient to prevent


Time: 2-8PM (PEDIA) brain hernation
Concept: PEDIA ■ Mannitol
Lecturer: PROF. ALENI IRAY ○ HOB slightly elevated → drain
CSF
MENINGITIS ○ Monitor VS
● Causative agent: ○ Measure Urine Output
○ Haemophilus Influenza → ■ Because of Mannitol
acute epiglottis ○ Assess LOC → GCS
○ Streptococcus Pneumonae →
Bacterial Viral
pneumonia
○ Neisseria Meningitidis → CHON ↑ >200g/dl <200g/dl
meningitis 15-45 g/dl
● Usually caused by respiratory
Turbidity Turbid Clear
diseases. From lungs to brain.
● Manifestations WBC ↑ 1000-2000 <300
○ Abrupt onset <8cells/μl lymphocyte
○ Fever
○ Chills Glucose ↓ Normal
○ Vomiting →
○ Seizure HYDROCEPHALUS
○ High-pitched crying and
bulging fontanels in younger
children
○ Irritability
○ Nuchal rigidity
○ Opisthtotonic region →
excessive arching of back
○ +Kernig’s Sign → knee, flexion
will cause hamstring muscle
pain = sign of meningococcal
● Choroid Plexus 400-600mL →
irritation
Arachnoid Villi (absorbs)
○ +Brudzinski Sign → nape
● 2-3-4
(batok), flexion will cause
● FAFA
flexion of the knee = sign of
● 2nd lateral ventricles → foramen of
meningococcal irritation
monroe → 3rd lateral ventricles →
● Monroe-Kellie doctrine
aqueduct of Sylvius → 4th lateral
○ 3 things constant in Brain
ventricles → foramen of luschka &
■ CSF
magendi → arachnoid villi
■ Brain volume
■ Intra Cerebral Blood
2 types:
○ Alterations in these ↑ ICP
1. Communicating
● Diagnostic tests
○ from Choroid Plexus
○ L3-L4 lumbar puncture
(CP) to Arachnoid Villi
● Management
(AV)
○ Respiratory isolation
○ Increased production
○ Antibiotic therapy →
of CP
Ceftriaxone (great penetrant
○ Or underabsorption of
from brain barrier)
AV
○ IVF Therapy → isotonic,
2. Non-communicating
0.9NaCl, PNSS
● Manifestations
○ Seizure precaution→dim light
○ Head enlargement
environment, safety, airway,
○ Bulging fontanels
duration of seizure
○ Separated suture lines
○ Control of increased ICP →
○ Dilated scalp vein
avoid unnecessary bending of

1
TRICIA JESHIEL P. TORRES, RN PEDIATRICS

○ Irritability ○ Translumination → check for


○ Child: crying, irritability fetal lung maturity in 3rd
○ Adult: confusion, trimester
lightheadedness ○ UTZ - locate placenta, locate
● Late signs fetus, locate amniotic fluid
○ Fontal enlargement pools
○ Sunset eyes ● Management
○ Lethargy ○ Prevent damage to the sac
○ Brief, shrill, high pitched cry, ■ Apply sterile, moist
headache non-adherent packs to
● Diagnostic evaluation prevent drying
○ CT SCAN ■ Sac is closely
○ MRI inspectedfor leaks or
○ Xray irritations
● Management ■ Sac must be cleansed
○ Removal of obstructing mass carefully if it becomes
○ Shunting solid
○ Spinal tap ○ Prevent complications
○ Support the head while ■ Prone positions
handling the patient ■ No diapers
○ Position on unoperated side ■ Soft foam
○ Keep child flat on bed ● Short term complication →
meningitis
NEURAL TUBE DEFECT ● Long term complication →
● Folic acid deficiency of pregnant hydrocephalus
mother
● Eggs, peanut, banana, organ meat DOWN SYNDROME
● Types
○ Anencephaly
■ Without a skull
○ Spina Bifida
■ Failure to fuse
■ Occulta - hidden →
Dimpling, tufts of hair
■ Cystica - sac with CSF
■ B1 meningocele - sac
with CSF with
meninges
■ B2 myelomeningocele
- sac with CSFwith
meninges & spinal
cord Physical assessment
● Flat, broad nasal brdige
● Inner epicanthal eye folds
● Upward, outward slanting of eyes
● Protruding tongue
● Short neck
● Simian crease
● Sandal sign
● Cystica - below L2
● Diagnostic evaluation Associated problems
○ Amniocentesin → first ● Cardiac defects
trimester, check for neural ● Respiratory infection
tube defects ● Feeding difficulties
● Delayed developmental skills
● Mental retardation

2
TRICIA JESHIEL P. TORRES, RN PEDIATRICS

○ Provide safe environment


Hemolytic Disease of the Newborn ○ No contact sports
○ Soft bristle toothbrush
RH incompatibility ● Recognize and control bleeding
● 1st 72hrs after delivery give rhogam ●

Erythroblastosis fetalis
● Very yellow baby → hemolysis
Hydrops fetalis SICKLE CELL ANEMIA
● ↓ O2 carrying cap ● Sickle cells that block blood flow to
● a condition in which large amounts organs deprive the affected organs of
of fluid build up in a baby's tissues blood and oxygen. I
and organs, causing extensive ● Manifestation
swelling (edema). ○ Growth retardation
ABO incompatibility ○ Chronic anemia
● Diagnostic evaluation ○ Delayed sexual maturation
○ Phototherapy ○ Marked susceptibility to
○ RhoGAM sepsis
○ Exchange transfusion ● Types
○ Vaso-occlusive → dehydration
■ Pain → morphine
sulfate
■ OFI
■ Dont give demerol
○ Sequestration crisis
■ Trapping of blood in
the liver, and pooling
of blood in spleen
■ Blood coming from
the spleen to the
systemic circulation is
Hemophilia lessened
● Types ■ Problem: hypovolemia
○ Factor VIII or hemophilia A ■ Splenectomy → high
○ Factor IX or hemophilia B risk for infection
○ Factor XI or hemophilia C ■ Spleen is responsible
● Manifestations for the maturation of
○ Prolonged bleeding WBC
○ Hemorrhage ■ Pneumovax 23
○ Easy bruising ○ Aplastic crisis
○ Hemarthrosis → hallmark sign ■ Decreased RBC
○ Spontaneous hematuria → ■ Exposure to benzene
classic sign ○ Hyperhemolytic crisis
● Replacement of the missing factor ■ G6PD deficiency
○ Factor VIII concentrate (glucose-6-phosphate
○ Cryoprecipitate dehydrogenase) →
○ DDVAP or 1-deamino-8-D maintains the shape of
arginine RBC
● Prevent bleeding ■ RBC will burst

3
TRICIA JESHIEL P. TORRES, RN PEDIATRICS

● Diagnostic evaluation
○ Sickledex (sickle turbidity test)
■ + turbid
○ HgB electrophoresis
■ Screens abnormal
blood
● Management
○ Bed rest
○ Oral & IV hydration
○ Electrolyte replacement 4. Coarctation of the aorta
○ Analgesic for pain - High bp upper extremities
○ Blood replacement - Low bp distal extremities
○ Antibiotics - Bounding pulse
○ O2 - Signs of arterial insufficiency
○ Exchange transfusion with o2
therapy

CONGENITAL HEART DEFECTS

Acyanotic Heart Defects


1. Atrial septal defects
- Patient may be
asymptomatic
- RV and PA enlarge 5. PuLmonic stenosis
- Bruit in 2nd-3rd Intercostal
Space, left
- May have some cyanosis of
CHF
- Characteristic murmur
- Evident cardiomegaly
- Management: balloon
angioplasty
2. Ventricular septal defect 6. AoRtic stenosis
- CHF - Bruit in 2nd-3rd Intercostal
- Bacterial endocarditis Space, right
- Ventricular hypertrophy - Manifestations
- Decreased CO
- Faint pulses,
hypotension,
tachycardia
- Chest pain
- Characteristic murmur
- Infection

3. Patent ductus arteriosus


CYANOTIC HEART DEFECTS
- May be asymptomatic
1. Tetralogy of Fallot
- Characteristic machinery like
a. Ventricular septal defect
murmur
b. Right ventricular hypertrophy
- Pulmonary vascular
c. Pulmonic stenosis
obstructive disease
d. Overriding of aorta
- Management
● Difficulty with feeding
- Indomethacin
● Failure to thrive
- Surgical ligation
● Episodes of bluish pale skin during
crying
● Exertional dyspnea

4
TRICIA JESHIEL P. TORRES, RN PEDIATRICS

● QRS prolongation may result from - Dopamine → + ino, +chrono


right ventricular dilatation - Morphine
● Management - Antibiotic
○ Palliative - Antihypertensives
○ Blalock trausig shunt
RHEUMATIC HEART FEVER
General manifestations
● Low-grade fever
● Unexplained epistaxis
● Abdominal pain
● Arthalgia without arthritic changes
● Weakness
● Fatigue
Major manifestations
● Carditis
● Migratory polyarthritis
● Chorea
● Subcutaneous nodule
Diagnostic evaluation
● Jones criteria → 2 major / 1 major, 2
minor
● ASO titer
● ESR
Management
● Srict bed rest
2. Transposition of the Great Vessel ● Monitor VS
● Management ● Oxygen therapy
○ To provide mixing → ● Bed cradle for optimum comfort
blalock-trausig ● Provide adequate nutrition
● Diagnostic exam Drugs
○ ECG ● Penicillin
○ Cardiac catheterization ● Erythromycin
○ ● Salicylates
● Prophylactic treatment

KAWASAKI DISEASE
● Constant fever
● Strawberry tongue

Diagnostic criteria
● Changes in peripheral extremities
● Polymorphous edema
● Bilateral, painless, bulbar
cionjunctival infection without
exudate
Management for Congenital Heart Disease Management
- Positioning ● High doses of IV gamma globulin
- Oxygen therapy ● Salicylate therapy
- Small frequent feedings
- No cnstrictuve clothing CYSTIC FIBROSIS
- Prepare for surgery Manifestations
- Drugs ● All secretions are viscious
- Cardiac glycoside ● Meconium ileus
- Diuretics ● Thich bronchial secretion
- Prostaglandin E ● Increased sweat chlorides
- Indomethacin ● Decreased fertility in females

5
TRICIA JESHIEL P. TORRES, RN PEDIATRICS

● Sterility in males ● Cleft palate - at age 18 months or


Evaluation older
● Iontophoretic pilocarpine test Preoperative care
● Duodenal analysis ● Feed in upright position
Management ● Place child on restraints periodically
● Pulmonary therapy ● Feed with rubber tipped asepto
● Gastrointestinal therapy syringe: small frequent feeding
○ Apple sauce mix with food Post operative care
but not hot ● Application of logan bow
● Aerosol therapy ● Elbow restraints
● Diet: clear liquid
PHENYLKETONURIA ● Prevent crying
Diagnostic evaluation ● Cleanse it free from serosanguinous
● Guthrie test fluid
○ Encourage breastfeedng ● Feed with cups: no bottles
● Phenistix ● Avoid straws, utensils, popsicle sticks,
● Ferric chloride chewing gums
Management ● Prone position/on the sides
● Dietary ● Avoid use of suction on the mouth
● If planning to be pregnant: 3 months ● Application of restraints (elbow and
before conception, no phenylalanine wrist)

ACUTE EPIGLOTTITIS TRACHEOESOPHAGELA FISTULA AND


Manifestation ATRESIA
● High fever ● Fistula - abnormal opening
● Sore throat ● Atresia - abnormal closing
● Dysphagia Manifestations
● Muffled voice, no hoarseness ● Excessive salvation and drooling
● Inspiratory stridor ● 3Cs of TEF: cough, choking, cyanosis
● Drooling ● Apnea
Causative agent ● Respiratory distress
● H. Influenza Diagnostic evaluation
Management ● Aspiration of stomach contents
● Keep child upright ● Radiography of abdomen
● Never use restraints Management
● Never use tongue blades ● Prevent aspiration pneumonia
● Corticosteroids → for inflammation
Hyperthropic pyloric stenosis
LARYNGOTRACHEOBRONCHITIS Manifestations
Manifestations ● Projectile vomiting
● Inspiratoiry stridor ● Non-bilous vomitus
● Suprasternal reractions ● Metabolic alkalosis
● Seal bark cough ● Weight loss
● Progressive hoarseness, followed by ● Palpable olive shaped mass at RUQ
aphonia ● Visible peristaltic movement
● Nasal flaring ● Distended upper abdomen
● Hypoxia ● Signs of dehydrations
● Management ● Constipation
● Corticosteroids Management
● Maintain hydration ● Pyloromyotomy
● Intubation if needed
● Semi fowler;’s position CELIAC DISEASE
Manifestations
CLEFT LIP/PALATE ● Impared fatnutrients absorption
● Surgical correction ● Behavioral changes
● Cleft lip - at age 2 months ● Celiac crisis

6
TRICIA JESHIEL P. TORRES, RN PEDIATRICS

Diagnostic evaluation Acute phase


● Jejunal biopsy ● Hematuria → tea cola urine
Management ● Oliguria-anuria
● No BROW - barley, rye, oat, wheat ● Increased specific gravity
● Proteinuria mild to moderate edema
INTUSSUSCEPTION ● Increased weight
● Manifestations ● Hypertension
● Sudden acute abdominal pain ● Diretic phase
● Passage of red currant jelly like stool ● Decreased weight
● Tender distended abdomen ● Decreased gross hematuria
● Palpable sausage-shaped mass in Diagnostic evaluation
RUQ ● Urinalysis
Diagnostic evaluation ● Increased ASO titer
● Barium enema ● Increased ESR
Management ● + throat culture for group A
● Barium enema Beta-Hemolytic Streptococci
● Pneumatic insufflation
● Laparotomy WILMS TUMOR

HIRSCHPRUNGS DISEASE

Nephroblastoma → capsulated
Agangliomic megacolon No massage
Manifestations No palpation
● Failure to pass meconium
● Reluctance to ingest food Manifestations
● Bile stained vomitus ● Abdominal swelling
● Abdominal distention ● Hematuria
● Foul breath ● Fatigue
● Ribbon-like stool ● Weight losss
● Visible peristalsis ● Fever
● Palpable fecal mass ● If + metastasis symptom of lung
Diagnostic evaluation involvement
● Xray ○ Dyspnea
● Rectal exam ○ Cough
● Rectal biopsy ○ SOB
● Barium enema ○ Chest pain
● Management
ACUTE GLOMERULONEPHRITIS ● Surgery
● Anti-neoplastic agents
○ Vincristine
○ B-interferons
● Diagnostic evaluation
● IV pyelogram

HYPOSPADIAS AND EPISPADIAS


Manifestations
● Abnormal placement of meatus
Manifestations
● Altered coding stream

7
TRICIA JESHIEL P. TORRES, RN PEDIATRICS

● Presence of chordee→ cobra like


appearance of penis
Management
● Urethroplasty

LEUKEMIA
Manifestation
● Bleeding tendencies
● Petechiae
● Nosebleeding
● Bleeding gums
● Ecchymoses

RANDOM BULLET POINTS


● Anterior Fontanel closes at 12-18
months
● Posterior Fontanel closes at 2-3
months
● Why only prick an infant, not
puncture → to prevent bone damage
● Fruit to decrease hyperactivity –?
Marang & durian
● Inotropic - something to do with the
force of the heart
● Chronotropic - something to do with
heart rate
● Great imitator - age 3
● 20 temporary teeth - age 2

8
TRICIA JESHIEL P. TORRES, RN PEDIATRICS

PEDIA POST TEST C. Cracking


1. What is the shape of an infant’s anterior D. Redness
fontanel?
A. Oblong 9. The best assessment for spina bifida
B. Triangle occulta is:
C. Diamond A. Sack with CSF
D. Square B. Sack with spinal cord
C. Sack with meninges
2. At what age does a child have a complete D. Dimpling
set of deciduous teeth?
A. 3 yrs old 10. (+)Brudzinski sign is best elicited by:
B. 2 yrs old A. Passively flexing the neck while patient
C. 5 yrs old lies prone results to flexion of knees.
D. 6 yrs old B. Passively flexing the neck while patient
lies to side results to flexion of knees.
3. A child aged 1 and a half years old is best C. Passively flexing the neck while patient
describe by nurse Beverly to be: lies prone results to extension of knees.
A. Having temper tantrums D. Passively flexing the neck while patient
B. The great imitator lies supine results to flexion of knees.
C. Selfish and impatient
D. To wild 11. Foods high in gluten includes the
following except:
4. A Patient is to undergo Guthrie test. The A. Pizza
nurse should instruct client to have which of B. Hotdogs
the following the day before the test? C. Sausage
A. Nothing per orem D. Rice
B. High CHON diet
C. High Gluten Diet 12. All of the following except one are
D. Increase fluid intake hallmark signs of nephrotic syndrome:
A. Hypoproteinemia
5. A breastfed infant’s stool is expected to B. Hypercholesterolemia
be? C. EDEMA
A. Having an offensive color D. Hypertension
B. Light yellow in color
C. Firm in consistency 13. A patient is having projectile vomiting for
D. Sour but clean odor the past 4 hrs. The complication best
associated with vomiting is:
6. Effect of cystic fibrosis to male babies A. Hypertension
includes: B. Metabolic Acidosis
A. sterility C. Metabolic Alkalosis
B. infertility D. Palpitations
C. Testosterone surge
D. BPH 14. Nurse must recognize that the long term
complication of spina bifida is?
7. Nurse must recognize that the short term A. Hydrocephalus
complication of spina bifida is? B. Meningitis
A. Hydrocephalus C. Learning disabilities
B. Meningitis D. Mental retardation
C. Learning disabilities
D. Mental retardation 15. A lumbar puncture was performed to a
child. The best position after the procedure
8. Nurse Paul is taking care of a baby with is?
Trisomy 21, the baby’s palm is expected to A. Flat on bed with 2 pillows
have: B. Flat on bed without pillow
A. Palmar creases C. Horizontal recumbent
B. Simian creases D. Semi-Fowler’s

9
TRICIA JESHIEL P. TORRES, RN PEDIATRICS

D. Rolls over from back to abdomen


16. In planning care for a teenager in sickle
cell crisis, the nurse should base any actions 23. The best position for the D.R. nurse to
on the knowledge that vaso-occlusive crises place the baby in myelomeningocele is:
arises from? A. Prone
A. Increased RBC destruction B. Supine
B. Hepatosplenomegaly C. Side-Lying
C. Occlusion of small blood vessels D. Trendelenburg
D. Sequestration of blood
24. A client with renal failue is having
17. Exaggerated arching of the back is difficulty in defecation. What is the best
termed as: nursing intervention for this?
A. Kernig Sign A. Increase fluid intake
B. Bruzinski sign B. Increase ambulation
C. Opisthotonus C. Laxative as ordered
D. Nuchal Rigidity D. Increase fruit in the diet

18. What will be the vomitus of a client with 25. What is the best diet for a patient with
pyloric stenosis? Nephrotic syndrome?
A. Stomach contents only A. Increased K
B. Stomach contents plus bile B. Increased Na
C. Stomach contents streaked with blood C. High Biologic Protein
D. Stomach contents stread with flecks of D. High CHO
feces
26. All of which contains high folate except:
19. Hyperhemolytic crisis results from A. Egg
deficiency in? B. Organ Meat
A. Amino acid C. Green Leafy Vegetables
B. Folate D. Squash
C. G-6-PD
D. Oxygen 27. Which among the following is not
classified as acyanotic heart defect?
20. To promote healthy teeth in a 2-yr old A. Tetralogy of Fallot
child, the nurse should: B. PDA
A. Teach child tooth brushing C. VSD
B. Encourage parents to provide tooth D. ASD
cleaning
C. Set-up dental opportunities 28. The most common sight for bleeding in
D. Teach children to rinse their mouth hemophilia is?
A. Eyes
21. Hemophilia is a bleeding disorder B. Abdomen
involving: C. Joints
A. Father as the carrier D. Skin
B. Mother as the one manifesting the
disease 29. The characteristic murmur in pulmonic
C. Mother passing the carrier state to the stenosis is located:
son A. Bruit on 2ND ICS to the left
D. Mother passing the carrier state to the B. Bruit on 2-3rd ICS to the right
daughter C. Bruit on the 5th ISC on the left
D. Bruit on 5th ICS to the right
22. Which one behavior should the nurse
expect in a 3-yr old to be capable of 30. The characteristic murmur in aortic
demonstrating? stenosis is located:
A. Climbs in stairs with alternate step A. Bruit on 2-3rd ICS to the left
B. Climbs in stairs one foot at a time B. Bruit on 2ND ICS to the right
C. Plays rattle C. Bruit on the 5th ISC on the left

10
TRICIA JESHIEL P. TORRES, RN PEDIATRICS

D. Bruit on 5th ICS to the right B. parents should play with child
C. provide sleeping
31. Complete repair of Transposition of the D. give Demerol
Great Vessel is:
A. Jatene Procedure 39. Nurse Lester is caring for a patient with
B. Blalock Procedure status epilepticus. Which nursing
C. Down’s Procedure intervention is most appropriate for the
D. Ren Procedure client?
A. IV injection
32. Best diagnostic test for Intussuception B. IM Demerol
is? C. give O2
A. Barium Enema D. put tongue depressor/blade
B. Angiogram
C. X-ray 40. The reason why to prick and not to inject
D. Doudenal Acidity Test or puncture in done in obtaining blood for
NBS?
33. Best sign in determining KAWASAKI A. prevent damage of tissue
disease is? B. prevent damage of blood vessels
A. Berry red Tongue C. prevent bone damage
B. L-R Shunting D. prevent muscle damage
C. R-L Shunting
D. High/Constant Fever 41. There are a lot of fruits in the market.
What type of fruits may help the client with
34. The 2 most common type of accidents hyperactivity?
among infants are: A. Marang and durian,
A. Accident and falls B. Apples and grapes,
B. Malaria and Dengue Fever C. Guava and atis,
C. Dengue and Kawasaki Disease D. Banana and papaya
D. Tetralogy of Fallot
42. The nurse is to take the blood specimen
35. If the child is having epiglottitis the nurse for NBS. The nurse should take the blood
must remember that: specimen from the newborns:
A. Eliciting gag reflex is avoided a. Umbilicus
B. There is no fever involve b. Heel
C. There is no inflammation involve c. Fingers
D. No need for intubation d. Brachial area

36. Before administering an injectable 43. Mella, a mother of a 6-year-old child


medication, as nurse you’ve learned that the having chicken pox asked the CHN nurse
proper injection site for infants is at: how to minimize itchiness. What would be
A. vastus lateralis, the most appropriate nursing intervention
B. gluteal area for the itchiness in patients with chicken
C. deltoid pox?
D. rectus femoris A. calamine lotion
B. talcum powder
37. In cystic fibrosis, the sweat chloride test is C. baking soda
expected to be: D. 70% alcohol
A. Decreased
B. Lofty 44. While admitting an infant, the nurse
C. Plummeted notes that which one is classic of a client
D. Normal with Hirschprung’s Disease?
A. Olive shape mass
38. Which of the following is appropriate for B. Projectile vomiting
the safety of the client with ADHD while C. Bile stained vomitus
playing? D. Diarrhea
A. padded toy

11
TRICIA JESHIEL P. TORRES, RN PEDIATRICS

45. The characteristic stool of a client with


intussusception is:
A. Black saffron
B. Currant jelly-like
C. Musty odor
D. Bloody

46. The characteristic mass in a client with


intussusception is:
A. Olive-shaped mass in LLQ
B. Olive-shaped mass in RUQ
C. Sausage-shaped mass in RUQ
D. Sausage-shaped mass in LUQ

47. Classic sign of hemophilia is:


A. Epistaxis
B. Nose bleeding
C. Spontaneous hematuria
D. Easy bruising

48. In Sickle Turbidity Test, blood is


considered sickled if the result is:
A. Turbid
B. Bluish
C. Cherry red
D. Cyanotic

49. Cystic Fibrosis can result to multiple


system disorder. As a nurse, which is your
priority?
A. Decrease fertility
B. Thick Bronchial Secretions
C. Sterility in Males
D. Meconium Ileus

50. A child is diagnosed with celiac disease.


Which of the following foods will you give
the child?
A. Rice oatmeal
B. Rice
C. Peanut Butter Sandwich
D. Wheat bread

ANSWER KEY
CBBBD ABBDD
DDCAB CCACC
DAACC DACAB
AADAA ABBCC
ABACB CDABB

12
TRICIA JESHIEL P. TORRES, RN RESEARCH

Date: February 6, 2022


Time: 9 AM - 4 PM CHAPTER 1: INTRODUCTION
Concept: RESEARCH Research Problem
Lecturer: PROF. RAYMUND MANAGO ● “What is wrong?”
○ Knowledge gap
○ Problems in society
VARIABLES ● Curiosity and interests
1. Independent variable: cause
2. Dependent variable: effect Statement of purpose
3. Confounding variable ● What do you want to happen?
● Goals and objectives
RELATIONSHIPS ○ Goals → general
○ Objectives → specific

Causal = effects
Associative = correlation

Research Question (PIRD)


● Population
○ Subjects / people
● Independent Variable
● Relationship
● Dependent variable

Among young adults (P) , is sedentary lifestyle (IV)


ETHICAL PRINCIPLES associated with (R) hypertension (DV)?
1. Beneficence: do good
2. Non-Maleficence: do no harm CHAPTER 2: RRL
3. Justice: equal risks and benefits Paraphrase + cite : to avoid plagiarism
4. Autonomy: voluntary decision
→ right to refuse/withdraw TYPES OF SOURCES
5. Veracity: complete information 1. Primary source
→ truth-telling → original study
6. Confidentiality: data not revealed → primary > secondary
→ data stored in secure computer 2. Secondary sources
7. Anonymity: data not linked to a person → reports based on an original study
→ remove identifying information (name, → critique, literature review
address, phone #, email, photos)
CHAPTER 3: METHODOLOGY
INFORMED CONSENT
Decision – Accept of decline participation QUANTITATIVE RESEARCH DESIGN
Protects autonomy
Elements (VICS): I. EXPERIMENTAL (True)
● Voluntary - no coercion, no bribery ● Randomized Controlled Trial (RCT)
● Informed - fully understood ○ Strongest design
● Competent - able to decide, (1) legal age, (2) ○ 3 elements (CRM)
coherent ■ Control group
● Signature - (1) subject, (2) witnesses: verifies
the signature of subject

1
TRICIA JESHIEL P. TORRES, RN RESEARCH

1. Descriptive
→ describe and document variables
→ no relationship established
→ ex: amount of alcohol intake (variable) of
Filipino young adults (Population)
2. Correlational
→ relationship between variables
→ ex: the relationship between alcohol intake
(IV) and happiness levels (DV) among Filipino
young adults (pop)
3. Prospective/Cohort
→ follow to the future
→ used in epidemiologic studies
Example:

→ longitudinal: multiple data collection


4. Retrospective / case-control
→ look back to the past
→ used in epidemiologic studies

■ Randomization - Random → cross-sectional: single data collection


assignment to exp and
control groups

QUALITATIVE RESEARCH DESIGNS


Identifies themes
■ Manipulation - giving
treatment/intervention to 1. Ethnography
EXP group → cultural patterns and lifeways
→ 2 perspectives:
II. QUASI-EXPERIMENTAL (“almost”) – emic: insider’s view (participant)
→ absence of randomization – etic: outsider’s view (researcher)
→ EXP and CTRL groups: non equivalent 2. Phenomenology

3. Grounded theory

III. NON EXPERIMENTAL


→ no manipulation (no treatment)
→ researchers are observers

2
TRICIA JESHIEL P. TORRES, RN RESEARCH

4. Case Studies → strata is a subgroup based on specific


characteristics.
→ Ex: year level
4. Purposive/ Judgemental sampling
→ hand-picking samples
→ inlcuison and exclusion criteria
5. Participatory action research (PAR)
B. Probability Sampling
→ random
→ all can be included
→ high representatives
1. Simple random
→ Uses sampling frame (list)
→ eg. draw lots, spin the wheel
2. Stratified random
SAMPLING → strata, random
→ selecting a portion of the population → “sample” → strata is a subgroup based on specific
→ Population (N), Sample (n) characteristics.
→ GOAL: representativeness, the sample must → Ex: year level
represent the population 3. Systematic sampling
→ uses sampling interval (k)
Sample size
1. Quantitative

4. Cluster/Multistage
→ successive sampling
Ph → regions→ provinces→ cities→
Brgy.--> TB patients
→ geographic locations

CHAPTER 4: DATA ANALYSIS


1. Descriptive statistics
→ simplest
→ synthesize and describe data
→ frequency and percentages
e– margin of error

Measure of central tendency
2. Qualitative
Mean: average
Data saturation:repetitive data
Median: middle
Mode: most frequent
TYPES OF SAMPLING

2. Inferential statistics
A. Non-probability
→ proves relationship between variables
→ non random
→ not all can be included
→ low representativeness

1. Convenience Sampling
→ conveniently available people
→ easiest, but most common
2. Snowball Sampling
→ referrals
3. Quota Sampling
→ strata, non-random

3
TRICIA JESHIEL P. TORRES, RN RESEARCH

RESEARCH PRETEST 7. You plan to conduct a survey on the eating habits


of preschoolers in a barangay. The population of
1. A researcher divided the province into cities and preschoolers is 1, 400 with a set margin of error of 5%.
randomly selected from those with the highest Compute for the sample size.
populations. He further randomly sampled those who a. 311
are aged 18 and above. What type of sampling did the b. 297
researcher employ? c. 302
a. Stratified d. 465
b. Simple Random
c. Systematic 8. In the RESEARCH question, “Among senior
d. Cluster citizens in Manila, is the free cinema screening
associated with their happiness levels?”, the “free
2. Which of the following is an example of a primary cinema screening” is referred to as:
source in a research study? a. Population
a. A published commentary on the findings of another b. Dependent variable
study c. Independent variable
b. A doctoral dissertation that critiques all research in d. Intervention
the area of attention deficit disorder
c. A textbook of medical-surgical nursing 9. The following are characteristics of a qualitative
d. An original journal article about cancer prevention design, except:
a. Naturalistic
3. What is the first step in the qualitative research b. Flexible
process? c. Controlled
a. Data analysis d. Narrative data
b. Sample
c. Review of literature 10. The title of a study is, “Discovering meaning in
d. Study design the experiences of Metro Manila college students who
moved from provinces.” The design of the study is most
4. The following are elements of a research likely:
question, except: a. Correlational
a. A relationship between two or more variables b. Randomized controlled trial
b. An operational definition of each variable c. Ethnography
c. The population being tested d. Phenomenology
d. Independent and dependent variables
11. A student nurse arrived at a sitio of Igorot people
5. Which criteria is used to determine testability of a with the purpose of observing and studying their culture
hypothesis? and lifeways. The design of the study is most likely:
a. Avoiding use of quantifiable words such as greater a. Ethnography
than or less than b. Case Study
b. A hypothesis stated in such a way that it can be c. Participatory action research
clearly supported or not supported d. Phenomenology
c. The use of biased words in a hypothesis
d. Data-collection efforts that prove the validity of the 12. Which of the following is a characteristic of a
hypothesis quantitative study?
a. Empirical – based on hunches and intuition
6. Given the following distribution of 10 scores from b. Firm/rigid research proposal
an exam of obstetric nursing, provide the mode score: c. Analysis of transcripts of interviews
95 90 89 89 90 91 92 89 93 94. d. Understands human experience
a. 94
b. 90 13. Nurse Nikki knows that the hallmark of a
c. 89 quasi-experimental design is the:
d. 91 a. Manipulation of independent variable
b. Presence of control group
c. Use of a treatment

4
TRICIA JESHIEL P. TORRES, RN RESEARCH

d. Absence of randomization

14. A prospective/cohort design involves the use of


multiple data collection points to gather complete data
about the subjects. Multiple data collection is called:
a. Longitudinal
b. Survey
c. Cross-sectional
d. Case-control

15. The results of a prospective study is considered


as more reliable than those of a retrospective study
because:
a. Prospective study is cheaper and requires less
resources.
b. The status of subjects can be followed and
monitored.
c. There is lesser recall bias in a retrospective study.
d. Prospective study is easier to conduct than
retrospective.

ANSWER KEY

DDCBB CACCD
ABDAB

5
TRICIA JESHIEL P. TORRES, RN RESEARCH

RESEARCH POST TEST a. Verbal assent


b. Assent form
1. Research refers to a systematic and disciplined c. Informed consent form
inquiry that seeks to answer questions and solve d. No assent needed
problems using a structured series of steps. The goal of
clinical nursing research is to: 6. An ICU nurse taking up her master’s degree came
a) Understand challenges and create improvements in up with the research title “Effects of Hospital Visitation
nursing education. on Anxiety among Intensive Care Unit Patients in Cebu”
b) Develop and expand the body of scientific for her thesis. Which is the independent variable?
knowledge. a. Intensive Care Unit Patients
c) Update best practice in nursing and improve quality b. Anxiety
of life. c. Cebu
d) Prove that sound evidence is an excellent basis for d. Hospital Visitation
clinical nursing practice
7. In the same research title mentioned above,
2. Quantitative research is different from qualitative which is the dependent variable?
research in all the following ways except: a. Intensive Care Unit Patients
a. Quantitative research is based on numerical and b. Hospital Visitation
narrative data, while qualitative research is based on c. Anxiety
objective and statistical inputs. d. Effects
b. Qualitative research is best conducted in a location
which the subjects consider as their habitat. 8. A nursing student wanted to study whether health
c. Increasing validity and decreasing bias is essential education affects the knowledge on self-care practices
for quantitative research. among patients with diabetes mellitus. Which of the
d. In qualitative research, the design and procedures following would be his independent variable?
are seen as flexible and allows for change. a. Self-care practices
b. Diabetes Mellitus
3. Which of the following statements demonstrate an c. Knowledge
associative relationship? d. Health education
a. An increase in the serum sodium level of a client
caused the increase in blood volume and pressure. 9. In the above study, which of the following is the
b. A black cat that passed by the road caused bad luck dependent variable?
to a motorist and suffered from a vehicular accident. a. Self-care practices
c. Sedentary lifestyle of Ms. Patricia is more likely to b. Diabetes Mellitus
increase her chances of developing diabetes mellitus. c. Knowledge
d. A fire alarm that set off in the building has resulted in d. Health education
rapid evacuaiton of the tenants.
10. Which is the dependent variable in the following:
4. A participant in your study is asking if you can “The Role of Feedback in Enhancing Students’
provide him the treatment dosage that is 5 times higher Academic Performance in Colleges”?
than prescribed. Upon reading the BON Resolution No. a. Academic Performance
220, the best response is to: b. Colleges
a. Follow the wishes of the participant in order to protect c. Feedback
right to self-determination. d. Students
b. Decline the request since this may deplete the funds
of the research project. 11. In the above study, which is the independent
c. Explain that the priority of the study is to protect the variable?
safety of the participants at all times. a. Academic Performance
d. Report the statement of the participant to your b. Colleges
co-researchers to discuss possible actions to take. c. Feedback

5. On the other hand, if the subject is already 18 d. Students


years old or above, which document is used to protect
their autonomy?

6
TRICIA JESHIEL P. TORRES, RN RESEARCH

12. A nurse seeks to determine the barriers to a. In quasi-experimental designs, there are possible
contraceptive use among postpartum adolescents. She confounding variables.
identified the following inclusion criteria in recruiting her
subjects: (1) aged 14 – 17 years old; and (2) had been b. Experimental designs are more practical and
pregnant. The exclusion criteria were: (1) acutely ill or in acceptable.
a medical emergency; and (2) mental or physical c. Quasi-experimental designs offer more flexibility in
disabilities. What kind of sampling design did the nurse performing research.
employ? d. Experimental designs involve manipulation.
a. Convenience
b. Purposive 18. In participatory action research (PAR),
c. Stratified Random researchers and participants collaborate in conducting
d. Cluster research and empowering people. Which of the
following is the type of PAR?
13. Which of the following sampling designs would a. Qualitative
least likely result in selection threat to validity? b. Quantitative
a. Snowball c. Action research
b. Quota d. Survey
c. Simple Random
d. Purposive 19. Which of the following is a type of probability
sampling method?
14. Given the research title “The relationship a. Simple random sampling
between clinical performance and years of experience b. Purposive sampling
of staff nurses in Manila,” which is the independent c. Quota sampling
variable? d. Snowball sampling
a. Clinical performance
b. Staff nurses 20. Nurse Minnie interpreted the Pearson’s r value in
c. Manila her study as low positive correlation. Which of the
d. Years of experience following values might she have computed?
a. + 0.45
15. The following are the elements of an b. – 1.0
experimental design. Which of the following is most c. + 0.21
capable of strengthening the claim of the experiment? d. + 0.82
a. Control group
b. Randomization 21. A study would like to determine the effect of a
c. Manipulation novel drug on the blood glucose levels of patients with
d. Integration diabetes mellitus. The type of relationship being
LM POST-TEST determined is:
15 of 15 points a. Associative
b. Causal
16. Nurse Kim seeks to understand the correlation of
hair dye and skin cancer. She gathered 2 groups of c. Both A and B
women who do and do not have skin cancer and asked d. Neither A nor B
which among them regularly dyed their hair in the past.
She utilized which design? 22. A researcher would like to know the correlation
a. Cohort between two variables. This means that the type of
b. Prospective relationship he aims to establish is:
c. Longitudinal a. Associative
d. Retrospective b. Causal
c. Both A and B
d. Neither A nor B
17. Why do quasi-experimental designs yield
conclusions that are weaker that those from 23. Which of the following demonstrates
experimental designs? convenience sampling?
a. Asking for student nurses to volunteer as subjects

7
TRICIA JESHIEL P. TORRES, RN RESEARCH

b. Selecting the subjects through lottery method 30. The following are components of a research
c. Recruiting participants through social media networks question, except:
d. Using random numbers to number the population a. Independent variable
b. Population
24. Which of the following demonstrates systematic c. Comparison
sampling? d. Relationship
a. Selecting every 15th person in the population
b. Conducting 4 successive randomizations ANSWER KEY
c. Defining the exclusion criteria
d. Asking for referrals CACCC DCDCA
CBCDB DACAC
25. Which of the following statements is true BAAAB AABBC
regarding sampling?
a. Non-probability sampling is better than probability
sampling.
b. Probability sampling has high representativeness.
c. Non-probability sampling gives a chance for all to be
selected.
d. Probability sampling involves non-random methods.

26. Which statement best describes qualitative


research?
a. Studies are conducted in natural settings.
b. Data are collected from a large number of subjects.
c. Data collected tend to be numeric.
d. The research design is systematic and objective.

27. What does a critique of a research study always


include?
a. Determining its strengths and weaknesses
b. Researching similar studies
c. Using critical listening skills
d. Explaining your own personal opinions

28. For which of the following research questions


would qualitative methods be most appropriate?
a. Which pain medications decrease the need for sleep
medication in elderly patients?
b. What is the meaning of health for migrant
farm-worker women?
c. Under what conditions does a decubitus ulcer heal
most quickly?
d. How does frequency of medication administration
impact the degree of pain experienced following knee
replacement surgery?

29. Given the following distribution, which is the


median score? 7 4 2 9 1 3 3 2 7.
a. 4
b. 3

c. 2
d. 7

8
TRICIA JESHIEL P. TORRES NURSING LEADERSHIP MANAGEMENT

Date: February 20, 2022


followers are and have
Time: 7am-1pm willing to authority to
Concept: LM follow enforce
Lecturer: PROF. ALENI IRAY decision

Relationship In intuitive Relate to


Nursing leadership and people
● Process whereby a nurse influences empathetic according to
manner rule
one or more persons to achieve
specific goals in the provision of Reward Rewarded Fulfill
nursing care for one or more patients from personal organizational
achievement mission
Nursing management
Position May or may Managers as
● Process of working through the not be long
nursing staff personnel or members successful appointment
to provide care, cure, and comfort to holds
patient Rules Interested in Cary out
risk-taking predetermined
Level of Management and exploring policies rules
new ideas and regulations
● Top → chief nurse
● Middle → supervisor Others Influence Maintain an
● Low → head nurse others toward orderly,
goal setting controlled
national
Types of Scheduling
equitable
1. Centralized scheduling → based on structure
the master staffing pattern. Mostly
carried out by one person; chief
Scientific Management
nurse.
● Frederick W. Taylor
2. Decentralized scheduling → carried
○ Proposed set of techniques to
out by middle & lower level
increase efficiency and
managers; head nurse.
productivity of each worker
3. Self-scheduling
○ Automation → observation,
4. Permanent scheduling
experimentation, analysis
5. Alternating or rotating work shift →
● Henri Fayol’s
personnel have alternating work
○ Father of modern
hours among days, evenings, and
management theory/process
nights (7am-3pm or 3pm-11pm)
○ 14 principles of management
6. Block / Cyclical scheduling →
1. Division of labor -
staffing pattern repeats itself every
according to their
4-6 weeks
knowledge and
7. Variable scheduling → determining
expertise
the number and mix of staff based
2. Authority - managers
on patient’s need
need to be able to give
○ Mix of staff = ratio of
orders so that they can
professionals to
get things done
non-professionals (RN:NA)
3. Discipline - members
8. Flex time → excess hours from
in the organization
overtime can be used to adjust
must respect the rules
tomorrow’s schedule
and the organization
that govern them
Leader Manager 4. Unity of command -
each employee must
Appointment May or may Officially
not have appointed receive his/her
official instructions about a
appointment particular operation
from 1 person
Power As long as the Have power

1
TRICIA JESHIEL P. TORRES NURSING LEADERSHIP MANAGEMENT

5. Unity of direction - change to maximize


those operations efficiency
within the 13. Initiative - employees
organization that have should be encouraged
the same objectives to develop and carry
should be directed out plans for
with only one plan improvement
6. Subordination of 14. Esprit de corps -
individual interest to organizations should
a common goal – in strive to promote team
any undertaking the spirit, unity, and
interest of the morale
employees should not ● Max Weber’s Bureaucratic
take precedence over organizational theory
the interest of the ○ Contributed to the
organization as a development of a classical
whole approach to organizational
7. Remuneration - design
compensation for ○ Tasks must be divided
work should be fair to ● Elton Mayo’s Behavioral School
both employees and ○ Hawthorne effect →
employers individuals react differently
8. Centralization - when they know that they are
balancing centralized being observed
decision-making (from ● Douglas McGregor’s Motivational
the top) with letting Theory
employees make
THEORY X THEORY Y
decisions. “A place for
everyone and Lazy Loves to work
everyone in his place”
9. Scalar chain - To exercise To provide
strict control opportunities for the
employees should
over his subordinates to be
know where they
subordinate to creative and productive
stand in the ensure that the and to encourage
organization’s work is carried growth and provide
hierarchy and who to out properly guidance
speak to within a chain
of command Be strict Provide opportunity
10. Order - “the right man
in the right place” ● William Ouchi’s Theory Z
forms an effective ○ Long term employment
social order. He
applied the same LEADERSHIP THEORIES
maxim to materials: 1. The Great Man Theory - leaders are
right one, right place born not made
11. Equity - managers 2. Trait theory - born with a
should be fair to all characteristic of a leader (charisma,
employees through a intelligence)
“combination of 3. Situational theory
kindliness and justice” ○ LEADERSHIP STYLES
12. Stability of tenure of i. Autocratic - High
personnel - concerns for task
organizations should accomplishment;
minimize staff Leader-oriented
turnover and role ii. Democratic -
People-oriented focus

2
TRICIA JESHIEL P. TORRES NURSING LEADERSHIP MANAGEMENT

on human aspects and


builds effective KINDS OF BUDGETS
teamwork. The leader 1. Direct Cost
acts as a facilitator ○ Directly related to patient
iii. Permissive / care within a manager’s unit
Laissez-faire - Leaders ○ Salary, supplies
appointed as 2. Indirect Cost
managers ○ Not directly related but
4. Contingency theory necessary to provide care
○ Task structure ○ Electricity, maintenance
○ Position power 3. Fixed Expenses
○ Relationship ○ Don't vary in volume
5. Path goal theory ○ Rent, monthly salary
○ A leader’s traits & behaviors 4. Variable Expenses
can directly affect the ○ Increase or decreased
satisfaction, motivation & depending on the # of
performance of their team patients
members ○ Patient meals, laundry of
linens
I. PLANNING 5. Controllable Cost
● Forecasting of events, making ○ Controlled and varied by
decisions, setting goals, setting managers
priorities, developing policies, and ○ Staffing pattern & mix
setting standard 6. Uncontrollable Cost
○ Equipment depreciation,
Types of planning overtime in response to
I. Strategic planning emergency
- Long-range goal
II. Operational planning II. ORGANIZING
- Deals with the day-to-day ● Grouping of activities for the purpose
maintenance of activity of achieving objectives
Organization: to accomplish goals and
Mission objectives of the organization efficiently
● Reasons for the organization's human and material resources are essential
existence
● States the services the organization
INFORMAL FORMAL
will provide
● DOH: ensure accessibility and quality Consists of personal Describes the
of health care to improve the quality and social positions, the
of all Filipinos, especially the poor relationships responsibilities of
Vision among members of those occupying
the organization the positions and
● What you want to become in the
the working
future Meets the needs of relationships
● DOH: Filipinos are among the individuals with among the various
healthiest people in SEA by 2022 similar units or
Philosophy backgrounds departments
● Reflects the values and beliefs of the
organization TYPES OF AN ORGANIZATION CLASSIFIED
Goals and Objectives BY NATURE OF STRUCTURE
● Goal: general, long term 1. Line organization
○ Ex. to improve ○ Simplest and most direct type
documentation in which position has general
● Objective: specific, short-term authority over the lower
○ Ex. an increase in form positions in the hierarchy
completion by 5% in the first 2. Informal organization
quarter of 2023

3
TRICIA JESHIEL P. TORRES NURSING LEADERSHIP MANAGEMENT

○ Refers to horizontal produce a desired level of care and to


relationships meet the patient's need
3. Staff organization ● RA 5901 - forty-hours/week law
○ Purely advisory ○ > 100-bed capacity → RNs will
○ Broken lines work 40hrs/wk (5 days)
4. Functional organization ○ < 100-bed capacity → RNs will
○ Each unit is responsible for a work 48hrs/wk (6 days)
given part of the organization ● NHPPD - standard measure to
nursing time
Before you delegate ● RA 4226 - hospital licensure act
- Check for potential harm ● RA 7160 - local government code
- Check 5 rights of delegation
1. Right task NURSING CARE DELIVERY SYSTEM
2. Right circumstances ● Case method
3. Right person ○ Pros: total patient care
4. Right direction ○ Cons: different nurse, different
5. Right supervision/evaluation shifts, different days
● Functional system
Vertical chart ○ Pros: efficiency
● Depicts the chief executives at the ○ Cons: nurses do managerial
top with lines of authority following work. Nurses aides do patient
down the hierarchy care
● Team nursing
Horizontal chart ○ Pros: team effort
● Depicts the manager at the top with ■ Frees patient care
aside span of control coordinator to
● Communication in this structure is manage the unit
direct simple and fast with minimal ■ Nursing care plan
distortions of the message since the ■ Nurses are divided into
distance between the top and the groups
lower level is shorter ● Primary nursing
○ Admission to discharge
PRINCIPLES OF ORGANIZATION ○ Pros: RNs give total patient
1. Communication - top to bottom care
a. Downward = top level → ■ Primary nurse has 24
subordinate (memo, hr responsibility
termination) ○ Cons: confines nurses’ talents
b. Upward = subordinate → top to a limited number of
level (incident report, patients
resignation) ■ Associate nurse may
c. Horizontal = same level change care plan
(endorsement) without discussing
d. Diagonal = interdepartmental with primary nurse
2. Unity of command
3. Span of control TYPES OF POWER
4. Specialization 1. Reward power
*Chain of command refers to the orderly 2. Punishment or coercive power
command hierarchy within an incident 3. Information power
management organization. Unity of 4. Legitimate power
command means that each individual 5. Expert power
reports to only one designated supervisor. 6. Referent power

III. STAFFING Transformational leader → committed and


● Process of determining and has the vision to empower others
providing the acceptable number
and mix of nursing personnel to

4
TRICIA JESHIEL P. TORRES NURSING LEADERSHIP MANAGEMENT

LM PRE TEST D. Organizing


1. As a nurse manager, which of the
following best describes this function? 8. Bruce was chosen as one of the leaders
A. initiate modification on clients lifestyle in their community. He observed that his
B. protects clients life subordinates are not responding well to him
C. coordinate the activities of other and that they are all passive. He then
members of the health care team in changed his leadership style from
managing patient care democratic to authoritarian form of
D. provide in service education program, use leadership. What theory did he use, base on
accurate nursing audit, formulate leadership theories.
philosophy and vision of the institution A. The great man theory
B. Trait Theory
2. These are nursing intervention that C. Contingency Theory
requires knowledge, skills and expertise of D. Situational Theory
multiple health professionals
A. dependent 9. An example of a management function
B. independent of a nurse is:
C. interdependent A. directing and evaluating the staff nurse
D. intradependent B. Performing nursing procedures for clients
C. Teaching patient do breathing and
3. They may or may not have official coughing exercises
appointment D. Preparing for a surprise party to the client
A. Appointment
B. manager 10. Your head nurse in the unit believes
C. Leader that the staff nurses are not capable of
D. Officially appointed decision making so she makes the decisions
for everyone without consulting anybody.
4. Lelian a nurse manager observes that This type of leadership is
her employees are lazy and can’t work A. Democratic leadership
independently. Base on motivational theory B. Autocratic leadership
where can we classify this set of employees? C. Managerial Leadership
A. Theory A D. Laissez Faire Leadership
B. Theory B
C. Theory Y 11. Father of modern management:
D. Theory X A. Henri Fayol
B. Max Weber
5. Long Term form of Planning; C. Elton Mayo
A. improves efficiency D. Douglas McGregor
B. strategic planning
C. operational planning 12. Joedel nurse manager observes that her
D. mission employees ca work independently. Base on
motivational theory where can we classify
6. Knowing that these are the this set of employees?
characteristic of her subordinates what A. Theory A
leadership style would she use? (Refer to B. Theory B
question 4 for the situation.) C. Theory Y
A. authoritarian D. Theory X
B. democratic
C. laisez faire 13. Employees has one supervisor and there
D. leadership style is one plan for a group of activities with the
same objectives only to one supervisor
7. A statement that identify the reason of A. Unity of command
an organization: B. Span of Control
A. Philosophy C. Unity of direction
B. mission D. Specialization
C. objectives

5
TRICIA JESHIEL P. TORRES NURSING LEADERSHIP MANAGEMENT

14. One of Nursing Care delivery system d. Synergistic.


where task are being divided.
A. Case System 4. Which of the following statements about
B. Functional System situational or contingency leadership theory
C. Team Nursing is true?
D. Primary Nursing a. High relationship behavior is much more
essential to a good manager than high task
15. Doing phase of management Process; behavior.
A. Leadership b. This leadership model is effective in
B. Planning bureaucratic organizations because it is task
C. Directing focused.
D. Management c. Management should be consistent in
different situations so workers understand
ANSWER KEY: CCCDB ABDAB ACXXC what is expected of them.
d. The leadership style chosen by a manager
should reflect the task/relationship behavior
LM POST TEST of those being managed.

SITUATION: A nurse leader who has 5. Which of the following statements depicts
a considerable amount of work leadership?
experience would select a leadership A leadership position is assigned.
and management style that would b. A leadership position carries a legitimate
work best in any circumstance. source of power.
c. Members of a group will only follow a
1. A nurse manager has the reputation of person in a leadership position by choice.
being an autocratic leader. Which of the d. Leadership requires meeting
following statements by this manager organizational goals
would support that reputation?
a. "I'm putting a suggestion box in the break 6.Which of the following is characteristic of
room if anyone has ideas that would be theory Z?
helpful to the unit" a. Nonconsensus decision making
b. "The new work schedule is posted for the b. Fitting employees to their jobs
next 6 weeks" c. Rapid career promotions
c. "I put the new procedure manual out. d. Authoritative problem solving
Please add your comments to the blank
sheet of paper attached to the front " 7. The person who is committed, has a
d. "I'd like to hear from you (addressing the vision, and is able to empower others with
staff) what your ideas are for promoting this vision is termed a(n)
better morale in this unit" a. Transactional leader
b. Transformational leader
2 .The nurse manager who practices the c. Interactional leader
leadership style that is leader-focused is d. Bureaucratic leader
demonstrating which type of
a. Democratic 8.Which explains why nurses must be
b. Laissez-faire leaders to be successful in client care?
c. Autocratic a. Nurses must be effective at giving orders.
d. Synergistic b. Only nurses as leaders have physician
respect.
3.The leader of a work group acts as a c. Nurses must direct client care activities to
facilitator, serves as a resource person, and reach outcomes.
encourages members to actively d. Only nurses as leaders can function in
participate. This leadership style would be managed care.
described as:
a. Democratic 9.Which indicates leadership or
b. Laissez-faire management positions in an organization
c. Autocratic. with legitimate authority?

6
TRICIA JESHIEL P. TORRES NURSING LEADERSHIP MANAGEMENT

1. Formal leaders 3. Nurse managers nurse hired Nimfa and told her that she can
2. Staff nurse 4. Informal leaders start. The chief nurse demonstrated which
a. All except 4 basic role in management?
b. 1 and 3 only a. Informational role
c. 2 and 3 only b. Decisional role
d. All of the above c. Liaison role
d. Interpersonal role
10. The nursing leader has initiated a
measure to solve a problem. This leader is 15. Chester Barnard’s acceptance theory of
demonstrating which characteristic authority is described as:
a. Critical thinking a. Naturally forming social groups that can
b. Delegation become contributors to an organization
c. Risk taking b. An individual worker who is the source of
d. Initiating action control, motivation, and productivity in an
organization
SITUATION: For nurses, knowledge of the c. People who have free will and choose to
management process assures them of the comply with orders they are given
smooth functioning of their units to attain d. People who benefit and are productive
their goal of quality care through the when they participate in decisions about
judicious use of available human and their work
material resources within specified periods.
16. The theory that includes maintenance
11. Managing conflict among staff members and motivation factors is:
would occur in which part of the a. Maslow’s hierarchy of needs
management process? b. Herzberg’s two-factor theory
a. Planning c. McGregor’s theory X and theory Y
b. Organizing d. Ouchi’s theory Z
c. Directing
d. Evaluating SITUATION: Planning is defined as
predetermining a course of action in order
12. Chief Nurse Donna is concerned on to arrive at a desired result.
developing and emphasizing definite and
consistent policies and procedures in 17. A value statement of the principles and
dealing with the staff nurses. This action by beliefs that direct the organization’s
chief nurse Donna reflects which of the behavior is called:
three-fold concept of management? a. Mission
a. System of authority b. Philosophy
b. An economic resource c. Purpose
c. Elite class d. Strategic plan
d. A level of coordination
18. Most organizations have mission
13. According to Hospital Licensure Law statements that address the following:
defines the term hospital as a place devoted a. Educating professionals
primarily to the maintenance and operation b. Promoting research
of facilities for the diagnosis, treatment, and c. Providing quality of care
care of individuals suffering from illness, d. Providing care
diseases or deformity, or in need of
obstetrical or other medical and nursing 19. The pivotal value of strategic planning is
care. This law is otherwise known as: that it requires an organization to focus on:
a. RA 4226 a. Internal environments
b. RA 7160 b. External environments
c. RA 4236 c. Change
d. RA 7060 d. Mission

14. Nurse Nimfa is applying as a staff nurse 20. One of the purposes of a strategic plan is:
at a local community hospital. The chief a. Various ideas are accepted

7
TRICIA JESHIEL P. TORRES NURSING LEADERSHIP MANAGEMENT

b. Resources are available d. Administration’s insight to financial


c. Diverse visions are accepted outcomes
d. Process varies
27. A standard measure that quantifies the
21. When planning for expenses that are nursing time available to each patient by
directly associated with patient care, it is available nursing staff is called:
known as: a. FTE
a. Fixed costs b. NHPPD
b. Indirect expenses c. Productive time
c. Direct expenses d. Nonproductive time
d. Variable costs
28. When a problem arises in the
22. Nurse Andrea, a nurse manager, is organization, Nurse Valerie first discusses it
preparing a budget plan for the year 2012. with the head nurse before bringing the
Costs like equipment depreciation,utilities, matter to the chief nurse. She is observing
fringe benefits, and salaries are considered: the principle of:
a. Fixed costs a. Chain of command
b. Variable costs b. Decentralization
c. Direct expenses c. Span of control
d. Indirect expenses d. Unity of command

23. Which of the following accounts for 29. As the unit manager, Nurse Tanya
approximately 50%-60% of operational costs: realizes that there is a limit on the number
a. Supplies of subordinates she can handle. This
b. Equipment principle is:
c. Salaries and benefits a. Span of control.
d. Pharmaceuticals b. Unity of command
c. Chain of command
24. Sick time, vacation time, and holiday d. Channel of communication
time is considered:
a. Productive time 30. A registered nurse in charge of the
b. Unproductive time nursing unit is preparing the assignments
c. Direct expenses for the day. The RN assigns a nursing
d. Variable costs assistant to make beds and bathe one of the
clients on the unit and assigns another
25. The Forty-Hour Week Law (Republic Act nursing assistant to fill the water pitchers
5901), provides that employees will work and serves juice to all the clients. Another
only 40 hours a week provided RN is assigned to administer all medications.
a. 50-bed capacity Based on the assignment designed, which
b. 75-bed capacity type of nursing care is being implemented?
c. 100-bed capacity a. Functional nursing
d. 150-bed capacity b. Team nursing
c. Exemplary model of nursing
SITUATION: Organizing is the d. Primary nursing
function of management which
follows planning. It is a function in ANSWER KEY:
which the synchronization and BCADC BBCDD
combination of human, physical and CABAC BBCDB
financial resources takes place. CACBC BBAAA

26. Overall, organization performance is


dependent on:
a. Managers’ insight to financial outcomes
b. Staff’s insight to quality and financial
outcomes
c. Organization’s strategic plan

8
Nursing Board Practice Test Compilation

FOUNDATION OF PROFESSIONAL NURSING PRACTICE 188

Contents ANSWER KEY - FOUNDATION OF PROFESSIONAL


NURSING PRACTICE I: FOUNDATION OF NURSING NURSING PRACTICE.................................................. 199
PRACTICE .......................................................................... 4 COMMUNITY HEALTH NURSING AND CARE OF THE
NURSING PRACTICE II ..................................................... 15 MOTHER AND CHILD .................................................... 200

NURSING PRACTICE III .................................................... 26 ANSWER KEY: COMMUNITY HEALTH NURSING AND
CARE OF THE MOTHER AND CHILD .......................... 211
NURSING PRACTICE IV.................................................... 36
Comprehensive Exam 1................................................ 213
NURSING PRACTICE V..................................................... 46
CARE OF CLIENTS WITH PHYSIOLOGIC AND
TEST I - Foundation of Professional Nursing Practice .... 56
PSYCHOSOCIAL ALTERATIONS...................................... 222
Answers and Rationale – Foundation of Professional
ANSWER KEY: CARE OF CLIENTS WITH PHYSIOLOGIC
Nursing Practice ......................................................... 66
AND PSYCHOSOCIAL ALTERATIONS ......................... 234
TEST II - Community Health Nursing and Care of the
Nursing Practice Test V ................................................ 235
Mother and Child ........................................................... 74
Nursing Practice Test V ................................................ 245
Answers and Rationale – Community Health Nursing
and Care of the Mother and Child ............................. 84 TEST I - Foundation of Professional Nursing Practice .. 255

TEST III - Care of Clients with Physiologic and Answers and Rationale – Foundation of Professional
Psychosocial Alterations ................................................ 91 Nursing Practice ....................................................... 265

Answers and Rationale – Care of Clients with TEST II - Community Health Nursing and Care of the
Physiologic and Psychosocial Alterations ................ 102 Mother and Child ......................................................... 273

TEST IV - Care of Clients with Physiologic and Answers and Rationale – Community Health Nursing
Psychosocial Alterations .............................................. 111 and Care of the Mother and Child ........................... 283

Answers and Rationale – Care of Clients with TEST III - Care of Clients with Physiologic and
Physiologic and Psychosocial Alterations ................ 122 Psychosocial Alterations .............................................. 290

TEST V - Care of Clients with Physiologic and Psychosocial Answers and Rationale – Care of Clients with
Alterations.................................................................... 133 Physiologic and Psychosocial Alterations ................ 301

Answers and Rationale – Care of Clients with TEST IV - Care of Clients with Physiologic and
Physiologic and Psychosocial Alterations ................ 144 Psychosocial Alterations .............................................. 310

PART III PRACTICE TEST I FOUNDATION OF NURSING . 153 Answers and Rationale – Care of Clients with
Physiologic and Psychosocial Alterations ................ 321
ANSWERS AND RATIONALE – FOUNDATION OF
NURSING .................................................................. 158 TEST V - Care of Clients with Physiologic and Psychosocial
Alterations.................................................................... 332
PRACTICE TEST II Maternal and Child Health ............... 162
Answers and Rationale – Care of Clients with
ANSWERS AND RATIONALE – MATERNAL AND CHILD
Physiologic and Psychosocial Alterations ................ 343
HEALTH..................................................................... 167
PART III ......................................................................... 352
MEDICAL SURGICAL NURSING ..................................... 173
PRACTICE TEST I FOUNDATION OF NURSING .............. 352
ANSWERS AND RATIONALE – MEDICAL SURGICAL
NURSING .................................................................. 178 ANSWERS AND RATIONALE – FOUNDATION OF
NURSING .................................................................. 357
PSYCHIATRIC NURSING ................................................ 180
PRACTICE TEST II Maternal and Child Health ............... 361
ANSWERS AND RATIONALE – PSYCHIATRIC NURSING
................................................................................. 185
ANSWERS AND RATIONALE – MATERNAL AND CHILD MEDICAL SURGICAL NURSING Part 1 ........................... 475
HEALTH..................................................................... 366 ANSWERS and RATIONALES for MEDICAL SURGICAL
MEDICAL SURGICAL NURSING ..................................... 372 NURSING Part 1 ........................................................ 479
ANSWERS AND RATIONALE – MEDICAL SURGICAL MEDICAL SURGICAL NURSING Part 2 ........................... 481
NURSING .................................................................. 377 MEDICAL SURGICAL NURSING Part 2 ....................... 485
PSYCHIATRIC NURSING ................................................ 379 ANSWERS and RATIONALES for MEDICAL SURGICAL
ANSWERS AND RATIONALE – PSYCHIATRIC NURSING NURSING Part 2 ........................................................ 489
................................................................................. 384 MEDICAL SURGICAL NURSING Part 3 ........................... 491
FUNDAMENTALS OF NURSING PART 1 ........................ 387 ANSWERS and RATIONALES for MEDICAL SURGICAL
FUNDAMENTALS OF NURSING PART 2 ........................ 392 NURSING Part 3 ........................................................ 495
ANSWERS and RATIONALES for FUNDAMENTALS OF PSYCHIATRIC NURSING Part 1 ...................................... 497
NURSING PART 2 ...................................................... 397 ANSWERS and RATIONALES for PSYCHIATRIC NURSING
FUNDAMENTALS OF NURSING PART 3 ........................ 401 Part 1 ........................................................................ 502
ANSWERS and RATIONALES for FUNDAMENTALS OF PSYCHIATRIC NURSING Part 2 ...................................... 504
NURSING PART 3 ...................................................... 405 ANSWERS and RATIONALES for PSYCHIATRIC NURSING
MATERNITY NURSING Part 1 ........................................ 409 Part 2 ........................................................................ 509
ANSWERS and RATIONALES for MATERNITY NURSING PSYCHIATRIC NURSING Part 3 ...................................... 512
Part 1 ........................................................................ 418 ANSWERS and RATIONALES for PSYCHIATRIC NURSING
MATERNITY NURSING Part 2 ........................................ 428 Part 3 ........................................................................ 516
Answer for maternity part 2 .................................... 433 PROFESSIONAL ADJUSTMENT ...................................... 519
PEDIATRIC NURSING .................................................... 434 LEADERSHIP and MANAGEMENT ................................. 522
ANSWERS and RATIONALES for PEDIATRIC NURSING NURSING RESEARCH Part 1 .......................................... 532
................................................................................. 439 NURSING RESEARCH Part 2 .......................................... 542
COMMUNITY HEALTH NURSING Part 1........................ 444 Nursing Research Suggested Answer Key ................ 546
COMMUNITY HEALTH NURSING Part 2........................ 454

2
3
5. Benner’s “Proficient” nurse level is different
from the other levels in nursing expertise in the
NURSING PRACTICE I: FOUNDATION OF NURSING context of having:
PRACTICE a. the ability to organize and plan activities
b. having attained an advanced level of
SITUATION: Nursing is a profession. The nurse should education
have a background on the theories and foundation of c. a holistic understanding and perception
nursing as it influenced what is nursing today. of the client
d. intuitive and analytic ability in new
1. Nursing is the protection, promotion and situations
optimization of health and abilities, prevention
of illness and injury, alleviation of suffering SITUATION: The nurse has been asked to administer an
through the diagnosis and treatment of human injection via Z TRACK technique. Questions 6 to 10 refer
response and advocacy in the care of the to this.
individuals, families, communities and the
population. This is the most accepted definition 6. The nurse prepares an IM injection for an adult
of nursing as defined by the: client using the Z track technique. 4 ml of
a. PNA medication is to be administered to the client.
b. ANA Which of the following site will you choose?
c. Nightingale a. Deltoid
d. Henderson b. Rectus femoris
c. Ventrogluteal
2. Advancement in Nursing leads to the d. Vastus lateralis
development of the Expanded Career Roles.
Which of the following is NOT an expanded 7. In infants 1 year old and below, which of the
career role for nurses? following is the site of choice for intramuscular
a. Nurse practitioner Injection?
b. Nurse Researcher a. Deltoid
c. Clinical nurse specialist b. Rectus femoris
d. Nurse anaesthesiologist c. Ventrogluteal
d. Vastus lateralis
3. The Board of Nursing regulated the Nursing
profession in the Philippines and is responsible 8. In order to decrease discomfort in Z track
for the maintenance of the quality of nursing in administration, which of the following is
the country. Powers and duties of the board of applicable?
nursing are the following, EXCEPT: a. Pierce the skin quickly and smoothly at
a. Issue, suspend, revoke certificates of a 90 degree angle
registration b. Inject the medication steadily at around
b. Issue subpoena duces tecum, ad 10 minutes per millilitre
testificandum c. Pull back the plunger and aspirate for 1
c. Open and close colleges of nursing minute to make sure that the needle did
d. Supervise and regulate the practice of not hit a blood vessel
nursing d. Pierce the skin slowly and carefully at a
90 degree angle
4. A nursing student or a beginning staff nurse who
has not yet experienced enough real situations 9. After injection using the Z track technique, the
to make judgments about them is in what stage nurse should know that she needs to wait for a
of Nursing Expertise? few seconds before withdrawing the needle and
a. Novice this is to allow the medication to disperse into
b. Newbie the muscle tissue, thus decreasing the client’s
c. Advanced Beginner discomfort. How many seconds should the nurse
d. Competent wait before withdrawing the needle?
a. 2 seconds

4
5

b. 5 seconds that the patient smokes and drinks coffee. When


c. 10 seconds taking the blood pressure of a client who
d. 15 seconds recently smoked or drank coffee, how long
should the nurse wait before taking the client’s
10. The rationale in using the Z track technique in an blood pressure for accurate reading?
intramuscular injection is: a. 15 minutes
a. It decreases the leakage of discolouring b. 30 minutes
and irritating medication into the c. 1 hour
subcutaneous tissues d. 5 minutes
b. It will allow a faster absorption of the
medication 15. While the client has pulse oximeter on his
c. The Z track technique prevent irritation fingertip, you notice that the sunlight is shining
of the muscle on the area where the oximeter is. Your action
d. It is much more convenient for the nurse will be to:
a. Set and turn on the alarm of the
SITUATION: A Client was rushed to the emergency room oximeter
and you are his attending nurse. You are performing a b. Do nothing since there is no identified
vital sign assessment. problem
c. Cover the fingertip sensor with a towel
11. All of the following are correct methods in or bedsheet
assessment of the blood pressure EXCEPT: d. Change the location of the sensor every
a. Take the blood pressure reading on both four hours
arms for comparison
b. Listen to and identify the phases of 16. The nurse finds it necessary to recheck the blood
Korotkoff’s sound pressure reading. In case of such re assessment,
c. Pump the cuff to around 50 mmHg the nurse should wait for a period of:
above the point where the pulse is a. 15 seconds
obliterated b. 1 to 2 minutes
d. Observe procedures for infection control c. 30 minutes
d. 15 minutes
12. You attached a pulse oximeter to the client. You
know that the purpose is to: 17. If the arm is said to be elevated when taking the
a. Determine if the client’s hemoglobin blood pressure, it will create a:
level is low and if he needs blood a. False high reading
transfusion b. False low reading
b. Check level of client’s tissue perfusion c. True false reading
c. Measure the efficacy of the client’s anti- d. Indeterminate
hypertensive medications
d. Detect oxygen saturation of arterial 18. You are to assessed the temperature of the
blood before symptoms of hypoxemia client the next morning and found out that he
develops ate ice cream. How many minutes should you
wait before assessing the client’s oral
13. After a few hours in the Emergency Room, The temperature?
client is admitted to the ward with an order of a. 10 minutes
hourly monitoring of blood pressure. The nurse b. 20 minutes
finds that the cuff is too narrow and this will c. 30 minutes
cause the blood pressure reading to be: d. 15 minutes
a. inconsistent
b. low systolic and high diastolic 19. When auscultating the client’s blood pressure
c. higher than what the reading should be the nurse hears the following: From 150 mmHg
d. lower than what the reading should be to 130 mmHg: Silence, Then: a thumping sound
continuing down to 100 mmHg; muffled sound
14. Through the client’s health history, you gather continuing down to 80 mmHg and then silence.
What is the client’s blood pressure? to lungs. This can be avoided by:
a. 130/80 a. Cleaning teeth and mouth with cotton
b. 150/100 swabs soaked with mouthwash to avoid
c. 100/80 rinsing the buccal cavity
d. 150/100 b. swabbing the inside of the cheeks and
lips, tongue and gums with dry cotton
20. In a client with a previous blood pressure of swabs
130/80 4 hours ago, how long will it take to c. use fingers wrapped with wet cotton
release the blood pressure cuff to obtain an washcloth to rub inside the cheeks,
accurate reading? tongue, lips and ums
a. 10-20 seconds d. suctioning as needed while cleaning the
b. 30-45 seconds buccal cavity
c. 1-1.5 minutes
d. 3-3.5 minutes 25. Your client has difficulty of breathing and is
mouth breathing most of the time. This causes
Situation: Oral care is an important part of hygienic dryness of the mouth with unpleasant odor. Oral
practices and promoting client comfort. hygiene is recommended for the client and in
addition, you will keep the mouth moistened by
21. An elderly client, 84 years old, is unconscious. using:
Assessment of the mouth reveals excessive a. salt solution
dryness and presence of sores. Which of the b. petroleum jelly
following is BEST to use for oral care? c. water
a. lemon glycerine d. mentholated ointment
b. Mineral oil
c. hydrogen peroxide Situation – Ensuring safety before, during and after a
d. Normal saline solution diagnostic procedure is an important responsibility of
the nurse.
22. When performing oral care to an unconscious
client, which of the following is a special 26. To help Fernan better tolerate the
consideration to prevent aspiration of fluids into bronchoscopy, you should instruct him to
the lungs? practice which of the following prior to the
a. Put the client on a sidelying position procedure?
with head of bed lowered a. Clenching his fist every 2 minutes
b. Keep the client dry by placing towel b. Breathing in and out through the nose
under the chin with his mouth open
c. Wash hands and observes appropriate c. Tensing the shoulder muscles while lying
infection control on his back
d. Clean mouth with oral swabs in a careful d. Holding his breath periodically for 30
and an orderly progression seconds

23. The advantages of oral care for a client include 27. Following a bronchoscopy, which of the
all of the following, EXCEPT: following complains to Fernan should be noted
a. decreases bacteria in the mouth and as a possible complication:
teeth a. Nausea and vomiting
b. reduces need to use commercial b. Shortness of breath and laryngeal
mouthwash which irritate the buccal stridor
mucosa c. Blood tinged sputum and coughing
c. improves client’s appearance and self- d. Sore throat and hoarseness
confidence
d. improves appetite and taste of food 28. Immediately after bronchoscopy, you instructed
Fernan to:
24. A possible problem while providing oral care to a. Exercise the neck muscles
unconscious clients is the risk of fluid aspiration b. Refrain from coughing and talking

6
7

c. Breathe deeply d. Weber’s test


d. Clear his throat
34. A nurse is reviewing the arterial blood gas values
29. Thoracentesis may be performed for cytologic of a client and notes that the ph is 7.31, Pco2 is
study of pleural fluid. As a nurse your most 50 mmHg, and the bicarbonate is 27 mEq/L. The
important function during the procedure is to: nurse concludes that which acid base
a. Keep the sterile equipment from disturbance is present in this client?
contamination a. Respiratory acidosis
b. Assist the physician b. Metabolic acidosis
c. Open and close the three-way stopcock c. Respiratory alkalosis
d. Observe the patient’s vital signs d. Metabolic alkalosis

30. Right after thoracentesis, which of the following 35. Allen’s test checks the patency of the:
is most appropriate intervention? a. Ulnar artery
a. Instruct the patient not to cough or deep b. Carotid artery
breathe for two hours c. Radial artery
b. Observe for symptoms of tightness of d. Brachial artery
chest or bleeding
c. Place an ice pack to the puncture site Situation 6: Eileen, 45 years old is admitted to the
d. Remove the dressing to check for hospital with a diagnosis of renal calculi. She is
bleeding experiencing severe flank pain, nauseated and with a
temperature of 39 0C.
Situation: Knowledge of the acid-base disturbance and
the functions of the electrolytes is necessary to 36. Given the above assessment data, the most
determine appropriate intervention and nursing actions. immediate goal of the nurse would be which of
the following?
31. A client with diabetes milletus has a blood a. Prevent urinary complication
glucose level of 644 mg/dL. The nurse interprets b. maintains fluid and electrolytes
that this client is at most risk for the c. Alleviate pain
development of which type of acid-base d. Alleviating nausea
imbalance?
a. Respiratory acidosis 37. After IVP a renal stone was confirmed, a left
b. Respiratory alkalosis nephrectomy was done. Her post-operative
c. Metabolic acidosis order includes “daily urine specimen to be sent
d. Metabolic alkalosis to the laboratory”. Eileen has a foley catheter
attached to a urinary drainage system. How will
32. In a client in the health care clinic, arterial blood you collect the urine specimen?
gas analysis gives the following results: pH 7.48, a. remove urine from drainage tube with
PCO2 32 mmHg, PO2 94 mmHg, HCO3 24 mEq/L. sterile needle and syringe and empty
The nurse interprets that the client has which urine from the syringe into the
acid base disturbance? specimen container
a. Respiratory acidosis b. empty a sample urine from the
b. Metabolic acidosis collecting bag into the specimen
c. Respiratory alkalosis container
d. Metabolic alkalosis c. Disconnect the drainage tube from the
indwelling catheter and allow urine to
33. A client has an order for ABG analysis on radial flow from catheter into the specimen
artery specimens. The nurse ensures that which container.
of the following has been performed or tested d. Disconnect the drainage from the
before the ABG specimens are drawn? collecting bag and allow the urine to
a. Guthrie test flow from the catheter into the
b. Romberg’s test specimen container.
c. Allen’s test
38. Where would the nurse tape Eileen’s indwelling regulation is secreted in the:
catheter in order to reduce urethral irritation? a. Thyroid gland
a. to the patient’s inner thigh b. Parathyroid gland
b. to the patient’ buttocks c. Hypothalamus
c. to the patient’s lower thigh d. Anterior pituitary gland
d. to the patient lower abdomen
45. While Parathormone, a hormone that negates
39. Which of the following menu is appropriate for the effect of calcitonin is secreted by the:
one with low sodium diet? a. Thyroid gland
a. instant noodles, fresh fruits and ice tea b. Parathyroid gland
b. ham and cheese sandwich, fresh fruits c. Hypothalamus
and vegetables d. Anterior pituitary gland
c. white chicken sandwich, vegetable
salad and tea Situation: The staff nurse supervisor requests all the staff
d. canned soup, potato salad, and diet soda nurses to “brainstorm” and learn ways to instruct
diabetic clients on self-administration of insulin. She
40. How will you prevent ascending infection to wants to ensure that there are nurses available daily to
Eileen who has an indwelling catheter? do health education classes.
a. see to it that the drainage tubing
touches the level of the urine 46. The plan of the nurse supervisor is an example of
b. change he catheter every eight hours a. in service education process
c. see to it that the drainage tubing does b. efficient management of human
not touch the level of the urine resources
d. clean catheter may be used since c. increasing human resources
urethral meatus is not a sterile area d. primary prevention

Situation: Hormones are secreted by the various glands 47. When Mrs. Guevarra, a nurse, delegates aspects
in the body. Basic knowledge of the endocrine system is of the clients care to the nurse-aide who is an
necessary. unlicensed staff, Mrs. Guevarra
a. makes the assignment to teach the staff
41. Somatocrinin or the Growth hormone releasing member
hormone is secreted by the: b. is assigning the responsibility to the
a. Hypothalamus aide but not the accountability for
b. Posterior pituitary gland those tasks
c. Anterior pituitary gland c. does not have to supervise or evaluate
d. Thyroid gland the aide
d. most know how to perform task
42. All of the following are secreted by the anterior delegated
pituitary gland except:
a. Somatotropin/Growth hormone 48. Connie, the new nurse, appears tired and
b. Thyroid stimulating hormone sluggish and lacks the enthusiasm she had six
c. Follicle stimulating hormone weeks ago when she started the job. The nurse
d. Gonadotropin hormone releasing supervisor should
hormone a. empathize with the nurse and listen to
her
43. All of the following hormones are hormones b. tell her to take the day off
secreted by the Posterior pituitary gland except: c. discuss how she is adjusting to her new
a. Vasopressin job
b. Anti-diuretic hormone d. ask about her family life
c. Oxytocin
d. Growth hormone 49. Process of formal negotiations of working
conditions between a group of registered nurses
44. Calcitonin, a hormone necessary for calcium and employer is

8
9

a. grievance d. It should disclose previous diagnosis,


b. arbitration prognosis and alternative treatments
c. collective bargaining available for the client
d. strike
55. Delegation is the process of assigning tasks that
50. You are attending a certification on can be performed by a subordinate. The RN
cardiopulmonary resuscitation (CPR) offered and should always be accountable and should not
required by the hospital employing you. This is lose his accountability. Which of the following is
a. professional course towards credits a role included in delegation?
b. in-service education a. The RN must supervise all delegated
c. advance training tasks
d. continuing education b. After a task has been delegated, it is no
longer a responsibility of the RN
Situation: As a nurse, you are aware that proper c. The RN is responsible and accountable
documentation in the patient chart is your responsibility. for the delegated task in adjunct with
the delegate
51. Which of the following is not a legally binding d. Follow up with a delegated task is
document but nevertheless very important in necessary only if the assistive personnel
the care of all patients in any health care is not trustworthy
setting?
a. Bill of rights as provided in the Philippine Situation: When creating your lesson plan for
constitution cerebrovascular disease or STROKE. It is important to
b. Scope of nursing practice as defined by include the risk factors of stroke.
RA 9173
c. Board of nursing resolution adopting the 56. The most important risk factor is:
code of ethics a. Cigarette smoking
d. Patient’s bill of rights b. binge drinking
c. Hypertension
52. A nurse gives a wrong medication to the client. d. heredity
Another nurse employed by the same hospital as
a risk manager will expect to receive which of 57. Part of your lesson plan is to talk about etiology
the following communication? or cause of stroke. The types of stroke based on
a. Incident report cause are the following EXCEPT:
b. Nursing kardex a. Embolic stroke
c. Oral report b. diabetic stroke
d. Complain report c. Hemorrhagic stroke
d. thrombotic stroke
53. Performing a procedure on a client in the
absence of an informed consent can lead to 58. Hemmorhagic stroke occurs suddenly usually
which of the following charges? when the person is active. All are causes of
a. Fraud hemorrhage, EXCEPT:
b. Harassment a. phlebitis
c. Assault and battery b. damage to blood vessel
d. Breach of confidentiality c. trauma
d. aneurysm
54. Which of the following is the essence of
informed consent? 59. The nurse emphasizes that intravenous drug
a. It should have a durable power of abuse carries a high risk of stroke. Which drug is
attorney closely linked to this?
b. It should have coverage from an a. Amphetamines
insurance company b. shabu
c. It should respect the client’s freedom c. Cocaine
from coercion d. Demerol
d. Iron 75 mg/100 ml
60. A participant in the STROKE class asks what is a
risk factor of stroke. Your best response is: 65. Which of the following laboratory test result
a. “More red blood cells thicken blood indicate presence of an infectious process?
and make clots more possible.” a. Erythrocyte sedimentation rate (ESR) 12
b. “Increased RBC count is linked to high mm/hr
cholesterol.” b. White blood cells (WBC) 18,000/mm3
c. “More red blood cell increases c. Iron 90 g/100ml
hemoglobin content.” d. Neutrophils 67%
d. “High RBC count increases blood
pressure.” Situation: Pleural effusion is the accumulation of fluid in
the pleural space. Questions 66 to 70 refer to this.
Situation: Recognition of normal values is vital in
assessment of clients with various disorders. 66. Which of the following is a finding that the nurse
will be able to assess in a client with Pleural
61. A nurse is reviewing the laboratory test results effusion?
for a client with a diagnosis of severe a. Reduced or absent breath sound at the
dehydration. The nurse would expect the base of the lungs, dyspnea, tachpynea
hematocrit level for this client to be which of the and shortness of breath
following? b. Hypoxemia, hypercapnea and
a. 60% respiratory acidosis
b. 47% c. Noisy respiration, crackles, stridor and
c. 45% wheezing
d. 32% d. Tracheal deviation towards the affected
side, increased fremitus and loud breath
62. A nurse is reviewing the electrolyte results of an sounds
assigned client and notes that the potassium
level is 5.6 mEq/L. Which of the following would 67. Thoracentesis is performed to the client with
the nurse expect to note on the ECG as a result effusion. The nurse knows that the removal of
of this laboratory value? fluid should be slow. Rapid removal of fluid in
a. ST depression thoracentesis might cause:
b. Prominent U wave a. Pneumothorax
c. Inverted T wave b. Cardiovascular collapse
d. Tall peaked T waves c. Pleurisy or Pleuritis
d. Hypertension
63. A nurse is reviewing the electrolyte results of an
assigned client and notes that the potassium 68. 3 Days after thoracentesis, the client again
level is 3.2 mEq/L. Which of the following would exhibited respiratory distress. The nurse will
the nurse expect to note on the ECG as a result know that pleural effusion has reoccurred when
of this laboratory value? she noticed a sharp stabbing pain during
a. U waves inspiration. The physician ordered a closed tube
b. Elevated T waves thoracotomy for the client. The nurse knows
c. Absent P waves that the primary function of the chest tube is to:
d. Elevated ST Segment a. Restore positive intrathoracic pressure
b. Restore negative intrathoracic pressure
64. Dorothy underwent diagnostic test and the c. To visualize the intrathoracic content
result of the blood examination are back. On d. As a method of air administration via
reviewing the result the nurse notices which of ventilator
the following as abnormal finding?
a. Neutrophils 60% 69. The chest tube is functioning properly if:
b. White blood cells (WBC) 9000/mm a. There is an oscillation
c. Erythrocyte sedimentation rate (ESR) is b. There is no bubbling in the drainage
39 mm/hr bottle

10
11

c. There is a continuous bubbling in the


waterseal 75. This form of Health Insurance provides
d. The suction control bottle has a comprehensive prepaid health services to
continuous bubbling enrollees for a fixed periodic payment.
a. Health Maintenance Organization
70. In a client with pleural effusion, the nurse is b. Medicare
instructing appropriate breathing technique. c. Philippine Health Insurance Act
Which of the following is included in the d. Hospital Maintenance Organization
teaching?
a. Breath normally Situation: Nursing ethics is an important part of the
b. Hold the breath after each inspiration nursing profession. As the ethical situation arises, so is
for 1 full minute the need to have an accurate and ethical decision
c. Practice abdominal breathing making.
d. Inhale slowly and hold the breath for 3
to 5 seconds after each inhalation 76. The purpose of having a nurses’ code of ethics is:
a. Delineate the scope and areas of nursing
SITUATION: Health care delivery system affects the practice
health status of every filipino. As a Nurse, Knowledge of b. identify nursing action recommended for
this system is expected to ensure quality of life. specific health care situations
c. To help the public understand
71. When should rehabilitation commence? professional conduct expected of
a. The day before discharge nurses
b. When the patient desires d. To define the roles and functions of the
c. Upon admission health care givers, nurses, clients
d. 24 hours after discharge
77. The principles that govern right and proper
72. What exemplified the preventive and promotive conduct of a person regarding life, biology and
programs in the hospital? the health professionals is referred to as:
a. Hospital as a center to prevent and a. Morality
control infection b. Religion
b. Program for smokers c. Values
c. Program for alcoholics and drug addicts d. Bioethics
d. Hospital Wellness Center
78. A subjective feeling about what is right or wrong
73. Which makes nursing dynamic? is said to be:
a. Every patient is a unique physical, a. Morality
emotional, social and spiritual being b. Religion
b. The patient participate in the overall c. Values
nursing care plan d. Bioethics
c. Nursing practice is expanding in the light
of modern developments that takes 79. Values are said to be the enduring believe about
place a worth of a person, ideas and belief. If Values
d. The health status of the patient is are going to be a part of a research, this is
constantly changing and the nurse must categorized under:
be cognizant and responsive to these a. Qualitative
changes b. Experimental
c. Quantitative
74. Prevention is an important responsibility of the d. Non Experimental
nurse in:
a. Hospitals 80. The most important nursing responsibility where
b. Community ethical situations emerge in patient care is to:
c. Workplace a. Act only when advised that the action is
d. All of the above ethically sound
b. Not takes sides, remain neutral and fair
c. Assume that ethical questions are the 85. Based on the Code of Ethics for Filipino Nurses,
responsibility of the health team what is regarded as the hallmark of nursing
d. Be accountable for his or her own responsibility and accountability?
actions a. Human rights of clients, regardless of
creed and gender
81. Why is there an ethical dilemma? b. The privilege of being a registered
a. the choices involved do not appear to be professional nurse
clearly right or wrong c. Health, being a fundamental right of
b. a client’s legal right co-exist with the every individual
nurse’s professional obligation d. Accurate documentation of actions and
c. decisions has to be made based on outcomes
societal norms.
d. decisions has to be mad quickly, often Situation: As a profession, nursing is dynamic and its
under stressful conditions practice is directed by various theoretical models. To
demonstrate caring behaviour, the nurse applies various
82. According to the code of ethics, which of the nursing models in providing quality nursing care.
following is the primary responsibility of the
nurse? 86. When you clean the bedside unit and regularly
a. Assist towards peaceful death attend to the personal hygiene of the patient as
b. Health is a fundamental right well as in washing your hands before and after a
c. Promotion of health, prevention of procedure and in between patients, you indent
illness, alleviation of suffering and to facilitate the body’s reparative processes.
restoration of health Which of the following nursing theory are you
d. Preservation of health at all cost applying in the above nursing action?
a. Hildegard Peplau
83. Which of the following is TRUE about the Code b. Dorothea Orem
of Ethics of Filipino Nurses, except: c. Virginia Henderson
a. The Philippine Nurses Association for d. Florence Nightingale
being the accredited professional
organization was given the privilege to 87. A communication skill is one of the important
formulate a Code of Ethics for Nurses competencies expected of a nurse. Interpersonal
which the Board of Nursing process is viewed as human to human
promulgated relationship. This statement is an application of
b. Code for Nurses was first formulated in whose nursing model?
1982 published in the Proceedings of the a. Joyce Travelbee
Third Annual Convention of the PNA b. Martha Rogers
House of Delegates c. Callista Roy
c. The present code utilized the Code of d. Imogene King
Good Governance for the Professions in
the Philippines 88. The statement “the health status of an individual
d. Certificates of Registration of registered is constantly changing and the nurse must be
nurses may be revoked or suspended for cognizant and responsive to these changes” best
violations of any provisions of the Code explains which of the following facts about
of Ethics. nursing?
a. Dynamic
84. Violation of the code of ethics might equate to b. Client centred
the revocation of the nursing license. Who c. Holistic
revokes the license? d. Art
a. PRC
b. PNA 89. Virginia Henderson professes that the goal of
c. DOH nursing is to work interdependently with other
d. BON health care working in assisting the patient to

12
13

gain independence as quickly as possible. Which include:


of the following nursing actions best a. Prescription of the doctor to the
demonstrates this theory in taking care of a 94 patient’s illness
year old client with dementia who is totally b. Plan of care for patient
immobile? c. Patient’s perception of one’s illness
a. Feeds the patient, brushes his teeth, d. Nursing problem and Nursing diagnosis
gives the sponge bath
b. Supervise the watcher in rendering 94. The medical records that are organized into
patient his morning care separate section from doctors or nurses has
c. Put the patient in semi fowler’s position, more disadvantages than advantages. This is
set the over bed table so the patient can classified as what type of recording?
eat by himself, brush his teeth and a. POMR
sponge himself b. Modified POMR
d. Assist the patient to turn to his sides and c. SOAPIE
allow him to brush and feed himself only d. SOMR
when he feels ready
95. Which of the following is the advantage of SOMR
90. In the self-care deficit theory by Dorothea Orem, or Traditional recording?
nursing care becomes necessary when a patient a. Increases efficiency in data gathering
is unable to fulfil his physiological, psychological b. Reinforces the use of the nursing
and social needs. A pregnant client needing process
prenatal check-up is classified as: c. The caregiver can easily locate proper
a. Wholly compensatory section for making charting entries
b. Supportive Educative d. Enhances effective communication
c. Partially compensatory among health care team members
d. Non compensatory
Situation: June is a 24 year old client with symptoms of
Situation: Documentation and reporting are just as dyspnea, absent breath sounds on the right lung and
important as providing patient care, As such, the nurse chest x ray revealed pleural effusion. The physician will
must be factual and accurate to ensure quality perform thoracentesis.
documentation and reporting.
96. Thoracentesis is useful in treating all of the
91. Health care reports have different purposes. The following pulmonary disorders except:
availability of patients’ record to all health team a. Hemothorax
members demonstrates which of the following b. Hydrothorax
purposes: c. Tuberculosis
a. Legal documentation d. Empyema
b. Research
c. Education 97. Which of the following psychological preparation
d. Vehicle for communication is not relevant for him?
a. Telling him that the gauge of the needle
92. When a nurse commits medication error, she and anesthesia to be used
should accurately document client’s response b. Telling him to keep still during the
and her corresponding action. This is very procedure to facilitate the insertion of
important for which of the following purposes: the needle in the correct place
a. Research c. Allow June to express his feelings and
b. Legal documentation concerns
c. Nursing Audit d. Physician’s explanation on the purpose
d. Vehicle for communication of the procedure and how it will be done

93. POMR has been widely used in many teaching 98. Before thoracentesis, the legal consideration you
hospitals. One of its unique features is SOAPIE must check is:
charting. The P in SOAPIE charting should a. Consent is signed by the client
b. Medicine preparation is correct
c. Position of the client is correct
d. Consent is signed by relative and
physician

99. As a nurse, you know that the position for June


before thoracentesis is:
a. Orthopneic
b. Low fowlers
c. Knee-chest
d. Sidelying position on the affected side

100. Which of the following anaesthetics drug is used


for thoracentesis?
a. Procaine 2%
b. Demerol 75 mg
c. Valium 250 mg
d. Phenobartbital 50 mg

14
15

D. Follicle stimulating hormone

NURSING PRACTICE II 5. The following month, Mariah suspects she is


pregnant. Her urine is positive for Human
Situation: Mariah is a 31 year old lawyer who has been chorionic gonadotrophin. Which structure
married for 6 months. She consults you for guidance in produces Hcg?
relation with her menstrual cycle and her desire to get A. Pituitary gland
pregnant. B. Trophoblastic cells of the embryo
C. Uterine deciduas
1. She wants to know the length of her menstrual D. Ovarian follicles
cycle. Her previous menstrual period is October
22 to 26. Her LMB is November 21. Which of the Situation: Mariah came back and she is now pregnant.
following number of days will be your correct
response? 6. At 5 month gestation, which of the following
A. 29 fetal development would probably be achieve?
B. 28 A. Fetal movement are felt by Mariah
C. 30 B. Vernix caseosa covers the entire body
D. 31 C. Viable if delivered within this period
D. Braxton hicks contractions are observed
2. You advised her to observe and record the signs
of Ovulation. Which of the following signs will 7. The nurse palpates the abdomen of Mariah.
she likely note down? Now At 5 month gestation, What level of the
1. A 1 degree Fahrenheit rise in basal body abdomen can the fundic height be palpated?
temperature A. Symphysis pubis
2. Cervical mucus becomes copious and B. Midpoint between the umbilicus and the
clear xiphoid process
3. One pound increase in weight C. Midpoint between the symphysis pubis
4. Mittelschmerz and the umbilicus
A. 1, 2, 4 D. Umbilicus
B. 1, 2, 3
C. 2, 3, 4 8. She worries about her small breasts, thinking
D. 1, 3, 4 that she probably will not be able to breastfeed
her baby. Which of the following responses of
3. You instruct Mariah to keep record of her basal the nurse is correct?
temperature every day, which of the following A. “The size of your breast will not affect
instructions is incorrect? your lactation”
A. If coitus has occurred; this should be B. “You can switch to bottle feeding”
reflected in the chart C. “You can try to have exercise to increase
B. It is best to have coitus on the evening the size of your breast”
following a drop in BBT to become D. “Manual expression of milk is possible”
pregnant
C. Temperature should be taken 9. She tells the nurse that she does not take milk
immediately after waking and before regularly. She claims that she does not want to
getting out of bed gain too much weight during her pregnancy.
D. BBT is lowest during the secretory Which of the following nursing diagnosis is a
phase priority?
A. Potential self-esteem disturbance
4. She reports an increase in BBT on December 16. related to physiologic changes in
Which hormone brings about this change in her pregnancy
BBT? B. Ineffective individual coping related to
A. Estrogen physiologic changes in pregnancy
B. Gonadotropine C. Fear related to the effects of pregnancy
C. Progesterone D. Knowledge deficit regarding nutritional
requirements of pregnancies related to 15. While talking with Susan, 2 new patients arrived
lack of information sources and they are covered with large towels and the
nurse noticed that there are many cameraman
10. Which of the following interventions will likely and news people outside of the OPD. Upon
ensure compliance of Mariah? assessment the nurse noticed that both of them
A. Incorporate her food preferences that are still nude and the male client’s penis is still
are adequately nutritious in her meal inside the female client’s vagina and the male
plan client said that “I can’t pull it”. Vaginismus was
B. Consistently counsel toward optimum your first impression. You know that The
nutritional intake psychological cause of Vaginismus is related to:
C. Respect her right to reject dietary A. The male client inserted the penis too
information if she chooses deeply that it stimulates vaginal closure
D. Inform her of the adverse effects of B. The penis was too large that is why the
inadequate nutrition to her fetus vagina triggered its defense to attempt
to close it
Situation: Susan is a patient in the clinic where you work. C. The vagina does not want to be
She is inquiring about pregnancy. penetrated
D. It is due to learning patterns of the
11. Susan tells you she is worried because she female client where she views sex as
develops breasts later than most of her friends. bad or sinful
Breast development is termed as:
A. Adrenarche Situation: Overpopulation is one problem in the
B. Thelarche Philippines that causes economic drain. Most Filipinos
C. Mamarche are against in legalizing abortion. As a nurse, Mastery of
D. Menarche contraception is needed to contribute to the society and
economic growth.
12. Kevin, Susan’s husband tells you that he is
considering vasectomy After the birth of their 16. Supposed that Dana, 17 years old, tells you she
new child. Vasectomy involves the incision of wants to use fertility awareness method of
which organ? contraception. How will she determine her
A. The testes fertile days?
B. The epididymis A. She will notice that she feels hot, as if
C. The vas deferens she has an elevated temperature.
D. The scrotum B. She should assess whether her cervical
mucus is thin, copious, clear and
13. On examination, Susan has been found of having watery.
a cystocele. A cystocele is: C. She should monitor her emotions for
A. A sebaceous cyst arising from the vulvar sudden anger or crying
fold D. She should assess whether her breasts
B. Protrusion of intestines into the vagina feel sensitive to cool air
C. Prolapse of the uterus into the vagina
D. Herniation of the bladder into the 17. Dana chooses to use COC as her family planning
vaginal wall method. What is the danger sign of COC you
would ask her to report?
14. Susan typically has menstrual cycle of 34 days. A. A stuffy or runny nose
She told you she had coitus on days 8, 10, 15 and B. Slight weight gain
20 of her menstrual cycle. Which is the day on C. Arthritis like symptoms
which she is most likely to conceive? D. Migraine headache
A. 8th day
B. Day 15 18. Dana asks about subcutaneous implants and she
C. 10th day asks, how long will these implants be effective.
D. Day 20 Your best answer is:
A. One month

16
17

B. Five years
C. Twelve months 23. Another client named Lilia is diagnosed as having
D. 10 years endometriosis. This condition interferes with
fertility because:
19. Dana asks about female condoms. Which of the A. Endometrial implants can block the
following is true with regards to female fallopian tubes
condoms? B. The uterine cervix becomes inflamed
A. The hormone the condom releases and swollen
might cause mild weight gain C. The ovaries stop producing adequate
B. She should insert the condom before estrogen
any penile penetration D. Pressure on the pituitary leads to
C. She should coat the condom with decreased FSH levels
spermicide before use
D. Female condoms, unlike male condoms, 24. Lilia is scheduled to have a
are reusable hysterosalphingogram. Which of the following
instructions would you give her regarding this
20. Dana has asked about GIFT procedure. What procedure?
makes her a good candidate for GIFT? A. She will not be able to conceive for 3
A. She has patent fallopian tubes, so months after the procedure
fertilized ova can be implanted on them B. The sonogram of the uterus will reveal
B. She is RH negative, a necessary any tumors present
stipulation to rule out RH incompatibility C. Many women experience mild bleeding
C. She has normal uterus, so the sperm can as an after effect
be injected through the cervix into it D. She may feel some cramping when the
D. Her husband is taking sildenafil, so all dye is inserted
sperms will be motile
25. Lilia’s cousin on the other hand, knowing nurse
Situation: Nurse Lorena is a Family Planning and Lorena’s specialization asks what artificial
Infertility Nurse Specialist and currently attends to insemination by donor entails. Which would be
FAMILY PLANNING CLIENTS AND INFERTILE COUPLES. your best answer if you were Nurse Lorena?
The following conditions pertain to meeting the nursing A. Donor sperm are introduced vaginally
needs of this particular population group. into the uterus or cervix
B. Donor sperm are injected intra-
21. Dina, 17 years old, asks you how a tubal ligation abdominally into each ovary
prevents pregnancy. Which would be the best C. Artificial sperm are injected vaginally to
answer? test tubal patency
A. Prostaglandins released from the cut D. The husband’s sperm is administered
fallopian tubes can kill sperm intravenously weekly
B. Sperm cannot enter the uterus because
the cervical entrance is blocked. Situation: You are assigned to take care of a group of
C. Sperm can no longer reach the ova, patients across the lifespan.
because the fallopian tubes are blocked
D. The ovary no longer releases ova as 26. Pain in the elder persons requires careful
there is nowhere for them to go. assessment because they:
A. experienced reduce sensory perception
22. The Dators are a couple undergoing testing for B. have increased sensory perception
infertility. Infertility is said to exist when: C. are expected to experience chronic pain
A. A woman has no uterus D. have a decreased pain threshold
B. A woman has no children
C. A couple has been trying to conceive for 27. Administration of analgesics to the older persons
1 year requires careful patient assessment because
D. A couple has wanted a child for 6 older people:
months A. are more sensitive to drugs
B. have increased hepatic, renal and D. Chronic poverty
gastrointestinal function
C. have increased sensory perception 34. Which of the following signs and symptoms
D. mobilize drugs more rapidly would you most likely find when assessing and
infant with Arnold-Chiari malformation?
28. The elderly patient is at higher risk for urinary A. Weakness of the leg muscles, loss of
incontinence because of: sensation in the legs, and restlessness
A. increased glomerular filtration B. Difficulty swallowing, diminished or
B. decreased bladder capacity absent gag reflex, and respiratory
C. diuretic use distress
D. dilated urethra C. Difficulty sleeping, hypervigilant, and an
arching of the back
29. Which of the following is the MOST COMMON D. Paradoxical irritability, diarrhea, and
sign of infection among the elderly? vomiting.
A. decreased breath sounds with crackles
B. pain 35. A parent calls you and frantically reports that her
C. fever child has gotten into her famous ferrous sulfate
D. change in mental status pills and ingested a number of these pills. Her
child is now vomiting, has bloody diarrhea, and is
30. Priorities when caring for the elderly trauma complaining of abdominal pain. You will tell the
patient: mother to:
A. circulation, airway, breathing A. Call emergency medical services (EMS)
B. airway, breathing, disability (neurologic) and get the child to the emergency room
C. disability (neurologic), airway, breathing B. Relax because these symptoms will pass
D. airway, breathing, circulation and the child will be fine
C. Administer syrup of ipecac
31. Preschoolers are able to see things from which D. Call the poison control center
of the following perspectives?
A. Their peers 36. A client says she heard from a friend that you
B. Their own and their mother’s stop having periods once you are on the “pill”.
C. Their own and their caregivers’ The most appropriate response would be:
D. Only their own A. “The pill prevents the uterus from
making such endometrial lining, that is
32. In conflict management, the win-win approach why periods may often be scant or
occurs when: skipped occasionally.”
A. There are two conflicts and the parties B. “If your friend has missed her period,
agree to each one she should stop taking the pills and get a
B. Each party gives in on 50% of the pregnancy test as soon as possible.”
disagreements making up the conflict C. “The pill should cause a normal
C. Both parties involved are committed to menstrual period every month. It
solving the conflict sounds like your friend has not been
D. The conflict is settled out of court so the taking the pills properly.”
legal system and the parties win D. “Missed period can be very dangerous
and may lead to the formation of
33. According to the social-interactional perspective precancerous cells.”
of child abuse and neglect, four factors place the
family members at risk for abuse. These risk 37. The nurse assessing newborn babies and infants
factors are the family members at risk for abuse. during their hospital stay after birth will notice
These risk factors are the family itself, the which of the following symptoms as a primary
caregiver, the child, and manifestation of Hirschsprung’s disease?
A. The presence of a family crisis A. A fine rash over the trunk
B. The national emphasis on sex B. Failure to pass meconium during the
C. Genetics first 24 to 48 hours after birth

18
19

C. The skin turns yellow and then brown release


over the first 48 hours of life B. a woman is less able to keep the
D. High-grade fever laceration clean because of her fatigue
C. healing is limited during pregnancy so
38. A client is 7 months pregnant and has just been these will not heal until after birth
diagnosed as having a partial placenta previa. D. increased bleeding can occur from
She is stable and has minimal spotting and is uterine pressure on leg veins
being sent home. Which of these instructions to
the client may indicate a need for further 43. In working with the caregivers of a client with an
teaching? acute or chronic illness, the nurse would:
A. Maintain bed rest with bathroom A. Teach care daily and let the caregivers
privileges do a return demonstration just before
B. Avoid intercourse for three days. discharge
C. Call if contractions occur. B. Difficulty swallowing, diminished or
D. Stay on left side as much as possible absent gag reflex, and respiratory
when lying down. distress.
C. Difficulty sleeping, hypervigilant, and an
39. A woman has been rushed to the hospital with arching of the back
ruptured membrane. Which of the following D. Paradoxical irritability, diarrhea, and
should the nurse check first? vomiting
A. Check for the presence of infection
B. Assess for Prolapse of the umbilical 44. Which of the following roles BEST exemplifies
cord the expanded role of the nurse?
C. Check the maternal heart rate A. Circulating nurse in surgery
D. Assess the color of the amniotic fluid B. Medication nurse
C. Obstetrical nurse
40. The nurse notes that the infant is wearing a D. Pediatric nurse practitioner
plastic-coated diaper. If a topical medication
were to be prescribed and it were to go on the 45. According to DeRosa and Kochura’s (2006)
stomachs or buttocks, the nurse would teach the article entitled “Implement Culturally Competent
caregivers to: Health Care in your work place,” cultures have
A. avoid covering the area of the topical different patterns of verbal and nonverbal
medication with the diaper communication. Which difference does?
B. avoid the use of clothing on top of the A. NOT necessarily belong?
diaper B. Personal behavior
C. put the diaper on as usual C. Subject matter
D. apply an icepack for 5 minutes to the D. Eye contact
outside of the diaper E. Conversational style

41. Which of the following factors is most important 46. You are the nurse assigned to work with a child
in determining the success of relationships used with acute glomerulonephritis. By following the
in delivering nursing care? prescribed treatment regimen, the child
A. Type of illness of the client experiences a remission. You are now checking
B. Transference and counter transference to make sure the child does not have a relapse.
C. Effective communication Which finding would most lead you to the
D. Personality of the participants conclusion that a relapse is happening?
A. Elevated temperature, cough, sore
42. Grace sustained a laceration on her leg from throat, changing complete blood count
automobile accident. Why are lacerations of (CBC) with diiferential
lower extremities potentially more serious B. A urine dipstick measurement of 2+
among pregnant women than other? proteinuria or more for 3 days, or the
A. lacerations can provoke allergic child found to have 3-4+ proteinutria
responses due to gonadotropic hormone plus edema.
C. The urine dipstick showing glucose in the
urine for 3 days, extreme thirst, increase 51. If a child with diarrhea registers two signs in the
in urine output, and a moon face. yellow row in the IMCI chart, we can classify the
D. A temperature of 37.8 degrees (100 patient as:
degrees F), flank pain, burning A. Moderate dehydration
frequency, urgency on voiding, and B. Severe dehydration
cloudy urine. C. Some dehydration
D. No dehydration
47. The nurse is working with an adolescent who
complains of being lonely and having a lack of 52. Celeste has had diarrhea for 8 days. There is no
fulfillment in her life. This adolescent shies away blood in the stool, he is irritable, his eyes are
from intimate relationships at times yet at other sunken, the nurse offers fluid to Celeste and he
times she appears promiscuous. The nurse will drinks eagerly. When the nurse pinched the
likely work with this adolescent in which of the abdomen it goes back slowly. How will you
following areas? classify Celeste’s illness?
A. Isolation A. Moderate dehydration
B. Lack of fulfillment B. Severe dehydration
C. Loneliness C. Some dehydration
D. Identity D. No dehydration

48. The use of interpersonal decision making, 53. A child who is 7 weeks has had diarrhea for 14
psychomotor skills, and application of days but has no sign of dehydration is classified
knowledge expected in the role of a licensed as:
health care professional in the context of public A. Persistent diarrhea
health welfare and safety is an example of: B. Dysentery
A. Delegation C. Severe dysentery
B. Responsibility D. Severe persistent diarrhea
C. Supervision
D. Competence 54. The child with no dehydration needs home
treatment. Which of the following is not
49. The painful phenomenon known as “back labor” included in the rules for home treatment in this
occurs in a client whose fetus in what position? case?
A. Brow position A. Forced fluids
B. Breech position B. When to return
C. Right Occipito-Anterior Position C. Give vitamin A supplement
D. Left Occipito-Posterior Position D. Feeding more

50. FOCUS methodology stands for: 55. Fever as used in IMCI includes:
A. Focus, Organize, Clarify, Understand A. Axillary temperature of 37.5 or higher
and Solution B. Rectal temperature of 38 or higher
B. Focus, Opportunity, Continuous, Utilize, C. Feeling hot to touch
Substantiate D. All of the above
C. Focus, Organize, Clarify, Understand, E. A and C only
Substantiate
D. Focus, Opportunity, Continuous Situation: Prevention of Dengue is an important nursing
(process), Understand, Solution responsibility and controlling it’s spread is a priority once
outbreak has been observed.
SITUATION: The infant and child mortality rate in the low
to middle income countries is ten times higher than 56. An important role of the community health
industrialized countries. In response to this, the WHO nurse in the prevention and control of Dengue
and UNICEF launched the protocol Integrated H-fever includes:
Management of Childhood Illnesses to reduce the A. Advising the elimination of vectors by
morbidity and mortality against childhood illnesses. keeping water containers covered

20
21

B. Conducting strong health education health worker should first:


drives/campaign directed towards A. Identify the myths and misconceptions
proper garbage disposal prevailing in the community
C. Explaining to the individuals, families, B. Identify the source of these myths and
groups and community the nature of misconceptions
the disease and its causation C. Explain how and why these myths came
D. Practicing residual spraying with about
insecticides D. Select the appropriate IEC strategies to
correct them
57. Community health nurses should be alert in
observing a Dengue suspect. The following is 62. How many percent of measles are prevented by
NOT an indicator for hospitalization of H-fever immunization at 9 months of age?
suspects? A. 80%
A. Marked anorexia, abdominal pain and B. 99%
vomiting C. 90%
B. Increasing hematocrit count D. 95%
C. Cough of 30 days
D. Persistent headache 63. After TT3 vaccination a mother is said to be
protected to tetanus by around:
58. The community health nurses’ primary concern A. 80%
in the immediate control of hemorrhage among B. 99%
patients with dengue is: C. 85%
A. Advising low fiber and non-fat diet D. 90%
B. Providing warmth through light weight
covers 64. If ever convulsions occur after administering
C. Observing closely the patient for vital DPT, what should the nurse best suggest to the
signs leading to shock mother?
D. Keeping the patient at rest A. Do not continue DPT vaccination
anymore
59. Which of these signs may NOT be REGARDED as B. Advise mother to comeback after 1 week
a truly positive signs indicative of Dengue H- C. Give DT instead of DPT
fever? D. Give pertussis of the DPT and remove DT
A. Prolonged bleeding time
B. Appearance of at least 20 petechiae 65. These vaccines are given 3 doses at one month
within 1cm square intervals:
C. Steadily increasing hematocrit count A. DPT, BCG, TT
D. Fall in the platelet count B. OPV, HEP. B, DPT
C. DPT, TT, OPV
60. Which of the following is the most important D. Measles, OPV, DPT
treatment of patients with Dengue H-fever?
A. Give aspirin for fever Situation – With the increasing documented cases of
B. Replacement of body fluids CANCER the best alternative to treatment still remains to
C. Avoid unnecessary movement of patient be PREVENTION. The following conditions apply.
D. Ice cap over the abdomen in case of
melena 66. Which among the following is the primary focus
of prevention of cancer?
Situation: Health education and Health promotion is an A. Elimination of conditions causing cancer
important part of nursing responsibility in the B. Diagnosis and treatment
community. Immunization is a form of health promotion C. Treatment at early stage
that aims at preventing the common childhood illnesses. D. Early detection

61. In correcting misconceptions and myths about 67. In the prevention and control of cancer, which of
certain diseases and their management, the the following activities is the most important
function of the community health nurse? C. 30 breaths per minute or more
A. Conduct community assemblies. D. 60 breaths per minute
B. Referral to cancer specialist those clients
with symptoms of cancer. 73. Nina, the 2nd child has diarrhea for 5 days.
C. Use the nine warning signs of cancer as There is no blood in the stool. She is irritable,
parameters in our process of detection, and her eyes are sunken. The nurse offered
control and treatment modalities. fluids and and the child drinks eagerly. How
D. Teach woman about proper/correct would you classify Nina’s illness?
nutrition. A. Some dehydration
B. Severe dehydration
68. Who among the following are recipients of the C. Dysentery
secondary level of care for cancer cases? D. No dehydration
A. Those under early case detection
B. Those under post case treatment 74. Nina’s treatment should include the following
C. Those scheduled for surgery EXCEPT:
D. Those undergoing treatment A. reassess the child and classify him for
dehydration
69. Who among the following are recipients of the B. for infants under 6 months old who are
tertiary level of care for cancer cases? not breastfed, give 100-200 ml clean
A. Those under early treatment water as well during this period
B. Those under early detection C. Give in the health center the
C. Those under supportive care recommended amount of ORS for 4
D. Those scheduled for surgery hours.
D. Do not give any other foods to the child
70. In Community Health Nursing, despite the for home treatment
availability and use of many equipment and
devices to facilitate the job of the community 75. While on treatment, Nina 18 months old
health nurse, the best tool any nurse should be weighed 18 kgs. and her temperature registered
wel be prepared to apply is a scientific approach. at 37 degrees C. Her mother says she developed
This approach ensures quality of care even at the cough 3 days ago. Nina has no general danger
community setting. This is nursing parlance is signs. She has 45 breaths/minute, no chest in-
nothing less than the: drawing, no stridor. How would you classify
A. nursing diagnosis Nina’s manifestation?
B. nursing research A. No pneumonia
C. nursing protocol B. Pneumonia
D. nursing process C. Severe pneumonia
D. Bronchopneumonia
Situation – Two children were brought to you. One with
chest indrawing and the other had diarrhea. The 76. Carol is 15 months old and weighs 5.5 kgs and it
following questions apply: is her initial visit. Her mother says that Carol is
not eating well and unable to breastfeed, he has
71. Using Integrated Management and Childhood no vomiting, has no convulsion and not
Illness (IMCI) approach, how would you classify abnormally sleepy or difficult to awaken. Her
the 1st child? temperature is 38.9 deg C. Using the integrated
A. Bronchopneumonia management of childhood illness or IMCI
B. Severe pneumonia strategy, if you were the nurse in charge of
C. No pneumonia : cough or cold Carol, how will you classify her illness?
D. Pneumonia A. a child at a general danger sign
B. severe pneumonia
72. The 1st child who is 13 months has fast C. very severe febrile disease
breathing using IMCI parameters he has: D. severe malnutrition
A. 40 breaths per minute or more
B. 50 breaths per minute 77. Why are small for gestational age newborns at

22
23

risk for difficulty maintaining body temperature? B. give the child more fluids
A. their skin is more susceptible to C. continue feeding the child
conduction of cold D. inform when to return to the health
B. they are preterm so are born relatively center
small in size
C. they do not have as many fat stored as 83. Ms. Jordan, RN, believes that a patient should be
other infants treated as individual. This ethical principle that
D. they are more active than usual so they the patient referred to:
throw off comes A. beneficence
B. respect for person
78. Oxytocin is administered to Rita to augment C. nonmaleficence
labor. What are the first symptoms of water D. autonomy
intoxication to observe for during this
procedure? 84. When patients cannot make decisions for
A. headache and vomiting themselves, the nurse advocate relies on the
B. a high choking voice ethical principle of:
C. a swollen tender tongue A. justice and beneficence
D. abdominal bleeding and pain B. beneficence and nonmaleficence
C. fidelity and nonmaleficence
79. Which of the following treatment should NOT be D. fidelity and justice
considered if the child has severe dengue
hemorrhagic fever? 85. Being a community health nurse, you have the
A. use plan C if there is bleeding from the responsibility of participating in protecting the
nose or gums health of people. Consider this situation:
B. give ORS if there is skin Petechiae, Vendors selling bread with their bare hands.
persistent vomiting, and positive They receive money with these hands. You do
tourniquet test not see them washing their hands. What should
C. give aspirin you say/do?
D. prevent low blood sugar A. “Miss, may I get the bread myself
because you have not washed your
80. In assessing the patient’s condition using the hands”
Integrated Management of Childhood Illness B. All of these
approach strategy, the first thing that a nurse C. “Miss, it is better to use a pick up
should do is to: forceps/ bread tong”
D. “Miss, your hands are dirty. Wash your
A. ask what are the child’s problem hands first before getting the bread”
B. check for the four main symptoms
C. check the patient’s level of Situation: The following questions refer to common
consciousness clinical encounters experienced by an entry level nurse.
D. check for the general danger signs
86. A female client asks the nurse about the use of a
81. A child with diarrhea is observed for the cervical cap. Which statement is correct
following EXCEPT: regarding the use of the cervical cap?
A. how long the child has diarrhea A. It may affect Pap smear results.
B. presence of blood in the stool B. It does not need to be fitted by the
C. skin Petechiae physician.
D. signs of dehydration C. It does not require the use of
spermicide.
82. The child with no dehydration needs home D. It must be removed within 24 hours.
treatment. Which of the following is NOT
included in the care for home management at 87. The major components of the communication
this case? process are:
A. give drugs every 4 hours A. Verbal, written and nonverbal
B. Speaker, listener and reply D. Parasites
C. Facial expression, tone of voice and
gestures 93. You are assigned in a private room of Mike.
D. Message, sender, channel, receiver and Which procedure should be of outmost
feedback importance;
A. Alcohol wash
88. The extent of burns in children are normally B. Washing Isolation
assessed and expressed in terms of: C. Universal precaution
A. The amount of body surface that is D. Gloving technique
unburned
B. Percentages of total body surface area 94. What primary health teaching would you give to
(TBSA) mike;
C. How deep the deepest burns are A. Daily exercise
D. The severity of the burns on a 1 to 5 B. reverse isolation
burn scale. C. Prevent infection
D. Proper nutrition
89. The school nurse notices a child who is wearing
old, dirty, poor-fitting clothes; is always hungry; 95. Exercise precaution must be taken to protect
has no lunch money; and is always tired. When health worker dealing with the AIDS patients .
the nurse asks the boy his tiredness, he talks of which among these must be done as priority:
playing outside until midnight. The nurse will A. Boil used syringe and needles
suspect that this child is: B. Use gloves when handling specimen
A. Being raised by a parent of low C. Label personal belonging
intelligence quotient (IQ) D. Avoid accidental wound
B. An orphan
C. A victim of child neglect Situation: Michelle is a 6 year old preschooler. She was
D. The victim of poverty reported by her sister to have measles but she is at
home because of fever, upper respiratory problem and
90. Which of the following indicates the type(s) of white sports in her mouth.
acute renal failure?
A. Four types: hemorrhagic with and 96. Rubeola is an Arabic term meaning Red, the rash
without clotting, and nonhemorrhagic appears on the skin in invasive stage prior to
with and without clottings eruption behind the ears. As a nurse, your
B. One type: acute physical examination must determine
C. Three types: prerenal, intrarenal and complication especially:
postrenal A. Otitis media
D. Two types: acute and subacute B. Inflammatory conjunctiva
C. Bronchial pneumonia
Situation: Mike 16 y/o has been diagnosed to have AIDS; D. Membranous laryngitis
he worked as entertainer in a cruise ship;
97. To render comfort measure is one of the
91. Which method of transmission is common to priorities, Which includes care of the skin, eyes,
contract AIDS? ears, mouth and nose. To clean the mouth, your
A. Syringe and needles antiseptic solution is in some form of which one
B. Sexual contact below?
C. Body fluids A. Water
D. Transfusion B. Alkaline
C. Sulfur
92. Causative organism in AIDS is one of the D. Salt
following;
A. Fungus 98. As a public health nurse, you teach mother and
B. retrovirus family members the prevention of complication
C. Bacteria of measles. Which of the following should be

24
25

closely watched?
A. Temperature fails to drop
B. Inflammation of the nasophraynx
C. Inflammation of the conjunctiva
D. Ulcerative stomatitis

99. Source of infection of measles is secretion of


nose and throat of infection person. Filterable
virus of measles is transmitted by:
A. Water supply
B. Food ingestion
C. Droplet
D. Sexual contact

100. Method of prevention is to avoid


exposure to an infection person. Nursing
responsibility for rehabilitation of patient
includes the provision of:
A. Terminal disinfection
B. Immunization
C. Injection of gamma globulin
D. Comfort measures
c. 50 days
d. 14 days
NURSING PRACTICE III
Situation: As a nurse researcher you must have a very
Situation: Leo lives in the squatter area. He goes to good understanding of the common terms of concept
nearby school. He helps his mother gather molasses used in research.
after school. One day, he was absent because of fever,
malaise, anorexia and abdominal discomfort. 6. The information that an investigator collects
from the subjects or participants in a research
1. Upon assessment, Leo was diagnosed to have study is usually called;
hepatitis A. Which mode of transmission has the a. Hypothesis
infection agent taken? b. Variable
a. Fecal-oral c. Data
b. Droplet d. Concept
c. Airborne
d. Sexual contact 7. Which of the following usually refers to the
independent variables in doing research
2. Which of the following is concurrent disinfection a. Result
in the case of Leo? b. output
a. Investigation of contact c. Cause
b. Sanitary disposal of faeces, urine and d. Effect
blood
c. Quarantine of the sick individual 8. The recipients of experimental treatment is an
d. removing all detachable objects in the experimental design or the individuals to be
room, cleaning lighting and air duct observed in a non experimental design are
surfaces in the ceiling, and cleaning called;
everything downward to the floor a. Setting
b. Treatment
3. Which of the following must be emphasized c. Subjects
during mother’s class to Leo’s mother? d. Sample
a. Administration of Immunoglobulin to
families 9. The device or techniques an investigator
b. Thorough hand washing before and employs to collect data is called;
after eating and toileting a. Sample
c. Use of attenuated vaccines b. hypothesis
d. Boiling of food especially meat c. Instrument
d. Concept
4. Disaster control should be undertaken when
there are 3 or more hepatitis A cases. Which of 10. The use of another person’s ideas or wordings
these measures is a priority? without giving appropriate credit results from
a. Eliminate faecal contamination from inaccurate or incomplete attribution of materials
foods to its sources. Which of the following is referred
b. Mass vaccination of uninfected to when another person’s idea is inappropriate
individuals credited as one’s own;
c. Health promotion and education to a. Plagiarism
families and communities about the b. assumption
disease it’s cause and transmission c. Quotation
d. Mass administration of Immunoglobulin d. Paraphrase

5. What is the average incubation period of Situation – Mrs. Pichay is admitted to your ward. The
Hepatitis A? MD ordered “Prepare for thoracentesis this pm to
a. 30 days remove excess air from the pleural cavity.”
b. 60 days

26
27

11. Which of the following nursing responsibilities is a. Ease the patient to the floor
essential in Mrs. Pichay who will undergo b. Lift the patient and put him on the bed
thoracentesis? c. Insert a padded tongue depressor
a. Support and reassure client during the between his jaws
procedure d. Restraint patient’s body movement
b. Ensure that informed consent has been
signed 17. Mr Santos is scheduled for CT SCAN for the next
c. Determine if client has allergic reaction day, noon time. Which of the following is the
to local anesthesia correct preparation as instructed by the nurse?
d. Ascertain if chest x-rays and other tests a. Shampoo hair thoroughly to remove oil
have been prescribed and completed and dirt
b. No special preparation is needed.
12. Mrs. Pichay who is for thoracentesis is assigned Instruct the patient to keep his head
by the nurse to which of the following positions? still and stead
a. Trendelenburg position c. Give a cleansing enema and give fluids
b. Supine position until 8 AM
c. Dorsal Recumbent position d. Shave scalp and securely attach
d. Orthopneic position electrodes to it

13. During thoracentesis, which of the following 18. Mr Santos is placed on seizure precaution.
nursing intervention will be most crucial? Which of the following would be
a. Place patient in a quiet and cool room contraindicated?
b. Maintain strict aseptic technique a. Obtain his oral temperature
c. Advice patient to sit perfectly still b. Encourage to perform his own personal
during needle insertion until it has been hygiene
withdrawn from the chest c. Allow him to wear his own clothing
d. Apply pressure over the puncture site as d. Encourage him to be out of bed
soon as the needle is withdrawn
19. Usually, how does the patient behave after his
14. To prevent leakage of fluid in the thoracic cavity, seizure has subsided?
how will you position the client after a. Most comfortable walking and moving
thoracentesis? about
a. Place flat in bed b. Becomes restless and agitated
b. Turn on the unaffected side c. Sleeps for a period of time
c. Turn on the affected side d. Say he is thirsty and hungry
d. On bed rest
20. Before, during and after seizure. The nurse
15. Chest x-ray was ordered after thoracentesis. knows that the patient is ALWAYS placed in what
When your client asks what is the reason for position?
another chest x-ray, you will explain: a. Low fowler’s
a. To rule out pneumothorax b. Side lying
b. To rule out any possible perforation c. Modified trendelenburg
c. To decongest d. Supine
d. To rule out any foreign body
Situation: Mrs. Damian an immediate post op
Situation: A computer analyst, Mr. Ricardo J. Santos, 25 cholecystectomy and choledocholithotomy patient,
was brought to the hospital for diagnostic workup after complained of severe pain at the wound site.
he had experienced seizure in his office.
21. Choledocholithotomy is:
16. Just as the nurse was entering the room, the a. The removal of the gallbladder
patient who was sitting on his chair begins to b. The removal of the stones in the
have a seizure. Which of the following must the gallbladder
nurse do first? c. The removal of the stones in the
common bile duct alleviate anxiety
d. The removal of the stones in the kidney c. Avoid overdosing to prevent
dependence/tolerance
22. The simplest pain relieving technique is: d. Monitor VS, more importantly RR
a. Distraction
b. Deep breathing exercise 28. The client complained of abdominal distention
c. Taking aspirin and pain. Your nursing intervention that can
d. Positioning alleviate pain is:
a. Instruct client to go to sleep and relax
23. Which of the following statement on pain is b. Advice the client to close the lips and
TRUE? avoid deep breathing and talking
a. Culture and pain are not associated c. Offer hot and clear soup
b. Pain accompanies acute illness d. Turn to sides frequently and avoid too
c. Patient’s reaction to pain Varies much talking
d. Pain produces the same reaction such as
groaning and moaning 29. Surgical pain might be minimized by which
nursing action in the O.R.
24. In pain assessment, which of the following a. Skill of surgical team and lesser
condition is a more reliable indicator? manipulation
a. Pain rating scale of 1 to 10 b. Appropriate preparation for the
b. Facial expression and gestures scheduled procedure
c. Physiological responses c. Use of modern technology in closing the
d. Patients description of the pain wound
sensation d. Proper positioning and draping of clients

25. When a client complains of pain, your initial 30. Inadequate anesthesia is said to be one of the
response is: common cause of pain both in intra and post op
a. Record the description of pain patients. If General anesthesia is desired, it will
b. Verbally acknowledge the pain involve loss of consciousness. Which of the
c. Refer the complaint to the doctor following are the 2 general types of GA?
d. Change to a more comfortable position a. Epidural and Spinal
b. Subarachnoid block and Intravenous
Situation: You are assigned at the surgical ward and c. Inhalation and Regional
clients have been complaining of post pain at varying d. Intravenous and Inhalation
degrees. Pain as you know, is very subjective.
Situation: Nurse’s attitudes toward the pain influence
26. A one-day postoperative abdominal surgery the way they perceive and interact with clients in pain.
client has been complaining of severe throbbing
abdominal pain described as 9 in a 1-10 pain 31. Nurses should be aware that older adults are at
rating. Your assessment reveals bowel sounds on risk of underrated pain. Nursing assessment and
all quadrants and the dressing is dry and intact. management of pain should address the
What nursing intervention would you take? following beliefs EXCEPT:
a. Medicate client as prescribed a. Older patients seldom tend to report
b. Encourage client to do imagery pain than the younger ones
c. Encourage deep breathing and turning b. Pain is a sign of weakness
d. Call surgeon stat c. Older patients do not believe in
analgesics, they are tolerant
27. Pentoxidone 5 mg IV every 8 hours was d. Complaining of pain will lead to being
prescribed for post abdominal pain. Which will labeled a ‘bad’ patient
be your priority nursing action?
a. Check abdominal dressing for possible 32. Nurses should understand that when a client
swelling responds favorably to a placebo, it is known as
b. Explain the proper use of PCA to the ‘placebo effect’. Placebos do not indicate

28
29

whether or not a client has: acting insulin first


a. Conscience
b. Disease 37. Janevi complains of nausea, vomiting,
c. Real pain diaphoresis and headache. Which of the
d. Drug tolerance following nursing intervention are you going to
carry out first?
33. You are the nurse in the pain clinic where you a. Withhold the client’s next insulin
have client who has difficulty specifying the injection
location of pain. How can you assist such client? b. Test the client’s blood glucose level
a. The pain is vague c. Administer Tylenol as ordered
b. By charting-it hurts all over d. Offer fruit juice, gelatine and chicken
c. Identify the absence and presence of bouillon
pain
d. As the client to point to the painful are 38. Janevi administered regular insulin at 7 A.M and
by just one finger the nurse should instruct Jane to avoid
exercising at around:
34. What symptom, more distressing than pain, a. 9 to 11 A.M
should the nurse monitor when giving opioids b. Between 8 A.M to 9 A.M
especially among elderly clients who are in pain? c. After 8 hours
a. Forgetfulness d. In the afternoon, after taking lunch
b. Drowsiness
c. Constipation 39. Janevi was brought at the emergency room after
d. Allergic reactions like pruritis four month because she fainted in her clinic. The
nurse should monitor which of the following test
35. Physical dependence occurs in anyone who to evaluate the overall therapeutic compliance
takes opiods over a period of time. What do you of a diabetic patient?
tell a mother of a ‘dependent’ when asked for a. Glycosylated hemoglobin
advice? b. Ketone levels
a. Start another drug and slowly lessen the c. Fasting blood glucose
opioid dosage d. Urine glucose level
b. Indulge in recreational outdoor activities
c. Isolate opioid dependent to a restful 40. Upon the assessment of Hba1c of Mrs. Segovia,
resort The nurse has been informed of a 9% Hba1c
d. Instruct slow tapering of the drug result. In this case, she will teach the patient to:
dosage and alleviate physical a. Avoid infection
withdrawal symptoms b. Prevent and recognize hyperglycaemia
c. Take adequate food and nutrition
Situation: The nurse is performing health education d. Prevent and recognize hypoglycaemia
activities for Janevi Segovia, a 30 year old Dentist with
Insulin dependent diabetes Miletus. 41. The nurse is teaching plan of care for Jane with
regards to proper foot care. Which of the
36. Janevi is preparing a mixed dose of insulin. The following should be included in the plan?
nurse is satisfied with her performance when a. Soak feet in hot water
she: b. Avoid using mild soap on the feet
a. Draw insulin from the vial of clear c. Apply a moisturizing lotion to dry feet
insulin first but not between the toes
b. Draw insulin from the vial of the d. Always have a podiatrist to cut your toe
intermediate acting insulin first nails; never cut them yourself
c. Fill both syringes with the prescribed
insulin dosage then shake the bottle 42. Another patient was brought to the emergency
vigorously room in an unresponsive state and a diagnosis of
d. Withdraw the intermediate acting hyperglycaemic hyperosmolar nonketotic
insulin first before withdrawing the short syndrome is made. The nurse immediately
prepares to initiate which of the following of which of the following physiologic changes
anticipated physician’s order? associated with aging.
a. Endotracheal intubation a. Ineffective airway clearance
b. 100 unites of NPH insulin b. Decreased alveolar surfaced area
c. Intravenous infusion of normal saline c. Decreased anterior-posterior chest
d. Intravenous infusion of sodium diameter
bicarbonate d. Hyperventilation

43. Jane eventually developed DKA and is being 47. The older patient is at higher risk for
treated in the emergency room. Which finding incontinence because of:
would the nurse expect to note as confirming a. Dilated urethra
this diagnosis? b. Increased glomerular filtration rate
a. Comatose state c. Diuretic use
b. Decreased urine output d. Decreased bladder capacity
c. Increased respiration and an increase in
pH 48. Merle, age 86, is complaining of dizziness when
d. Elevated blood glucose level and low she stands up. This may indicate:
plasma bicarbonate level a. Dementia
b. Functional decline
44. The nurse teaches Jane to know the difference c. A visual problem
between hypoglycaemia and ketoacidosis. Jane d. Drug toxicity
demonstrates understanding of the teaching by
stating that glucose will be taken if which of the 49. Cardiac ischemia in an older patient usually
following symptoms develops? produces:
a. Polyuria a. ST-T wave changes
b. Shakiness b. Chest pain radiating to the left arm
c. Blurred Vision c. Very high creatinine kinase level
d. Fruity breath odour d. Acute confusion

45. Jane has been scheduled to have a FBS taken in 50. The most dependable sign of infection in the
the morning. The nurse tells Jane not to eat or older patient is:
drink after midnight. Prior to taking the blood a. Change in mental status
specimen, the nurse noticed that Jane is holding b. Fever
a bottle of distilled water. The nurse asked Jane c. Pain
if she drink any, and she said “yes.” Which of the d. Decreased breath sounds with crackles
following is the best nursing action?
a. Administer syrup of ipecac to remove Situation – In the OR, there are safety protocols that
the distilled water from the stomach should be followed. The OR nurse should be well versed
b. Suction the stomach content using NGT with all these to safeguard the safety and quality of
prior to specimen collection patient delivery outcome.
c. Advice to physician to reschedule to
diagnostic examination next day 51. Which of the following should be given highest
d. Continue as usual and have the FBS priority when receiving patient in the OR?
analysis performed and specimen be a. Assess level of consciousness
taken b. Verify patient identification and
informed consent
Situation: Elderly clients usually produce unusual signs c. Assess vital signs
when it comes to different diseases. The ageing process d. Check for jewelry, gown, manicure, and
is a complicated process and the nurse should dentures
understand that it is an inevitable fact and she must be
prepared to care for the growing elderly population. 52. Surgeries like I and D (incision and drainage) and
debridement are relatively short procedures but
46. Hypoxia may occur in the older patients because considered ‘dirty cases’. When are these

30
31

procedures best scheduled? The nurse knows that the temperature and time
a. Last case is set to the optimum level to destroy not only
b. In between cases the microorganism, but also the spores. Which
c. According to availability of of the following is the ideal setting of the
anaesthesiologist autoclave machine?
d. According to the surgeon’s preference a. 10,000 degree Celsius for 1 hour
b. 5,000 degree Celsius for 30 minutes
53. OR nurses should be aware that maintaining the c. 37 degree Celsius for 15 minutes
client’s safety is the overall goal of nursing care d. 121 degree Celsius for 15 minutes
during the intraoperative phase. As the
circulating nurse, you make certain that 58. It is important that before a nurse prepares the
throughout the procedure… material to be sterilized, a chemical indicator
a. the surgeon greets his client before strip should be placed above the package,
induction of anesthesia preferably, Muslin sheet. What is the color of
b. the surgeon and anesthesiologist are in the striped produced after autoclaving?
tandem a. Black
c. strap made of strong non-abrasive b. Blue
materials are fastened securely around c. Gray
the joints of the knees and ankles and d. Purple
around the 2 hands around an arm
board. 59. Chemical indicators communicate that:
d. Client is monitored throughout the a. The items are sterile
surgery by the assistant anesthesiologist b. That the items had undergone
sterilization process but not necessarily
54. Another nursing check that should not be missed sterile
before the induction of general anesthesia is: c. The items are disinfected
a. check for presence underwear d. That the items had undergone
b. check for presence dentures disinfection process but not necessarily
c. check patient’s ID disinfected
d. check baseline vital signs
60. If a nurse will sterilize a heat and moisture labile
55. Some lifetime habits and hobbies affect instruments, It is according to AORN
postoperative respiratory function. If your client recommendation to use which of the following
smokes 3 packs of cigarettes a day for the past method of sterilization?
10 years, you will anticipate increased risk for: a. Ethylene oxide gas
a. perioperative anxiety and stress b. Autoclaving
b. delayed coagulation time c. Flash sterilizer
c. delayed wound healing d. Alcohol immersion
d. postoperative respiratory infection
Situation 5 – Nurses hold a variety of roles when
Situation: Sterilization is the process of removing ALL providing care to a perioperative patient.
living microorganism. To be free of ALL living
microorganism is sterility. 61. Which of the following role would be the
responsibility of the scrub nurse?
56. There are 3 general types of sterilization use in a. Assess the readiness of the client prior
the hospital, which one is not included? to surgery
a. Steam sterilization b. Ensure that the airway is adequate
b. Physical sterilization c. Account for the number of sponges,
c. Chemical sterilization needles, supplies, used during the
d. Sterilization by boiling surgical procedure.
d. Evaluate the type of anesthesia
57. Autoclave or steam under pressure is the most appropriate for the surgical client
common method of sterilization in the hospital.
62. As a perioperative nurse, how can you best meet patients will need surgical amputation but there
the safety need of the client after administering are no sterile surgical equipments. In this case,
preoperative narcotic? which of the following will the nurse expect?
a. Put side rails up and ask the client not a. Equipments needed for surgery need not
to get out of bed be sterilized if this is an emergency
b. Send the client to OR with the family necessitating life saving measures
c. Allow client to get up to go to the b. Forwarding the trauma client to the
comfort room nearest hospital that has available sterile
d. Obtain consent form equipments is appropriate
c. The nurse will need to sterilize the item
63. It is the responsibility of the pre-op nurse to do before using it to the client using the
skin prep for patients undergoing surgery. If hair regular sterilization setting at 121
at the operative site is not shaved, what should degree Celsius in 15 minutes
be done to make suturing easy and lessen d. In such cases, flash sterlizer will be use
chance of incision infection? at 132 degree Celsius in 3 minutes
a. Draped
b. Pulled 68. Tess, the PACU nurse, discovered that Malou,
c. Clipped who weighs 110 lbs prior to surgery, is in severe
d. Shampooed pain 3 hrs after cholecystectomy. Upon checking
the chart, Malou found out that she has an order
64. It is also the nurse’s function to determine when of Demerol 100 mg I.M. prn for pain. Tess should
infection is developing in the surgical incision. verify the order with:
The perioperative nurse should observe for what a. Nurse Supervisor
signs of impending infection? b. Surgeon
a. Localized heat and redness c. Anesthesiologist
b. Serosanguinous exudates and skin d. Intern on duty
blanching
c. Separation of the incision 69. Rosie, 57, who is diabetic is for debridement if
d. Blood clots and scar tissue are visible incision wound. When the circulating nurse
checked the present IV fluid, she found out that
65. Which of the following nursing interventions is there is no insulin incorporated as ordered.
done when examining the incision wound and What should the circulating nurse do?
changing the dressing? a. Double check the doctor’s order and
a. Observe the dressing and type and odor call the attending MD
of drainage if any b. Communicate with the ward nurse to
b. Get patient’s consent verify if insulin was incorporated or not
c. Wash hands c. Communicate with the client to verify if
d. Request the client to expose the incision insulin was incorporated
wound d. Incorporate insulin as ordered.

Situation – The preoperative nurse collaborates with the 70. The documentation of all nursing activities
client significant others, and healthcare providers. performed is legally and professionally vital.
Which of the following should NOT be included
66. To control environmental hazards in the OR, the in the patient’s chart?
nurse collaborates with the following a. Presence of prosthetoid devices such as
departments EXCEPT: dentures, artificial limbs hearing aid, etc.
a. Biomedical division b. Baseline physical, emotional, and
b. Infection control committee psychosocial data
c. Chaplaincy services c. Arguments between nurses and
d. Pathology department residents regarding treatments
d. Observed untoward signs and symptoms
67. An air crash occurred near the hospital leading and interventions including contaminant
to a surge of trauma patient. One of the last intervening factors

32
33

Situation: Basic knowledge on Intravenous solutions is


Situation – Team efforts is best demonstrated in the OR. necessary for care of clients with problems with fluids
and electrolytes.
71. If you are the nurse in charge for scheduling
surgical cases, what important information do 76. A client involved in a motor vehicle crash
you need to ask the surgeon? presents to the emergency department with
a. Who is your internist severe internal bleeding. The client is severely
b. Who is your assistant and hypotensive and unresponsive. The nurse
anaesthesiologist, and what is your anticipates which of the following intravenous
preferred time and type of surgery? solutions will most likely be prescribed to
c. Who are your anaesthesiologist, increase intravascular volume, replace
internist, and assistant immediate blood loss and increase blood
d. Who is your anaesthesiologist pressure?
a. 0.45% sodium chloride
72. In the OR, the nursing tandem for every surgery b. 0.33% sodium chloride
is: c. Normal saline solution
a. Instrument technician and circulating d. Lactated ringer’s solution
nurse
b. Nurse anaesthetist, nurse assistant, and 77. The physician orders the nurse to prepare an
instrument technician isotonic solution. Which of the following IV
c. Scrub nurse and nurse anaesthetist solution would the nurse expect the intern to
d. Scrub and circulating nurses prescribe?
a. 5% dextrose in water
73. While team effort is needed in the OR for b. 0.45% sodium chloride
efficient and quality patient care delivery, we c. 10% dextrose in water
should limit the number of people in the room d. 5% dextrose in 0.9% sodium chloride
for infection control. Who comprise this team?
a. Surgeon, anaesthesiologist, scrub nurse, 78. The nurse is making initial rounds on the nursing
radiologist, orderly unit to assess the condition of assigned clients.
b. Surgeon, assistants, scrub nurse, The nurse notes that the client’s IV Site is cool,
circulating nurse, anaesthesiologist pale and swollen and the solution is not infusing.
c. Surgeon, assistant surgeon, The nurse concludes that which of the following
anaesthesiologist, scrub nurse, complications has been experienced by the
pathologist client?
d. Surgeon, assistant surgeon, a. Infection
anaesthesiologist, intern, scrub nurse b. Phlebitis
c. Infiltration
74. Who usually act as an important part of the OR d. Thrombophelibitis
personnel by getting the wheelchair or stretcher,
and pushing/pulling them towards the operating 79. A nurse reviews the client’s electrolyte
room? laboratory report and notes that the potassium
a. Orderly/clerk level is 3.2 mEq/L. Which of the following would
b. Nurse Supervisor the nurse note on the electrocardiogram as a
c. Circulating Nurse result of the laboratory value?
d. Anaesthesiologist a. U waves
b. Absend P waves
75. The breakdown in teamwork is often times a c. Elevated T waves
failure in: d. Elevated ST segment
a. Electricity
b. Inadequate supply 80. One patient had a ‘runaway’ IV of 50% dextrose.
c. Leg work To prevent temporary excess of insulin or
d. Communication transient hyperinsulin reaction what solution
you prepare in anticipation of the doctor’s
order?
a. Any IV solution available to KVO 86. As an OR nurse, what are your foremost
b. Isotonic solution considerations for selecting chemical agents for
c. Hypertonic solution disinfection?
d. Hypotonic solution a. Material compatibility and efficiency
b. Odor and availability
81. An informed consent is required for: c. Cost and duration of disinfection process
a. closed reduction of a fracture d. Duration of disinfection and efficiency
b. irrigation of the external ear canal
c. insertion of intravenous catheter 87. Before you use a disinfected instrument it is
d. urethral catheterization essential that you:
a. Rinse with tap water followed by alcohol
82. Which of the following is not true with regards b. Wrap the instrument with sterile water
to the informed consent? c. Dry the instrument thoroughly
a. It should describe different treatment d. Rinse with sterile water
alternatives
b. It should contain a thorough and 88. You have a critical heat labile instrument to
detailed explanation of the procedure sterilize and are considering to use high level
to be done disinfectant. What should you do?
c. It should describe the client’s diagnosis a. Cover the soaking vessel to contain the
d. It should give an explanation of the vapor
client’s prognosis b. Double the amount of high level
disinfectant
83. You know that the hallmark of nursing c. Test the potency of the high level
accountability is the: disinfectant
a. accurate documentation and reporting d. Prolong the exposure time according to
b. admitting your mistakes manufacturer’s direction
c. filing an incidence report
d. reporting a medication error 89. To achieve sterilization using disinfectants,
which of the following is used?
84. A nurse is assigned to care for a group of clients. a. Low level disinfectants immersion in 24
On review of the client’s medical records, the hours
nurse determines that which client is at risk for b. Intermediate level disinfectants
excess fluid volume? immersion in 12 hours
a. The client taking diuretics c. High level disinfectants immersion in 1
b. The client with renal failure hour
c. The client with an ileostomy d. High level disinfectant immersion in 10
d. The client who requires gastrointestinal hours
suctioning
90. Bronchoscope, Thermometer, Endoscope, ET
85. A nurse is assigned to care for a group of clients. tube, Cytoscope are all BEST sterilized using
On review of the client’s medical records, the which of the following?
nurse determines that which client is at risk for a. Autoclaving at 121 degree Celsius in 15
deficient fluid volume? minutes
a. A client with colostomy b. Flash sterilizer at 132 degree Celsius in 3
b. A client with congestive heart failure minutes
c. A client with decreased kidney function c. Ethylene Oxide gas aeration for 20 hours
d. A client receiving frequent wound d. 2% Glutaraldehyde immersion for 10
irrigation hours

Situation: As a perioperative nurse, you are aware of the Situation: The OR is divided into three zones to control
correct processing methods for preparing instruments traffic flow and contamination
and other devices for patient use to prevent infection.

34
35

91. What OR attires are worn in the restricted area? process


a. Scrub suit, OR shoes, head cap
b. Head cap, scrub suit, mask, OR shoes 97. 2 organizations endorsed that sterility are
c. Mask, OR shoes, scrub suit affected by factors other than the time itself,
d. Cap, mask, gloves, shoes these are:
a. The PNA and the PRC
92. Nursing intervention for a patient on low dose IV b. AORN and JCAHO
insulin therapy includes the following, EXCEPT: c. ORNAP and MCNAP
a. Elevation of serum ketones to monitor d. MMDA and DILG
ketosis
b. Vital signs including BP 98. All of these factors affect the sterility of the OR
c. Estimate serum potassium equipments, these are the following except:
d. Elevation of blood glucose levels a. The material used for packaging
b. The handling of the materials as well as
93. The doctor ordered to incorporate 1000”u” its transport
insulin to the remaining on-going IV. The c. Storage
strength is 500 /ml. How much should you d. The chemical or process used in
incorporate into the IV solution? sterililzing the material
a. 10 ml
b. 0.5 ml 99. When you say sterile, it means:
c. 2 ml a. The material is clean
d. 5 ml b. The material as well as the equipments
are sterilized and had undergone a
94. Multiple vial-dose-insulin when in use should be rigorous sterilization process
a. Kept at room temperature c. There is a black stripe on the paper
b. Kept in narcotic cabinet indicator
c. Kept in the refrigerator d. The material has no microorganism nor
d. Store in the freezer spores present that might cause an
infection
95. Insulins using insulin syringe are given using how
many degrees of needle insertion? 100. In using liquid sterilizer versus autoclave
a. 45 machine, which of the following is true?
b. 180 a. Autoclave is better in sterilizing OR
c. 90 supplies versus liquid sterilizer
d. 15 b. They are both capable of sterilizing the
equipments, however, it is necessary to
Situation: Maintenance of sterility is an important soak supplies in the liquid sterilizer for
function a nurse should perform in any OR setting. a longer period of time
c. Sharps are sterilized using autoclave and
96. Which of the following is true with regards to not cidex
sterility? d. If liquid sterilizer is used, rinsing it
a. Sterility is time related, items are not before using is not necessary
considered sterile after a period of 30
days of being not use.
b. for 9 months, sterile items are
considered sterile as long as they are
covered with sterile muslin cover and
stored in a dust proof covers.
c. Sterility is event related, not time
related
d. For 3 weeks, items double covered with
muslin are considered sterile as long as
they have undergone the sterilization
d. CT Scan and Incidence report

NURSING PRACTICE IV Situation: An entry level nurse should be able to apply


theoretical knowledge in the performance of the basic
Situation: After an abdominal surgery, the circulating nursing skills.
and scrub nurses have critical responsibility about
sponge and instrument count. 6. A client has an indwelling urinary catheter and
she is suspected of having urinary infection. How
1. Counting is performed thrice: During the should you collect a urine specimen for culture
preincision phase, the operative phase and and sensitivity?
closing phase. Who counts the sponges, needles a. clamp tubing for 60 minutes and insert a
and instruments? sterile needle into the tubing above the
a. The scrub nurse only clamp to aspirate urine
b. The circulating nurse only b. drain urine from the drainage bag into
c. The surgeon and the assistant surgeon the sterile container
d. The scrub nurse and the circulating c. disconnect the tubing from the urinary
nurse catheter and let urine flow into a sterile
container
2. The layer of the abdomen is divided into 5. d. wipe the self-sealing aspiration port
Arrange the following from the first layer going with antiseptic solution and insert a
to the deepest layer: sterile needle into the self-sealing port
1. Fascia
2. Muscle 7. To obtain specimen for sputum culture and
3. Peritoneum sensitivity, which of the following instruction is
4. Subcutaneous/Fat best?
5. Skin a. Upon waking up, cough deeply and
a. 5,4,3,2,1 expectorate into container
b. 5,4,1,3,2 b. Cough after pursed lip breathing
c. 5,4,2,1,3 c. Save sputum for two days in covered
d. 5,4,1,2,3 container
d. After respiratory treatment, expectorate
3. When is the first sponge/instrument count into a container
reported?
a. Before closing the subcutaneous layer 8. The best time for collecting the sputum
b. Before peritoneum is closed specimen for culture and sensitivity is:
c. Before closing the skin a. Before retiring at night
d. Before the fascia is sutured b. Anytime of the day
c. Upon waking up in the morning
4. Like any nursing interventions, counts should be d. Before meals
documented. To whom does the scrub nurse
report any discrepancy of counts so that 9. When suctioning the endotracheal tube, the
immediate and appropriate action is instituted? nurse should:
a. Anaesthesiologists a. Explain procedure to patient; insert
b. Surgeon catheter gently applying suction.
c. OR nurse supervisor Withdrawn using twisting motion
d. Circulating nurse b. Insert catheter until resistance is met,
and then withdraw slightly, applying
5. Which of the following are 2 interventions of the suction intermittently as catheter is
surgical team when an instrument was withdrawn
confirmed missing? c. Hyperoxygenate client insert catheter
a. MRI and Incidence report using back and forth motion
b. CT Scan, MRI, Incidence report d. Insert suction catheter four inches into
c. X-RAY and Incidence report the tube, suction 30 seconds using

36
37

twirling motion as catheter is withdrawn Nursing intervention includes:


a. Bed rest
10. The purpose of NGT IMMEDIATELY after an b. Warm moist soak A transurethral resection
operation is: c. Early ambulation of the prostate (TURP) is
a surgical procedure that
a. For feeding or gavage d. Hot sitz bath involves cutting away a
b. For gastric decompression section of the prostate

c. For lavage, or the cleansing of the Situation – Mang Felix, a 79 year old man who is brought
stomach content to the Surgical Unit from PACU after a transurethral
d. For the rapid return of peristalsis resection. You are assigned to receive him. You noted
that he has a 3-way indwelling urinary catheter for
Situation - Mr. Santos, 50, is to undergo cystoscopy due continuous fast drip bladder irrigation which is
to multiple problems like scantly urination, hematuria connected to a straight drainage.
and dysuria. A cystoscopy is a procedure to look inside the
bladder using a thin camera called a cystoscope. 16. Immediately after surgery, what would you
11. You are the nurse in charge in Mr. Santos. When expect his urine to be?
asked what are the organs to be examined a. Light yellow
during cystoscopy, you will enumerate as b. Bright red
follows: c. Amber
a. Urethra, kidney, bladder, urethra d. Pinkish to red
b. Urethra, bladder wall, trigone, ureteral
opening 17. The purpose of the continuous bladder irrigation
c. Bladder wall, uterine wall, and urethral is to:
opening a. Allow continuous monitoring of the fluid
d. Urethral opening, ureteral opening output status
bladder b. Provide continuous flushing of clots and
debris from the bladder
12. In the OR, you will position Mr. Santos who is c. Allow for proper exchange of
cystoscopy in: electrolytes and fluid
a. Supine d. Ensure accurate monitoring of intake
b. Lithotomy and output
c. Semi-fowler
d. Trendelenburg 18. Mang Felix informs you that he feels some
discomfort on the hypogastric area and he has to
13. After cystoscopy, Mr. Santos asked you to void. What will be your most appropriate action?
explain why there is no incision of any kind. a. Remove his catheter then allow him to
What do you tell him? void on his own
a. “Cystoscopy is direct visualization and b. Irrigate his catheter
examination by urologist”. c. Tell him to “Go ahead and void. You
b. “Cystoscopy is done by x-ray have an indwelling catheter.”
visualization of the urinary tract”. d. Assess color and rate of outflow, if
c. “Cystoscopy is done by using lasers on there is changes refer to urologist for
the urinary tract”. possible irrigation.
d. “Cystoscopy is an endoscopic procedure
of the urinary tract”. 19. You decided to check on Mang Felix’s IV fluid
infusion. You noted a change in flow rate, pallor
14. Within 24-48 hours post cystoscopy, it is normal and coldness around the insertion site. What is
to observe one the following: your assessment finding?
a. Pink-tinged urine a. Phlebitis
b. Distended bladder b. Infiltration to subcutaneous tissue
c. Signs of infection c. Pyrogenic reaction
d. Prolonged hematuria d. Air embolism

15. Leg cramps are NOT uncommon post cystoscopy. 20. Knowing that proper documentation of
assessment findings and interventions are d. Fever, Irritability and a large output of
important responsibilities of the nurse during diluted urine
first post-operative day, which of the following is
the LEAST relevant to document in the case of 25. What kind of renal failure will melamine
Mang Felix? poisoning cause?
a. Chest pain and vital signs a. Chronic, Prerenal
b. Intravenous infusion rate b. Chronic, Intrarenal
c. Amount, color, and consistency of c. Acute, Postrenal
bladder irrigation drainage d. Acute, Prerenal
d. Activities of daily living started
Situation: Leukemia is the most common type of
Situation: Melamine contamination in milk has brought childhood cancer. Acute Lymphoid Leukemia is the cause
worldwide crisis both in the milk production sector as of almost 1/3 of all cancer that occurs in children under
well as the health and economy. Being aware of the age 15.
current events is one quality that a nurse should possess
to prove that nursing is a dynamic profession that will 26. The survival rate for Acute Lymphoid Leukemia is
adapt depending on the patient’s needs. approximately:
a. 25%
21. Melamine is a synthetic resin used for b. 40%
whiteboards, hard plastics and jewellery box c. 75%
covers due to its fire retardant properties. Milk d. 95%
and food manufacturers add melamine in order
to: 27. Whereas acute nonlymphoid leukemia has a
a. It has a bacteriostatic property leading survival rate of:
to increase food and milk life as a way of a. 25%
preserving the foods b. 40%
b. Gives a glazy and more edible look on c. 75%
foods d. 95%
c. Make milks more tasty and creamy
d. Create an illusion of a high protein 28. The three main consequence of leukemia that
content on their products cause the most danger is:
a. Neutropenia causing infection, anemia
22. Most of the milks contaminated by Melamine causing impaired oxygenation and
came from which country? thrombocytopenia leading to bleeding
a. India tendencies
b. China b. Central nervous system infiltration,
c. Philippines anemia causing impaired oxygenation
d. Korea and thrombocytopenia leading to
bleeding tendencies
23. Which government agency is responsible for c. Splenomegaly, hepatomegaly, fractures
testing the melamine content of foods and food d. Invasion by the leukemic cells to the
products? bone causing severe bone pain
a. DOH
b. MMDA 29. Gold standard in the diagnosis of leukemia is by
c. NBI which of the following?
d. BFAD a. Blood culture and sensitivity
b. Bone marrow biopsy
24. Infants are the most vulnerable to melamine c. Blood biopsy
poisoning. Which of the following is NOT a sign d. CSF aspiration and examination
of melamine poisoning?
a. Irritability, Back ache, Urolithiasis 30. Adriamycin,Vincristine,Prednisone and L
b. High blood pressure, fever asparaginase are given to the client for long
c. Anuria, Oliguria or Hematuria term therapy. One common side effect,

38
39

especially of adriamycin is alopecia. The child sensitivity of the breast.


asks: “Will I get my hair back once again?” The
nurse best respond is by saying: 34. Carmen, who is asking the nurse the most
a. “Don’t be silly, ofcourse you will get your appropriate time of the month to do her self-
hair back” examination of the breast. The MOST
b. “We are not sure, let’s hope it’ll grow” appropriate reply by the nurse would be:
c. “This side effect is usually permanent, a. the 26th day of the menstrual cycle
But I will get the doctor to discuss it for b. 7 to 8 days after conclusion of the
you” menstrual period
d. “Your hair will regrow in 3 to 6 months c. during her menstruation
but of different color, usually darker d. the same day each month
and of different texture”
35. Carmen being treated with radiation therapy.
Situation: Breast Cancer is the 2nd most common type of What should be included in the plan of care to
cancer after lung cancer and 99% of which, occurs in minimize skin damage from the radiation
woman. Survival rate is 98% if this is detected early and therapy?
treated promptly. Carmen is a 53 year old patient in the a. Cover the areas with thick clothing
high risk group for breast cancer was recently diagnosed materials
with Breast cancer. b. Apply a heating pad to the site
c. Wash skin with water after the therapy
31. All of the following are factors that said to d. Avoid applying creams and powders to
contribute to the development of breast cancer the area
except:
a. Prolonged exposure to estrogen such as 36. Based on the DOH and World Health
an early menarche or late menopause, Organization (WHO) guidelines, the mainstay for
nulliparity and childbirth after age 30 early detection method for breast cancer that is
b. Genetics recommended for developing countries is:
c. Increasing Age a. a monthly breast self-examination (BSE)
d. Prolonged intake of Tamoxifen and an annual health worker breast
(Nolvadex) examination (HWBE)
b. an annual hormone receptor assay
32. Protective factors for the development of breast c. an annual mammogram
cancer includes which of the following except: d. a physician conduct a breast clinical
a. Exercise examination every 2 years
b. Breast feeding
c. Prophylactic Tamoxifen 37. The purpose of performing the breast self-
d. Alcohol intake examination (BSE) regularly is to discover:
a. fibrocystic masses
33. A patient diagnosed with breast cancer has been b. areas of thickness or fullness
offered the treatment choices of breast c. cancerous lumps
conservation surgery with radiation or a d. changes from previous BSE
modified radical mastectomy. When questioned
by the patient about these options, the nurse 38. If you are to instruct a postmenopausal woman
informs the patient that the lumpectomy with about BSE, when would you tell her to do BSE:
radiation: a. on the same day of each month
a. reduces the fear and anxiety that b. on the first day of her menstruation
accompany the diagnosis and treatment c. right after the menstrual period
of cancer d. on the last day of her menstruation
b. has about the same 10-year survival rate
as the modified radical mastectomy 39. During breast self-examination, the purpose of
c. provides a shorter treatment period with standing in front of the mirror it to observe the
a fewer long term complications breast for:
d. preserves the normal appearance and a. thickening of the tissue
b. lumps in the breast tissue d. Ineffective tissue perfusion, peripheral,
c. axillary lymphnodes cerebral, cardiovascular,
d. change in size and contour gastrointestinal, renal

40. When preparing to examine the left breast in a 45. What intervention should you include in your
reclining position, the purpose of placing a small care plan?
folded towel under the client’s left shoulder is a. Inspect his skin for petechiae, bruising,
to: GI bleeding regularly
a. bring the breast closer to the examiner’s b. Place Albert on strict isolation
right hand precaution
b. tense the pectoral muscle c. Provide rest in between activities
c. balance the breast tissue more evenly d. Administer antipyretics if his
on the chest wall temperature exceeds 38C
d. facilitate lateral positioning of the breast
Situation: Burn are cause by transfer of heat source to
Situation – Radiation therapy is another modality of the body. It can be thermal, electrical, radiation or
cancer management. With emphasis on multidisciplinary chemical.
management you have important responsibilities as
nurse. 46. A burn characterized by Pale white appearance,
charred or with fat exposed and painlessness is:
41. Albert is receiving external radiation therapy and a. Superficial partial thickness burn
he complains of fatigue and malaise. Which of b. Deep partial thickness burn
the following nursing interventions would be c. Full thickness burn
most helpful for Albert? d. Deep full thickness burn
a. Tell him that sometimes these feelings
can be psychogenic 47. Which of the following BEST describes superficial
b. Refer him to the physician partial thickness burn or first degree burn?
c. Reassure him that these feelings are a. Structures beneath the skin are damage
normal b. Dermis is partially damaged
d. Help him plan his activities c. Epidermis and dermis are both damaged
d. Epidermis is damaged
42. Immediately following the radiation teletherapy,
Albert is 48. A burn that is said to be “WEEPING” is classified
a. Considered radioactive for 24 hrs as:
b. Given a complete bath a. Superficial partial thickness burn
c. Placed on isolation for 6 hours b. Deep partial thickness burn
d. Free from radiation c. Full thickness burn
d. Deep full thickness burn
43. Albert is admitted with a radiation induced
thrombocytopenia. As a nurse you should 49. During the Acute phase of the burn injury, which
observe the following symptoms: of the following is a priority?
a. Petechiae, ecchymosis, epistaxis a. wound healing
b. Weakness, easy fatigability, pallor b. emotional support
c. Headache, dizziness, blurred vision c. reconstructive surgery
d. Severe sore throat, bacteremia, d. fluid resuscitation
hepatomegaly
50. While in the emergent phase, the nurse knows
44. What nursing diagnosis should be of highest that the priority is to:
priority? a. Prevent infection
a. Knowledge deficit regarding b. Prevent deformities and contractures
thrombocytopenia precautions c. Control pain
b. Activity intolerance d. Return the hemodynamic stability via
c. Impaired tissue integrity fluid resuscitation

40
41

the client is developing:


51. The MOST effective method of delivering pain a. Cerebral hypoxia
medication during the emergent phase is: b. metabolic acidosis
a. intramuscularly c. Hypervolemia
b. orally d. Renal failure
c. subcutaneously
d. intravenously 58. A 165 lbs trauma client was rushed to the
emergency room with full thickness burns on the
52. When a client accidentally splashes chemicals to whole face, right and left arm, and at the
his eyes, The initial priority care following the anterior upper chest sparing the abdominal area.
chemical burn is to: He also has superficial partial thickness burn at
a. irrigate with normal saline for 1 to 15 the posterior trunk and at the half upper portion
minutes of the left leg. He is at the emergent phase of
b. transport to a physician immediately burn. Using the parkland’s formula, you know
c. irrigate with water for 15 minutes or that during the first 8 hours of burn, the amount
longer of fluid will be given is:
d. cover the eyes with a sterile gauze a. 5,400 ml
b. 9, 450 ml
53. Which of the following can be a fatal c. 10,800 ml
complication of upper airway burns? d. 6,750 ml
a. stress ulcers
b. shock 59. The doctor incorporated insulin on the client’s
c. hemorrhage fluid during the emergent phase. The nurse
d. laryngeal spasms and swelling knows that insulin is given because:
a. Clients with burn also develops
54. When a client will rush towards you and he has a Metabolic acidosis
burning clothes on, It is your priority to do which b. Clients with burn also develops
of the following first? hyperglycemia
a. log roll on the grass/ground c. Insulin is needed for additional energy
b. slap the flames with his hands and glucose burning after the stressful
c. Try to remove the burning clothes incidence to hasten wound healing,
d. Splash the client with 1 bucket of cool regain of consciousness and rapid return
water of hemodynamic stability
d. For hyperkalemia
55. Once the flames are extinguished, it is most
important to: 60. The IV fluid of choice for burn as well as
a. cover clientwith a warm blanket dehydration is:
b. give him sips of water a. 0.45% NaCl
c. calculate the extent of his burns b. Sterile water
d. assess the Sergio’s breathing c. NSS
d. D5LR
56. During the first 24 hours after the thermal injury,
you should asses Sergio for: Situation: ENTEROSTOMAL THERAPY is now considered a
a. hypokalemia and hypernatremia specialty in nursing. You are participating in the OSTOMY
b. hypokalemia and hyponatremia CARE CLASS.
c. hyperkalemia and hyponatremia
d. hyperkalemia and hypernatremia 61. You plan to teach Fermin how to irrigate the
colostomy when:
57. A client who sustained deep partial thickness a. The perineal wound heals And Fermin
and full thickness burns of the face, whole can sit comfortably on the commode
anterior chest and both upper extremities two b. Fermin can lie on the side comfortably,
days ago begins to exhibit extreme restlessness. about the 3rd postoperative day
You recognize that this most likely indicates that c. The abdominal incision is closed and
contamination is no longer a danger it is important for nurses to gather as much information
d. The stools starts to become formed, to be able to address their needs for nursing care.
around the 7th postoperative day
66. Critically ill patients frequently complain about
62. When preparing to teach Fermin how to irrigate which of the following when hospitalized?
colostomy, you should plan to do the procedure: a. Hospital food
a. When Fermin would have normal bowel b. Lack of privacy
movement c. Lack of blankets
b. At least 2 hours before visiting hours d. Inadequate nursing staff
c. Prior to breakfast and morning care
d. After Fermin accepts alteration in body 67. Who of the following is at greatest risk of
image developing sensory problem?
a. Female patient
63. When observing a return demonstration of a b. Transplant patient
colostomy irrigation, you know that more c. Adoloscent
teaching is required if Fermin: d. Unresponsive patient
a. Lubricates the tip of the catheter prior to
inserting into the stoma 68. Which of the following factors may inhibit
b. Hangs the irrigating bag on the learning in critically ill patients?
bathroom door cloth hook during fluid a. Gender
insertion b. Educational level
c. Discontinues the insertion of fluid after c. Medication
500 ml of fluid has been instilled d. Previous knowledge of illness
d. Clamps of the flow of fluid when felling
uncomfortable 69. Which of the following statements does not
apply to critically ill patients?
64. You are aware that teaching about colostomy a. Majority need extensive rehabilitation
care is understood when Fermin states, “I will b. All have been hospitalized previously
contact my physician and report: c. Are physically unstable
a. If I have any difficulty inserting the d. Most have chronic illness
irrigating tub into the stoma.”
b. If I noticed a loss of sensation to touch in 70. Families of critically ill patients desire which of
the stoma tissue.” the following needs to be met first by the nurse?
c. The expulsion of flatus while the a. Provision of comfortable space
irrigating fluid is running out.” b. Emotional support
d. When mucus is passed from the stoma c. Updated information on client’s status
between the irrigations.” d. Spiritual counselling

65. You would know after teaching Fermin that Situation: Johnny, sought consultation to the hospital
dietary instruction for him is effective when he because of fatigability, irritability, jittery and he has been
states, “It is important that I eat: experiencing this sign and symptoms for the past 5
a. Soft food that is easily digested and months.
absorbed by my large intestines.”
b. Bland food so that my intestines do not 71. His diagnosis was hyperthyroidism, the following
become irritated.” are expected symptoms except:
c. Food low in fiber so that there are fewer a. Anorexia
stools.” b. Fine tremors of the hand
d. Everything that I ate before the c. Palpitation
operation, while avoiding foods that d. Hyper alertness
cause gas”.
72. She has to take drugs to treat her
Situation: Based on studies of nurses working in special hyperthyroidism. Which of the following will you
units like the intensive care unit and coronary care unit, NOT expect that the doctor will prescribe?

42
43

a. Colace (Docusate) to:


b. Tapazole (Methimazole) a. Decrease the vascularity and size of the
c. Cytomel (Liothyronine) thyroid gland
d. Synthroid (Levothyroxine) b. Decrease the size of the thyroid gland
only
73. The nurse knows that Tapazole has which of the c. Increase the vascularity and size of the
following side effect that will warrant immediate thyroid gland
withholding of the medication? d. Increase the size of the thyroid gland
a. Death only
b. Hyperthermia
c. Sore throat 79. Which of the following is a side effect of Lugol’s
d. Thrombocytosis solution?
a. Hypokalemia
74. You asked questions as soon as she regained b. Enlargement of the Thryoid gland
consciousness from thyroidectomy primarily to c. Nystagmus
assess the evidence of: d. Excessive salivation
a. Thyroid storm
b. Damage to the laryngeal nerve 80. In administering Lugol’s solution, the
c. Mediastinal shift precautionary measure should include:
d. Hypocalcaemia tetany a. Administer with glass only
b. Dilute with juice and administer with a
75. Should you check for haemorrhage, you will: straw
a. Slip your hand under the nape of her c. Administer it with milk and drink it
neck d. Follow it with milk of magnesia
b. Check for hypotension
c. Apply neck collar to prevent Situation: Pharmacological treatment was not effective
haemorrhage for Johnny’s hyperthyroidism and now, he is scheduled
d. Observe the dressing if it is soaked with for Thyroidectomy.
blood
81. Instruments in the surgical suite for surgery is
76. Basal Metabolic rate is assessed on Johnny to classified as either CRITICAL, SEMI CRITICAL and
determine his metabolic rate. In assessing the NON CRITICAL. If the instrument are introduced
BMR using the standard procedure, you need to directly into the blood stream or into any
tell Johnny that: normally sterile cavity or area of the body it is
a. Obstructing his vision classified as:
b. Restraining his upper and lower a. Critical
extremities b. Non Critical
c. Obstructing his hearing c. Semi Critical
d. Obstructing his nostrils with a clamp d. Ultra Critical

77. The BMR is based on the measurement that: 82. Instruments that do not touch the patient or
a. Rate of respiration under different have contact only to intact skin is classified as:
condition of activities and rest a. Critical
b. Amount of oxygen consumption under b. Non Critical
resting condition over a measured c. Semi Critical
period of time d. Ultra Critical
c. Amount of oxygen consumption under
stressed condition over a measured 83. If an instrument is classified as Semi Critical, an
period of time acceptable method of making the instrument
d. Ratio of respiration to pulse rate over a ready for surgery is through:
measured period of time a. Sterilization
b. Disinfection
78. Her physician ordered lugol’s solution in order c. Decontamination
d. Cleaning d. Tetany

84. While critical items and should be: 90. After surgery Johnny develops peripheral
a. Clean numbness, tingling and muscle twitching and
b. Sterilized spasm. What would you anticipate to
c. Decontaminated administer?
d. Disinfected a. Magnesium sulfate
b. Potassium iodide
85. As a nurse, you know that intact skin acts as an c. Calcium gluconate
effective barrier to most microorganisms. d. Potassium chloride
Therefore, items that come in contact with the
intact skin or mucus membranes should be: Situation: Budgeting is an important part of a nurse
a. Disinfected managerial activity. The correct allocation and
b. Clean distribution of resources is vital in the harmonious
c. Sterile operation of the financial balance of the agency.
d. Alcoholized
91. Which of the following best defines Budget?
86. You are caring for Johnny who is scheduled to a. Plan for the allocation of resources for
undergo total thyroidectomy because of a future use
diagnosis of thyroid cancer. Prior to total b. The process of allocating resources for
thyroidectomy, you should instruct Johnny to: future use
a. Perform range and motion exercise on c. Estimate cost of expenses
the head and neck d. Continuous process in seeing that the
b. Apply gentle pressure against the goals and objective of the agency is met
incision when swallowing
c. Cough and deep breathe every 2 hours 92. Which of the following best defines Capital
d. Support head with the hands when Budget?
changing position a. Budget to estimate the cost of direct
labour, number of staff to be hired and
87. As Johnny’s nurse, you plan to set up emergency necessary number of workers to meet
equipment at her bedside following the general patient needs
thyroidectomy. You should include: b. Includes the monthly and daily expenses
a. An airway and rebreathing tube and expected revenue and expenses
b. A tracheostomy set and oxygen c. These are related to long term planning
c. A crush cart with bed board and includes major replacement or
d. Two ampules of sodium bicarbonate expansion of the plant, major
equipment and inventories.
88. Which of the following nursing interventions is d. These are expenses that are not
appropriate after a total thyroidectomy? dependent on the level of production or
a. Place pillows under your patient’s sales. They tend to be time-related, such
shoulders. as salaries or rents being paid per month
b. Raise the knee-gatch to 30 degrees
c. Keep you patient in a high-fowler’s 93. Which of the following best described
position. Operational Budget?
d. Support the patient’s head and neck a. Budget to estimate the cost of direct
with pillows and sandbags. labour, number of staff to be hired and
necessary number of workers to meet
89. If there is an accidental injury to the parathyroid the general patient needs
gland during a thyroidectomy which of the b. Includes the monthly and daily
following might Leda develops postoperatively? expenses and expected revenue and
a. Cardiac arrest expenses
b. Respiratory failure c. These are related to long term planning
c. Dyspnea and includes major replacement or

44
45

expansion of the plant, major c. The Chinese Triad


equipments and inventories. d. Charcot’s Triad
d. These are expenses that are not
dependent on the level of production or 98. Which of the following is true with the Triad
sales. They tend to be time-related, such seen in head injuries?
as rent a. Narrowing of Pulse pressure, Cheyne
stokes respiration, Tachycardia
94. Which of the following accurately describes a b. Widening Pulse pressure, Irregular
Fixed Cost in budgeting? respiration, Bradycardia
a. These are usually the raw materials and c. Hypertension, Kussmaul’s respiration,
labour salaries that depend on the Tachycardia
production or sales d. Hypotension, Irregular respiration,
b. These are expenses that change in Bradycardia
proportion to the activity of a business
c. These are expenses that are not 99. In a client with a Cheyne stokes respiration,
dependent on the level of production or which of the following is the most appropriate
sales. They tend to be time-related, nursing diagnosis?
such as rent a. Ineffective airway clearance
d. This is the summation of the Variable b. Impaired gas exchange
Cost and the Fixed Cost c. Ineffective breathing pattern
d. Activity intolerance
95. Which of the following accurately describes
Variable Cost in budgeting? 100. You know the apnea is seen in client’s with
a. These are related to long term planning cheyne stokes respiration, APNEA is defined as:
and include major replacement or a. Inability to breathe in a supine position
expansion of the plant, major so the patient sits up in bed to breathe
equipments and inventories. b. The patient is dead, the breathing stops
b. These are expenses that change in c. There is an absence of breathing for a
proportion to the activity of a business period of time, usually 15 seconds or
c. These are expenses that are not more
dependent on the level of production or d. A period of hypercapnea and hypoxia
sales. They tend to be time-related, such due to the cessation of respiratory effort
as rent inspite of normal respiratory functioning
d. This is the summation of the Variable
Cost and the Fixed Cost

Situation – Andrea is admitted to the ER following an


assault where she was hit in the face and head. She was
brought to the ER by a police woman. Emergency
measures were started.

96. Andrea’s respiration is described as waxing and


waning. You know that this rhythm of respiration
is defined as:
a. Biot’s
b. Cheyne stokes
c. Kussmaul’s
d. Eupnea

97. What do you call the triad of sign and symptoms


seen in a client with increasing ICP?
a. Virchow’s Triad
b. Cushing’s Triad
5. A Client state, “I get down on myself when I
NURSING PRACTICE V make mistake.” Using Cognitive therapy
approach, the nurse should:
Situation: Understanding different models of care is a A. Teach the client relaxation exercise to
necessary part of the nurse patient relationship. diminish stress
B. Provide the client with Mastery
1. The focus of this therapy is to have a positive experience to boost self esteem
environmental manipulation, physical and social C. Explore the client’s past experiences that
to effect a positive change. causes the illness
A. Milieu D. Help client modify the belief that
B. Psychotherapy anything less than perfect is horrible
C. Behaviour
D. Group 6. The most advantageous therapy for a preschool
age child with a history of physical and sexual
2. The client asks the nurse about Milieu therapy. abuse would be:
The nurse responds knowing that the primary A. Play
focus of milieu therapy can be best described by B. Psychoanalysis
which of the following? C. Group
A. A form of behavior modification therapy D. Family
B. A cognitive approach of changing the
behaviour 7. An 18 year old client is admitted with the
C. A living, learning or working diagnosis of anorexia nervosa. A cognitive
environment behavioural approach is used as part of her
D. A behavioural approach to changing treatment plan. The nurse understands that the
behaviour purpose of this approach is to:
A. Help the client identify and examine
3. A nurse is caring for a client with phobia who is dysfunctional thoughts and beliefs
being treated for the condition. The client is B. Emphasize social interaction with clients
introduced to short periods of exposure to the who withdraw
phobic object while in relaxed state. The nurse C. Provide a supportive environment and a
understands that this form of behaviour therapeutic community
modification can be best described as: D. Examine intrapsychic conflicts and past
A. Systematic desensitization events in life
B. Self-control therapy
C. Aversion Therapy 8. The nurse is preparing to provide reminiscence
D. Operant conditioning therapy for a group of clients. Which of the
following clients will the nurse select for this
4. A client with major depression is considering group?
cognitive therapy. The client say to the nurse, A. A client who experiences profound
“How does this treatment works?” The nurse depression with moderate cognitive
responds by telling the client that: impairment
A. “This type of treatment helps you B. A catatonic, immobile client with
examine how your thoughts and moderate cognitive impairment
feelings contribute to your difficulties” C. An undifferentiated schizophrenic client
B. “This type of treatment helps you with moderate cognitive impairment
examine how your past life has D. A client with mild depression who
contributed to your problems.” exhibits who demonstrates normal
C. “This type of treatment helps you to cognition
confront your fears by exposing you to
the feared object abruptly. 9. Which intervention would be typical of a nurse
D. “This type of treatment will help you using cognitive-behavioral approach to a client
relax and develop new coping skills.” experiencing low self-esteem?

46
47

A. Use of unconditional positive regard C. refer the client to the psychiatrist


B. Analysis of free association D. refer the matter to the police
C. Classical conditioning
D. Examination of negative thought Situation: Rose seeks psychiatric consultation because of
patterns intense fear of flying in an airplane which has greatly
affected her chances of success in her job.
10. Which of the following therapies has been
strongly advocated for the treatment of post- 16. The most common defense mechanism used by
traumatic stress disorders? phobic clients is:
A. ECT A. Supression
B. Group Therapy B. Denial
C. Hypnotherapy C. Rationalization
D. Psychoanalysis D. Displacement

11. The nurse knows that in group therapy, the 17. The goal of the therapy in phobia is:
maximum number of members to include is: A. Change her lifestyle
A. 4 B. Ignore tension producing situation
B. 8 C. Change her reaction towards anxiety
C. 10 D. Eliminate fear producing situations
D. 16
18. The therapy most effective for client’s with
12. The nurse is providing information to a client phobia is:
with the use of disulfiram (antabuse) for the A. Hypnotherapy
treatment of alcohol abuse. The nurse B. Cognitive therapy
understands that this form of therapy works on C. Group therapy
what principle? D. Behavior therapy
A. Negative Reinforcement
B. Operant Conditioning 19. The fear and anxiety related to phobia is said to
C. Aversion Therapy be abruptly decreased when the patient is
D. Gestalt therapy exposed to what is feared through:
A. Guided Imagery
13. A biological or medical approach in treating B. Systematic desensitization
psychiatric patient is: C. Flooding
A. Million therapy D. Hypotherapy
B. Behavioral therapy
C. Somatic therapy 20. Based on the presence of symptom, the
D. Psychotherapy appropriate nursing diagnosis is:
A. Self-esteem disturbance
14. Which of these nursing actions belong to the B. Activity intolerance
secondary level of preventive intervention? C. Impaired adjustment
A. Providing mental health consultation to D. Ineffective individual coping
health care providers
B. Providing emergency psychiatric Situation: Mang Jose, 39 year old farmer, unmarried, had
services been confined in the National center for mental health
C. Being politically active in relation to for three years with a diagnosis of schizophrenia.
mental health issues
D. Providing mental health education to 21. The most common defense mechanism used by
members of the community a paranoid client is:
A. Displacement
15. When the nurse identifies a client who has B. Rationalization
attempted to commit suicide the nurse should: C. Suppression
A. call a priest D. Projection
B. counsel the client
22. When Mang Jose says to you: “The voices are is best described in one of the following
telling me bad things again!” The best response statements:
is: A. Unacceptable feelings or behavior are
A. “Whose voices are those?” kept out of awareness by developing the
B. “I doubt what the voices are telling you” opposite behavior or emotion
C. “I do not hear the voice you say you B. Consciously unacceptable instinctual
hear” drives are diverted into personally and
D. “Are you sure you hear these voices?” socially acceptable channels
C. Something unacceptable already done
23. A relevant nursing diagnosis for clients with is symbolically acted out in reverse
auditory hallucination is: D. Transfer of emotions associated with a
A. Sensory perceptual alteration particular person, object or situation to
B. Altered thought process another less threatening person, object
C. Impaired social interaction or situation
D. Impaired verbal communication
29. To be more effective, the nurse who cares for
24. During mealtime, Jose refused to eat telling that persons with obsessive compulsive disorder
the food was poisoned. The nurse should: must possess one of the following qualities:
A. Ignore his remark A. Compassion
B. Offer him food in his own container B. Patience
C. Show him how irrational his thinking is C. Consistency
D. Respect his refusal to eat D. Friendliness

25. When communicating with Jose, The nurse 30. Persons with OCD usually manifest:
considers the following except: A. Fear
A. Be warm and enthusiastic B. Apathy
B. Refrain from touching Jose C. Suspiciousness
C. Do not argue regarding his hallucination D. Anxiety
and delusion
D. Use simple, clear language Situation: The patient who is depressed will undergo
electroconvulsive therapy.
Situation: Gringo seeks psychiatric counselling for his
ritualistic behavior of counting his money as many as 10 31. Studies on biological depression support
times before leaving home. electroconvulsive therapy as a mode of
treatment. The rationale is:
26. An initial appropriate nursing diagnosis is: A. ECT produces massive brain damage
A. Impaired social interaction which destroys the specific area
B. Ineffective individual coping containing memories related to the
C. Impaired adjustment events surrounding the development of
D. Anxiety Moderate psychotic condition
B. The treatment serves as a symbolic
27. Obsessive compulsive disorder is BEST described punishment for the client who feels
by: guilty and worthless
A. Uncontrollable impulse to perform an C. ECT relieves depression psychologically
act or ritual repeatedly by increasing the norepinephrine level
B. Persistent thoughts D. ECT is seen as a life-threatening
C. Recurring unwanted and disturbing experience and depressed patients
thought alternating with a behavior mobilize all their bodily defences to deal
D. Pathological persistence of unwilled with this attack.
thought, feeling or impulse
32. The preparation of a patient for ECT ideally is
28. The defense mechanism used by persons with MOST similar to preparation for a patient for:
obsessive compulsive disorder is undoing and it A. electroencephalogram

48
49
Myasthenic crisis is a life-
B. general anesthesia A. Emotional crisis threatening condition that
C. X-ray B. Cholinergic crisis occurs when the muscles
D. electrocardiogram C. Menopausal crisis that control breathing
D. Myasthenia crisis become too weak to work.
33. Which of the following is a possible side effect
which you will discuss with the patient? 40. If you are not extra careful and by chance you
A. hemorrhage within the brain give over medication, this would lead to;
B. encephalitis A. Cholinergic crisis A cholinergic
crisis develops as a result
C. robot-like body stiffness B. Menopausal crisis of overstimulation of
D. confusion, disorientation and short C. Emotional crisis nicotinic and muscarinic
receptors at the
term memory loss D. Myasthenia crisis neuromuscular junctions.

34. Informed consent is necessary for the treatment Situation: Rosanna 20 y/o unmarried patient believes
for involuntary clients. When this cannot be that the toilet for the female patient in contaminated
obtained, permission may be taken from the: with AIDS virus and refuses to use it unless she flushes it
A. social worker three times and wipes the seat same number of times
B. next of kin or guardian with antiseptic solution.
C. doctor
D. chief nurse 41. The fear of using “contaminated” toilet seat can
be attributed to Rosanna’s inability to;
35. After ECT, the nurse should do this action before A. Adjust to a strange environment
giving the client fluids, food or medication: B. Express her anxiety
A. assess the gag reflex C. Develop the sense of trust in other
B. next of kin or guardian person
C. assess the sensorium D. Control unacceptable impulses or
D. check O2 Sat with a pulse oximeter feelings

Situation: Mrs Ethel Agustin 50 y/o, teacher is afflicted 42. Assessment data upon admission help the nurse
with myasthenia gravis. myasthenia gravis is chronic autoimmune,
neuromuscular disease that causes
to identify this appropriate nursing diagnosis
weakness in the skeletal muscles A. Ineffective denial
36. Looking at Mrs Agustin, your assessment would B. Impaired adjustment
include the following except; C. Ineffective individual coping
A. Nystagmus D. Impaired social interaction
B. Difficulty of hearing
C. Weakness of the levator palpebrae 43. An effective nursing intervention to help Rosana
D. Weakness of the ocular muscle is;
A. Convincing her to use the toilet after the
37. In an effort to combat complications which nurse has used it first
might occur relatives should he taught; B. Explaining to her that AIDS cannot be
A. Checking cardiac rate transmitted by using the toilet
B. Taking blood pressure reading C. Allowing her to flush and clear the
C. Techniques of oxygen inhalation toilet seat until she can manage her
D. Administration of oxygen inhalation anxiety
D. Explaining to her how AIDS is
38. The drug of choice for her condition is; transmitted
A. Prostigmine
B. Morphine 44. The goal for treatment for Rosana must be
C. Codeine directed toward helping her to;
D. Prednisone A. Walk freely about her past experience
B. Develop trusting relationship with others
39. As her nurse, you have to be cautious about C. Gain insight that her behaviour is due
administration of medication, if she is under to feeling of anxiety
medicated this can cause; D. Accept the environment unconditionally
B. Self-esteem disturbance
45. Psychotherapy which is prescribed for Rosana is C. Ineffective individual coping
described as; D. Defensive coping
A. Establishing an environment adapted to
an individual patient needs 50. Most appropriate nursing intervention for a
B. Sustained interaction between the client with suspicious behavior is one of the
therapist and client to help her develop following;
more functional behaviour A. Talk to the client constantly to reinforce
C. Using dramatic techniques to portray reality
interpersonal conflicts B. Involve him in competitive activities
D. Biologic treatment for mental disorder C. Use Non Judgmental and Consistent
approach
Situation: Dennis 40 y/o married man, an electrical D. Project cheerfulness in interacting with
engineer was admitted with the diagnosis of paranoid the patient
disorders. He has become suspicious and distrustful 2
months before admission. Upon admission, he kept on Situation: Clients with Bipolar disorder receives a very
saying, “my wife has been planning to kill me.” high nursing attention due to the increasing rate of
suicide related to the illness.
46. A paranoid individual who cannot accept the
guilt demonstrate one of the following defense 51. The nurse is assigned to care for a recently
mechanism; admitted client who has attempted suicide.
A. Denial What should the nurse do?
B. Projection A. Search the client's belongings and room
C. Rationalization carefully for items that could be used to
D. Displacement attempt suicide.
B. Express trust that the client won't cause
47. One morning, Dennis was seen tilting his head as self-harm while in the facility.
if he was listening to someone. An appropriate C. Respect the client's privacy by not
nursing intervention would be; searching any belongings.
A. Tell him to socialize with other patient to D. Remind all staff members to check on
divert his attention the client frequently.
B. Involve him in group activities
C. Address him by name to ask if he is 52. In planning activities for the depressed client,
hearing voices again especially during the early stages of
D. Request for an order of antipsychotic hospitalization, which of the following plan is
medicine best?
A. Provide an activity that is quiet and
48. When he says, “these voices are telling me my solitary to avoid increased fatigue such
wife is going to kill me.” A therapeutic as working on a puzzle and reading a
communication of the nurse is which one of the book.
following; B. Plan nothing until the client asks to
A. “i do not hear the voices you say you participate in the milieu
hear” C. Offer the client a menu of daily activities
B. “are you really sure you heard those and ask the client to participate in all of
voices?” them
C. “I do not think you heard those D. Provide a structured daily program of
voices?” activities and encourage the client to
D. “Whose voices are those?” participate

49. The nurse confirms that Dennis is manifesting 53. A client with a diagnosis of major depression,
auditory hallucination. The appropriate nursing recurrent with psychotic features is admitted to
diagnosis she identifiesis; the mental health unit. To create a safe
A. Sensory perceptual alteration environment for the client, the nurse most

50
51

importantly devises a plan of care that deals The nurse would initially:
specifically with the clients: A. Ask the client to leave the group session
A. Disturbed thought process B. Tell the client that she will not be
B. Imbalanced nutrition allowed to attend any more group
C. Self-Care Deficit sessions
D. Deficient Knowledge C. Tell the client that she needs to allow
other client in a group time to talk
54. The client is taking a Tricyclic anti-depressant, D. Ask another nurse to escort the client
which of the following is an example of TCA? out of the group session
A. Paxil
B. Nardil 59. A professional artist is admitted to the
C. Zoloft psychiatric unit for treatment of bipolar
D. Pamelor disorder. During the last 2 weeks, the client has
created 154 paintings, slept only 2 to 3 hours
55. A client visits the physician's office to seek every 2 days, and lost 18 lb (8.2 kg). Based on
treatment for depression, feelings of Maslow's hierarchy of needs, what should the
hopelessness, poor appetite, insomnia, fatigue, nurse provide this client with first?
low self-esteem, poor concentration, and A. The opportunity to explore family
difficulty making decisions. The client states that dynamics
these symptoms began at least 2 years ago. B. Help with re-establishing a normal
Based on this report, the nurse suspects: sleep pattern
A. cyclothymic disorder. C. Experiences that build self-esteem
B. Bipolar disorder D. Art materials and equipment
C. major depression.
D. dysthymic disorder. 60. The physician orders lithium carbonate
(Lithonate) for a client who's in the manic phase
56. The nurse is planning activities for a client who of bipolar disorder. During lithium therapy, the
has bipolar disorder, which aggressive social nurse should watch for which adverse reactions?
behaviour. Which of the following activities A. Anxiety, restlessness, and sleep
would be most appropriate for this client? disturbance
A. Ping Pong B. Nausea, diarrhea, tremor, and lethargy
B. Linen delivery C. Constipation, lethargy, and ataxia
C. Chess D. Weakness, tremor, and urine retention
D. Basketball
Situation – Annie has a morbid fear of heights. She asks
57. The nurse assesses a client with admitted the nurse what desensitization therapy is:
diagnosis of bipolar affective disorder, mania.
The symptom presented by the client that 61. The accurate information of the nurse of the
requires the nurse’s immediate intervention is goal of desensitization is:
the client’s: A. To help the clients relax and
A. Outlandish behaviour and inappropriate progressively work up a list of anxiety
dress provoking situations through imagery.
B. Grandiose delusion of being a royal B. To provide corrective emotional
descendant of king arthut experiences through a one-to-one
C. Nonstop physical activity and poor intensive relationship.
nutritional intake C. To help clients in a group therapy setting
D. Constant incessant talking that includes to take on specific roles and reenact in
sexual topics and teasing the staff front of an audience, situations in which
interpersonal conflict is involved.
58. A nurse is conducting a group therapy session D. To help clients cope with their problems
and during the session, A client with mania by learning behaviors that are more
consistently talks and dominates the group. The functional and be better equipped to
behaviour is disrupting the group interaction. face reality and make decisions.
problem. After the identification of the research
62. It is essential in desensitization for the patient problem, which of the following should be done?
to: A. Methodology
A. Have rapport with the therapist B. Acknowledgement
B. Use deep breathing or another C. Review of related literature
relaxation technique D. Formulate hypothesis
C. Assess one’s self for the need of an
anxiolytic drug 68. Which of the following communicate the results
D. Work through unresolved unconscious of the research to the readers. They facilitate the
conflicts description of the data.
A. Hypothesis
63. In this level of anxiety, cognitive capacity B. Research problem
diminishes. Focus becomes limited and client C. Statistics
experiences tunnel vision. Physical signs of D. Tables and Graphs
anxiety become more pronounced.
A. Severe anxiety 69. In Quantitative date, which of the following is
B. Mild anxiety described as the distance in the scoring unites of
C. Panic the variable from the highest to the lower?
D. Moderate anxiety A. Frequency
B. Median
64. Antianxiety medications should be used with C. Mean
extreme caution because long term use can lead D. Range
to:
A. Parkinsonian like syndrome 70. This expresses the variability of the data in
B. Hepatic failure reference to the mean. It provides as with a
C. Hypertensive crisis numerical estimate of how far, on the average
D. Risk of addiction the separate observation are from the mean:
A. Mode
65. The nursing management of anxiety related with B. Median
post-traumatic stress disorder includes all of the C. Standard deviation
following EXCEPT: D. Frequency
A. Encourage participation in recreation or
sports activities Situation: Survey and Statistics are important part of
B. Reassure client’s safety while touching research that is necessary to explain the characteristics
client of the population.
C. Speak in a calm soothing voice
D. Remain with the client while fear level is 71. According to the WHO statistics on the Homeless
high population around the world, which of the
following groups of people in the world
SITUATION: You are fortunate to be chosen as part of disproportionately represents the homeless
the research team in the hospital. A review of the population?
following IMPORTANT nursing concepts was made. A. Hispanics
B. Asians
66. As a professional, a nurse can do research for C. African Americans
varied reason except: D. Caucasians
A. Professional advancement through
research participation 72. All but one of the following is not a measure of
B. To validate results of new nursing Central Tendency:
modalities A. Mode
C. For financial gains B. Standard Deviation
D. To improve nursing care C. Variance
D. Range
67. Each nurse participants was asked to identify a

52
53

73. In the value: 87, 85, 88, 92, 90; what is the A. There is a control group
mean? B. There is an experimental group
A. 88.2 C. Selection of subjects in the control group
B. 88.4 is randomized
C. 87 D. There is a careful selection of subjects
D. 90 in the experimental group

74. In the value: 80, 80, 80, 82, 82, 90, 90, 100; what 80. The researcher implemented a medication
is the mode? regimen using a new type of combination drugs
A. 80 to manic patients while another group of manic
B. 82 patient receives the routine drugs. The
C. 90 researcher however handpicked the
D. 85.5 experimental group for they are the clients with
75. In the value: 80, 80, 10, 10, 25, 65, 100, 200; multiple episodes of bipolar disorder. The
what is the median? researcher utilized which research design?
A. 71.25 A. Quasi-experimental
B. 22.5 B. Phenomenological
C. 10 and 25 C. Pure experimental
D. 72.5 D. Longitudinal

76. Draw Lots, Lottery, Table of random numbers or Situation 19: As a nurse, you are expected to participate
a sampling that ensures that each element of the in initiating or participating in the conduct of research
population has an equal and independent studies to improve nursing practice. You to be updated
chance of being chosen is called: on the latest trends and issues affected the profession
A. Cluster and the best practices arrived at by the profession.
B. Stratified
C. Simple 81. You are interested to study the effects of
D. Systematic mediation and relaxation on the pain
experienced by cancer patients. What type of
77. An investigator wants to determine some of the variable is pain?
problems that are experienced by diabetic A. Dependent
clients when using an insulin pump. The B. Independent
investigator went into a clinic where he C. Correlational
personally knows several diabetic clients having D. Demographic
problem with insulin pump. The type of sampling
done by the investigator is called: 82. You would like to compare the support system
A. Probability of patient with chronic illness to those with
B. Snowball acute illness. How will you best state your
C. Purposive problem?
D. Incidental A. A descriptive study to compare the
support system of patients with chronic
78. If the researcher implemented a new structured illness and those with acute illness in
counselling program with a randomized group of terms of demographic data and
subject and a routine counselling program with knowledge about intervention.
another randomized group of subject, the B. The effects of the types of support
research is utilizing which design? system of patients with chronic illness
A. Quasi experimental and those with acute illness.
B. Comparative C. A comparative analysis of the support
C. Experimental system of patients with chronic illness
D. Methodological and those with acute illness.
D. A study to compare the support system
79. Which of the following is not true about a Pure of patients with chronic illness and those
Experimental research? with acute illness.
E. What are the differences of the support 87. Which of the following studies is based on
system being received by patient with quantitative research?
chronic illness and patients with acute A. A study examining the bereavement
illness? process in spouse of clients with
terminal cancer
83. You would like to compare the support system B. A study exploring the factors influencing
of patients with chronic illness to those with weight control behaviour
acute illness. Considering that the hypothesis C. A Study measuring the effects of sleep
was: “Client’s with chronic illness have lesser deprivation on wound healing
support system than client’s with acute illness.” D. A study examining client’s feelings
What type of research is this? before, during and after bone marrow
A. Descriptive aspiration.
B. Correlational, Non experimental
C. Experimental 88. Which of the following studies is based on the
D. Quasi Experimental qualitative research?
A. A study examining clients’ reaction to
84. In any research study where individual persons stress after open heart surgery
are involved, it is important that an informed B. A study measuring nutrition and weight
consent of the study is obtained. The following loss/gain in clients with cancer
are essential information about the consent that C. A study examining oxygen levels after
you should disclose to the prospective subjects endotracheal suctioning
except: D. A study measuring differences in blood
A. Consent to incomplete disclosure pressure before, during and after
B. Description of benefits, risks and procedure
discomforts
C. Explanation of procedure 89. An 85 year old client in a nursing home tells a
D. Assurance of anonymity and nurse, “I signed the papers of that research
confidentiality study because the doctor was so insistent and I
want him to continue taking care for me” Which
85. In the Hypothesis: “The utilization of technology client right is being violated?
in teaching improves the retention and attention A. Right of self determination
of the nursing students.” Which is the B. Right to full disclosure
dependent variable? C. Right to privacy and confidentiality
A. Utilization of technology D. Right not to be harmed
B. Improvement in the retention and
attention 90. A supposition or system of ideas that is
C. Nursing students proposed to explain a given phenomenon best
D. Teaching defines:
A. A paradigm
Situation: You are actively practicing nurse who has just B. A theory
finished you graduate studies. You learned the value of C. A Concept
research and would like to utilize the knowledge and D. A conceptual framework
skills gained in the application of research to the nursing Situation: Mastery of research design determination is
service. The following questions apply to research. essential in passing the NLE.

86. Which type of research inquiry investigates the 91. Ana wants to know if the length of time she will
issues of human complexity (e.g understanding study for the board examination is proportional
the human expertise)? to her board rating. During the June 2008 board
A. Logical position examination, she studied for 6 months and
B. Positivism gained 68%, On the next board exam, she
C. Naturalistic inquiry studied for 6 months again for a total of 1 year
D. Quantitative research and gained 74%, On the third board exam, She
studied for 6 months for a total of 1 and a half

54
55

year and gained 82%. The research design she collected 100 random individuals and determine
used is: who is their favourite comedian actor. 50% said
A. Comparative Dolphy, 20% said Vic Sotto, while some
B. Experimental answered Joey de Leon, Allan K, Michael V.
C. Correlational Tonyo conducted what type of research study?
D. Qualitative A. Phenomenological
B. Non experimental
92. Anton was always eating high fat diet. You want C. Case Study
to determine if what will be the effect of high D. Survey
cholesterol food to Anton in the next 10 years.
You will use: 98. Jane visited a tribe located somewhere in China,
A. Comparative it is called the Shin Jea tribe. She studied the way
B. Historical of life, tradition and the societal structure of
C. Correlational these people. Jane will best use which research
D. Longitudinal design?
A. Historical
93. Community A was selected randomly as well as B. Phenomenological
community B, nurse Edna conducted teaching to C. Case Study
community A and assess if community A will D. Ethnographic
have a better status than community B. This is
an example of: 99. Anjoe researched on TB. Its transmission,
A. Comparative Causative agent and factors, treatment sign and
B. Experimental symptoms as well as medication and all other in
C. Correlational depth information about tuberculosis. This study
D. Qualitative is best suited for which research design?
A. Historical
94. Ana researched on the development of a new B. Phenomenological
way to measure intelligence by creating a 100 C. Case Study
item questionnaire that will assess the cognitive D. Ethnographic
skills of an individual. The design best suited for
this study is: 100. Diana is to conduct a study about the
A. Historical relationship of the number of family members in
B. Survey the household and the electricity bill. Which of
C. Methodological the following is the best research design suited
D. Case study for this study?
1. Descriptive
95. Gen is conducting a research study on how mark, 2. Exploratory
an AIDS client lives his life. A design suited for 3. Explanatory
this is: 4. Correlational
A. Historical 5. Comparative
B. Phenomenological 6. Experimental
C. Case Study A. 1,4
D. Ethnographic B. 2,5
C. 3,6
96. Marco is to perform a study about how nurses D. 1,5
perform surgical asepsis during World War II. A E. 2,4
design best for this study is:
A. Historical
B. Phenomenological
C. Case Study
D. Ethnographic

97. Tonyo conducts sampling at barangay 412. He

You might also like